WholeIssue 30 6 - CMS-SMC

64
* (x, y) (3, -4) (5, 0) (2, 4.58) x 2 + y 2 = 25 S = 2 2 - 1 2 2 × 3 2 - 1 3 2 × 4 2 - 1 4 2 ×···× 2004 2 - 1 2004 2 S 8338 50705 A B C A + B = C C x 2 + y 2 =1 y =7x +5 A B O AOB x (x 2 - 3x + 1) x+1 =1 f (a, b) a b f (1, 5) = 1 + 2 + 3 + 4 + 5 = 15 f (3, 6) = 3 + 4 + 5 + 6 = 18 f (133333, 533333)

Transcript of WholeIssue 30 6 - CMS-SMC

���������� ����� ���������

�����! #"%$'&)(+*,"-/.1032�45064,03798�:<;>=9?@4,;>.A=<BDC1;<7/4�BE;�BGF/0)H/?D;/I<.J0LKM4�CN7OBGF>CN4P0L8<CQBDC1;<7RIL:TS�UWVYX[ZN\N]_^<`�`3a/bced ;<H�:f;>gWhji�k<l�mnhji�k<oqp/isrMhjiut!l�mnhwv�x�y{zu|6}+CA.N.�I/0~H9?D034,037>BE0L8OBE;�;<790)H9?D0L�=97<CQ�<03?[4�CQB�:f?D032�8/0L?�}6F/;�450L798<4PCN7�45;>.N=<BDCJ;<7/4�I/03gq;<?D0)BGF/0�89032�8>.NCN7909b���F/0M890 d CJ4GC1;<7;>gnBGF/060L8<CQBE;<?WCJ4��97/2L.�b� 0�}+CN.A.�;<7<. :�H9?[CJ7>B!45;>.N=<BDCJ;<7/4!BD;�H/?D;/I<.J0LKM4sKM2�?E��0�86}+CABGF~2�7M2L4GBD0L?[CJ45�)� ∗ � CAg}60+?D0 d 03C �<0PBGF/03K�g1?D;<K�4GBD=n8/0L7>B[4�CJ7M��?[2�8/0 ��� ;<?_=9798903?��1;<?!0��>=/CQ�>23.1037>B �G� ;<?_CNg/}60?D0 d 03C �<0�2+=97<C1�>=n0645;>.N=<BDCJ;<7M;<?!26�30L7903?[2L.ACA�G2�BDC1;<7�b�YF>CN4�KO;<7>BGF�� 46�>=n0L4EBDC1;<7/4+2�?D0~BD2���0L7�g1?D;<K�BGF/0���;L�>2R� d ;�BDCN2���2�BGF'�L0L2���=n0d F<2�KMH/CJ;<7/4,F>CNHfF/0�.18MCN7���2L.ACNg12¡ ~;<7���2�: ��¢ � �L����£ b���F<2�79��4@BE;�¤YC d F<2�?D8f�+;<4,F>CN79;;>gu¥�23.1F/;>=94GC10�¦s7<CQ�<03?[4�CQB�:�2L798P§5;/F<7)¨_?[2�7>B!� d �L;>=n�3F>.ACJ76;>g>BGF/0�¦s7<C �<0L?[4GCAB{:6;>g��P0¡}©n?D=97/4D}+C d ��gq;<?!450L798>CJ79��BGF/0L450��<=n034GBDCJ;<7/4Lbªj«�¬P­¯®�°_«@±u²E­´³�­_±�µ·¶�¸@­�¹!º6¸#»�­s¼�¸ ½~¾�»f¿�«�º6ÀÂÁTº)¸@Ã�±�²G«�Ä+Ã

Å/ÆnÇ�È�ZA\N\�É�ÆL\�ÊLËOÌqÍ�Ë6Î�ÆL\N\{Æ/È�Z1Ï<ÐfÌ{Ë�ÏOVYX�Æ9ÑL\{Ë�ÒMÉ)Ì�Æ�Ð>Ë�ÌqÍ�Ë�X�ÓLÉ�ÓfÌ{Ë�ÓLÒ�Ô�U�Ì@ÌqÍ�ËË�Ï�Õ�Æ�Î9Ön`�Ò�Z1ÏuÇ/Ì{Ë�É�]�ÈPË�È�ZN\A\u×EÆL\N\{Ë�×5ÌYØ�ÆnÇLXWÓ�Ï<É�ÈPË�X_É,Í�Ë�Ë�ÌDÔ�Å/ÆnÇ�X�Ì�Ë�Ó�ÒÙÈ�ZA\N\9X�ËL×GË,Z�ÊLËÚ V�ÆLZ1ÏYÌJÉ@Î�Æ�XsË�Ó>×DÍf×EÆ�XEX�ËL×5Ì�ÓLÏ/É�ÈPË�X¡ÔÛ b c \1Ó<Ì[ÌJZ�×EË�V�ÆLZJÏuÌ�CN4@26H9;>CJ7>B

(x, y) � }6F/0L?D0�BGF/0 d ;/;<?D8>CJ7/2�BE034s2L?D06I/;�BGF�CN7>BE0L�30L?[43bÜ<;<?!0� L2LKOH<.10 � (3, −4)2L798

(5, 0)2L?D0+.N23B[BDC d 0+H9;>CJ7>B[4 � I>=<B (2, 4.58)

CJ4@79;�B�b¥@0�BD0L?DKMCN790�BGF/0T7<=9KRI/0L?�;>g�.N23B[BDC d 0�H9;>CJ7>B[4R;<7'BGF/0 d CJ? d =9K�gq0L?D037 d 0Ý;>g�BGF/0d CJ? d .J0x2 + y2 = 25

bÞ bT�L0�B

S =22 − 1

22× 32 − 1

32× 42 − 1

42× · · · × 20042 − 1

20042

b�ß3 �H/?D0L4�4S2L4�2g1?[2 d BDCJ;<7 � ?D0�8>= d 0L8�BD;~.1;�}6034GB_BD0L?DKO43bà b c VYÓ3\AZ1Ï�Õ>X�Æ�ÒfËPCN4�2)7<=9KOI903?!BGF<2�B@?D032�8<4@BGF/0)4�2�KO0�gq;<?�}�2�?D8<4�2�798RI<2 d ��}�2�?D8<4 �4�= d F�2�4

83382L798

50705b�L0�B

A2L798

BI90ágq;>=9?G��8<CJ��CAB�H923.NCN798/?D;<KO0L4 � 2L798�.10�B C

I/0j2â�<�<0L�58>C1��CQBH923.NCN798/?D;<KO09b!ãqgA + B = C � 890�BE03?EK�CJ790�23.N.nH9;<454GC1I>.10��>2L.A=n0L4s;>g C b½ b���F/0 d CJ? d .J0�}+CABGF�0��>=923BDCJ;<7

x2 +y2 = 1CN7>BE03?[4,0 d BD4�BGF/0s.NCN790

y = 7x+523BuB�}6;8<CN4GBDCN7 d B_Hn;>CN7>BD4 � A 2L798

Bb��L0�B

OI90sBGF/0 d 0L7>BE?D0�;>g<BGF/0 d CN? d .10/bäÜ�CN798�BGF/0PKO0L2L4�=9?D0;>g

∠AOBb

å b@¥@0�BD0L?DKMCN790�23.N.9CN7>BE0L�30L?[4x4�= d F)BGF<2�B

(x2 − 3x + 1)x+1 = 1b

æ b��L0�Bf(a, b)

8/0L79;�BD0@BGF/0s4G=9Kç;>g�BGF/0@CN7>BE0L�30L?[4WI90�B�}60�037a2L798

b � CN7 d .N=94GCQ�<0/bWÜ<;<?0� L2LKOH<.10 � f(1, 5) = 1+2+3+4+5 = 152L798

f(3, 6) = 3+4+5+6 = 18b¥@0�BD0L?DKMCN790�BGF/0P�>23.N=n0�;>g

f(133333, 533333)b

������ b c F/0G L2��3;<7O2�798M2�7R0��>=/CN.N23BD0L?[23.9BE?DCN2�79��.J0)F<2��<0)0��>=92L.�Hn03?DCNKR0�BE03?[4Lbsãqg�BGF/062L?D0L2;>gnBGF/06F/0G L2��3;<7�CJ4

6√

345�<=92L?D0+=97<CQBD4 � }6F<23BäCN4!BGF/0�2�?D0326;>gnBGF/0�BE?[CJ2L79��.10��

� b@¥@0�BD0L?DKMCN790�23.N.<�>23.N=n034�;>gxgq;<?_}6F>C d F

(1999x − 99)3 = (1234x − 56)3 + (765x − 43)3b

� b!�CN798M23.N.n45;>.N=<BDCJ;<7/4(x, y)

CJ7M?D032L.n7<=9KRI/0L?[4�BD;1

x+

1

y=

5

6�

x2y + xy2 = 30b

Û�� b!Ü�CN798~BGF/0+7<=9KRI/0L?�;>gY?D032L.n4,;>.A=<BDC1;<7/4äBE;)BGF/0�0��>=923BDCJ;<7sin(x) =

x

315

b

�P0G LBÝ}60 ��C �<0�BGF/0 ;�� d CN2L.O4,;>.A=<BDC1;<7/4'BE;çBGF/0 �L����� ��2�?[CABDCNKR0 ��2�BGF/0LKM23BDC d 4 ;<KMHn0�BDCABDCJ;<7� �L����� �<��¢����>��¢�£�� b

Þ ��� à�³�­s¿n²q±�²E¼�¸¯³�­_±�µ+¸@¼´­_±�²G°�Ã��M«+¼´À�¸_±u²q±�²G«�Ä� �����<������� ��!#"%$'&�(*)!,+-$.&�/10��2!,�3��!,�4"%$5��/6&�/7+8!,��9Y����:

Û b � F/037�2)g123BGF/03?�8>CJ4EBE?DCJI<=<BD0L4�2~7<=9KOI903?�;>g d 2L798<CJ0L4�2LKR;<79�)F>CN4 d F>CN.J8/?D037 � 0L2 d Fd F>CN.J8M?D0 d 03C �<0L4 ��¢6d 2�798>C1034s2�798~BGF/03?D0+CJ4s;<790+.J03gNB�;L�<03?�b�ãqg � F/;�}60,�<0L? � B�}6;)g1?DCJ0L798<4;{;>CJ7~BGF/06��?D;>=9HO2�798~BGF/0 d 2L798<CJ0L4s2L?D0�?D0L8<CN4GBE?[C1I>=<BE0L8 � BGF/037R032 d F d F>CN.J8M?D0 d 0�CQ�<034 ���d 2L798<CJ0L4@2�798~BGF/03?D0�2�?D0PBGF<?D0�0�.10�gNB�;L�<03?�b � F<2�BäCJ4!BGF/0PBE;�B[2L.n7<=9KOI903?@;>g d 2L798<CJ0L4<�= ÆL\�Ç/ÌJZ{Æ�Ï?>_�L0�B

xI90@BGF/0�7<=9KOI903?W;>g d 2�798>C1034ä2L798�.10�B

yI90@BGF/0�7<=9KOI903?W;>g d F>CA.18<?D0L7�179;�BäCJ7 d .A=n8<CN79�PBGF/0PB�}6;~g1?[C103798/4 � b��YF/037~}60�F<2��<0

x = 15y + 12L798

x = 11(y + 2) + 3b

�YF>=94 � 15y+1 = 11(y+2)+3}6F>C d F�:<CJ03.J8/4

y = 6b!��;�}

x = 15(6)+1 = 91b��037 d 0 � BGF/0PBE;�B[2L.n7<=9KOI903?@;>g d 2L798<CJ0L4�CJ4 91

bÞ bu-<;>=9? d F<2��<=n06037>BDC103?!4EBE?DC d BE03KR037>B!Hn;<4GCABDCAg

n � Hn;<45;<7/4f(n) =

(

4(1)2 − 1)

×(

4(2)2 − 1)

× · · · × (4n2 − 1)b

->2�?!0� L2LKOH<.10 � f(1) = 30�B

f(2) = 3 × 15 = 45b�9?D;>=>�<0L?9BD;>=<BE034Y.1034n�>2L.J03=9?[4�890

nHn;>=9?9.1034,�>=n03.A.1034

f(n)034GB9=97 d 2L?E?�@0�H/2�?[g12LCQB�b

= ÆL\�Ç/ÌJZ{Æ�Ï >n-/=/CN4,�>=n04k2 − 1 = (2k)2 − 1 = (2k − 1)(2k + 1) � ;<7M2

f(n) =(

2(1) − 1)(

2(1) + 1)

×(

2(2) − 1)(

2(2) + 1)

× · · ·× (2n − 1)(2n + 1)

= (1)(3) × (3)(5) × (5)(7) × · · ·× (2n − 3)(2n − 1) × (2n − 1)(2n + 1)

= 3252 · · · (2n − 1)2(2n + 1)

=(

(3)(5) · · · (2n − 1))2

(2n + 1)b

�����

-<;>=9?W�<=n0f(n)

4,;>CQB�=97 d 2�?D?�@0�H92L?Dg123CAB � CA.>g12L=<BW0�B�CN.<4G=.�MBW�>=n0 2n+145;>CAB�=97 d 2L?E?�@0H92L?Dg123CAB�b�3C�.10f79;<KRI<?D0fCJKMH923CJ?

2n + 1034GB�=97 d 2�?D?�@0�H/2�?[g12LCQB � CN.@0L4EB�.10 d 2L?E?�@0�8Y� =9779;<KRI<?D0@CNKOH/2LCN? � 8>CJ45;<7/4 2a+1

b<¥�2L7/4 d 0 d 2�4 � 2n+1 = (2a+1)2 = 4a2+4a+1 �89;<7 dn = 2a(a+1)

b<¤ @0 d CNH9?D;/�>=n0LKO0L7>B � 4GC n = 2a(a+1)H9;>=9?�=97 d 03?[B[2LCN7�037>BDC103?Hn;<4GCABDCAg

a � 2L.J;<?[4 2n+1 = (2a+1)2034GB�=97 d 2L?E?�@0PH/2�?[g12LCQB�b9�L0L4��>2L.J03=9?[4!8/0

nH9;>=9?.1034,�>=n03.A.1034

f(n)034GB@=97 d 2�?D?�@0~H92L?Dg123CABs45;<7>B�8/;<7 d 8/;<7/7�@0�034PH92L?

n = 2a(a + 1) �a = 1 � 2 � 3 � . . .

bà b c

10�GKR0�BE?D0f.N2�8/8903?�?D0L4EBD4~2���2LCN7/4GB�2R�<03?[BDC d 23.W}�23.N.�b'�YF/0fK�C18/��H9;>CJ7>B6;>g�BGF/0.J2�898/0L?äCJ4!B�}+C d 0�2L4@g12�?äg1?D;<K�BGF/06��?D;>=9798O2L4@CQBäCJ4@g1?D;<K�BGF/0P}�23.N.�b � F<23B�F/03CJ�3F�B@;<7BGF/0P}�2L.A.�8/;/0L4!BGF/0+.N2�8/8903?!?D032 d F.�

= ÆL\�Ç/ÌJZ{Æ�Ï?>��L0�B�BGF/0�Hn;>CN7>BD4P2�B�}6F>C d FOBGF/0~.N2�8/8903?@BD;>= d F/0L4BGF/0!}�23.N.L2�798�BGF/0���?D;>=9798�I/0A2�798

B � ?D0L4�Hn0 d BDCQ�<0�.Q: � 2L798.10�BCI/0�BGF/0sH9;>CJ7>Bu23B9}6F>C d F�BGF/0!}�23.N.�KO0�0�BD4uBGF/0���?D;>=9798�b�L0�B

PI90�BGF/0�KMCJ89�GHn;>CN7>Bä;>g<BGF/0�.J2�898/0L? � 2�7986.J0�B_H9;>CJ7>B DI90M;<7RBGF/06}�2L.A.�45;OBGF<2�B

PDCN4�H/2�?[23.N.J03.YBE;OBGF/0M��?D;>=9798�b�CN7/2L.A.Q: � .10�B E

I90RBGF/0�Hn;>CN7>B6;<7ÝBGF/0f��?D;>=9798'4G= d FTBGF<2�BPE

CJ4@H/2�?[23.N.J03./BE;6BGF/0P}�23.N.�b�L0�BxI906BGF/0~.J0L79�¡BGFf;>g

DPb+�YF/037�BGF/0).10379�¡BGF�;>g

EPCJ4

2xb_�3CN7 d 0sBGF/0s.J2�898/0L?uCJ4

10KO0�BE?D034W.J;<79� � }60�F<2��<0

|AP | = |BP | = 5b

.

.

..

.

..

.

..

.

..

.

..

..

.

..

.

..

.

..

.

..

.

..

..

.

..

.

..

.

..

.

..

.

..

.

..

..

.

..

.

..

.

..

.

..

.

..

..

.

..

.

..

.

..

.

..

.

..

..

.

..

.

..

.

..

.

..

.

..

..

.

..

.

..

.

..

.

..

.

..

..

.

..

.

..

.

..

.

..

.

..

.

..

..

.

..

.

..

.

..

.

..

.

..

..

.

..

.

..

.

..

.

..

.

..

..

.

..

.

..

.

..

.

..

.

..

..

.

..

.

..

.

..

.

..

.

..

..

.

..

.

..

.

..

.

..

.

..

.

..

..

.

..

.

..

.

..

.

..

.

..

..

.

..

.

..

.

..

.

..

.

..

..

.

..

.

..

.

..

.

..

.

..

..

.

..

.

..

.

..

.

..

..............................................................................................................................................................................................................................................................................................................................................................................................................................................................................................................................................

.

..

.

..

.

..

.

..

.

..

.

..

.

..

.

..

.

..

.

..

.

..

.

..

.

..

.

..

..

.

..

.

..

.

..

.

..

.

..

.

..

.

..

.

..

.

..

.

..

.

..

.

..

.

..

.

..

.

..

.

..

.

..

..

.

..

.

..

.

..

.

..

.

..

.

..

.

..

.

..

.

..

.

..

.

..

.

...........................................................................................

A

BC

D

E

P

�P;�}∠DAP = ∠EPB

2L798∠DPA = ∠EBP � }6F/0L7 d 0 � 4DAP

CJ4d ;<79��?[=n0L7>B�BD; 4EPBb6�YF>=94 � |DA| = |EP | = 2x

2�798 |EB| = |DP | = xbc H9H<.Q:>CJ79��BGF/0�->:�BGF<2��3;<?D032�7~�YF/0L;<?D0LK BE; 4DAP � }60�;/I>B[2LCN7 (2x)2 + x2 = 52 �}6F>C d F6:<CJ03.J8/4

x =√

5bW�3CJ7 d 0 |DA| = 2x

2�798 |CD| = |EP | = 2x � }606�30�B|AC| = 4x = 4

√5b

�YF/03?D03gq;<?D0 � BGF/0+.N2�8/8903?!?D032 d F/034s26F/03CJ�3F�B�;>g 4√5KR0�BE?D0L4s;<7~BGF/0P}�23.N.�b

½ b��9?D;>=>�<03?O=97 79;<KOI/?D0 �2 4�C   d F>C��u?D034�89;<7>BM.10'H/?D0LK�C103? d F>C��Y?D0á034GB � 0�B��>=/C890,�>C1037>B�BE?D;>CJ4Pgq;>CJ4�H<.N=94���?[2L798�4GC�.J0MH/?D0LK�C103? d F>C��u?D0O034GBP8�@0LH<.J2 d @0 �2M.{� 23=<BE?D0RI/;>=<BHn;>=9?�8/0,�<0L7<CN?ä.10 d F>C��u?D068/0L4�=97<CQB @0L43b= ÆL\�Ç/ÌJZ{Æ�Ï >���;>CQB

1abcde.10�79;<KOI/?D0O0L7��>=n0L4EBDC1;<7�b��@7�2

1abcde = 105 + x; �=

x = abcdeb��L;<?[45�<=n0+.J0�H/?D0LK�C103? d F>C��Y?D060L4EB�8�@03H/.N2 d @0 �2�.�� 2L=<BE?D06I/;>=<B � ;<7O;/I�BDC1037>B

abcde1 = 10x + 1b 0T79;>=>�<032L=j79;<KOI/?D0Ý034GB6BE?D;>CN4�gq;>CJ4MH/.A=94R��?[2L798j�>=n0�.10H9?D03KMCJ0L? � d 06�>=/CY8/;<7/790

10x + 1 = 3(105 + x) �7x = 3 × 105 − 1 �x = 42857

b�L0�79;<KOI/?D0 d F/03? d F�@060L4EB�8/;<7 d

142857b

å b!ß��>23.N=92�BE03

5 + 2√

13 +3

5 − 2√

13b

����£

= ÆL\�Ç/ÌJZ{Æ�Ï?>��L0�Bx =

3√

5 + 2√

132�798

y =3√

5 − 2√

13 � 2�798�.J0�B s = x + yb�YF>=94 � s CN4!BGF/0�?D0L�<=/CN?D0�8M4�=9K�b � 06F<2��<0

s3 = (x + y)3 = x3 + 3x2y + 3xy2 + y3 = x3 + y3 + 3xysb

�P;�}x3 = 5 + 2

√132L798

y3 = 5 − 2√

13 � 2�798xy =

3

(5 + 2√

13)(5 − 2√

13) = 3√

25 − 4(13) = 3√

−27 = −3b

��037 d 0 �s3 = 5 + 2

√13 + 5 − 2

√13 + 3(−3)s �}6F>C d F�4GCJKMH/.ACN�n0L4äBD;

s3 + 9s − 10 = 0b@©<:~CJ7/4�Hn0 d BDC1;<7 � s = 1

CN4@2�?D;/;�B!;>g9BGF>CJ40��>=923BDCJ;<7�b���F>=94 � s − 1CN4!2Pg12 d BE;<?W;>g

s3 +9s − 10b@©<:6.J;<79�P8>CQ�>CJ4GC1;<7 � }60�;/I>B[2LCN7

s2 + s + 102�4!BGF/0�;�BGF/0L?Wg12 d BD;<?�b���F/0L?D0�gq;<?D0 �

(s − 1)(s2 + s + 10) = 0b

�P;�}s2+s+10 = 0

F<2L4W79;�?D032L.�?D;/;�BD43bW��037 d 0 � BGF/0�;<7<.Q:+4,;>.A=<BDC1;<7PBE;PBGF/0�2�I/;L�<00��>=923BDCJ;<7MCN4s = 1

b��YF/03?D03gq;<?D0 �3

5 + 2√

13 +3

5 − 2√

13 = 1b

æ b<�9?D;>=>�<0L?WBD;>=<BE034�.1034@H923CJ?D034�8�� 037>BDC103?[4sHn;<4GCABDCAg14(x, y)

BE0�.N.J0L4s�>=n0x2 − 11y! = 2003

b�[->2L?�8�@0��n7<CQBDC1;<7 � 1! = 1 � 2! = 1 · 2 = 2 � 3! = 1 · 2 · 3 = 6 � 0�B d b �= ÆL\�Ç/ÌJZ{Æ�Ï >_�3C

x034GB6H/2LCN? � 8>CJ45;<7/4 x = 2k � 23.1;<?[4 x2 = (2k)2 = 4k2

0L4EB�=97KM=/.QBDCJH<.10�890��<=92�BE?D09bW¥�� 23=<BE?D0�H/2�?�B � 4�C x0L4EBsCNKOH/2LCN? � 8>CJ45;<7/4 x = 2k + 1 � 2L.J;<?[4

x2 = (2k + 1)2 = 4k2 + 4k + 1 = 4(k2 + k) + 10L4EBä=97OKM=/.QBDCJH<.1068/06�<=92�BE?D0H/.A=94W=97�b �@7 d ;<7 d .N=<B_�>=n0�Hn;>=9?�=97 d 03?[B[2LCN7)0L7>BDCJ0L?

m � x2 = 4m;>=

x2 = 4m+1b¥@0�H/.A=94 � .1;<?[45�<=n0 y ≥ 4 � y!

d ;<7>BDC1037>B!.J0+g12 d BE0�=9? £ b��L0�KO0LKOI/?D068906��23= d F/0890O.{�7@0L�<=92�BDC1;<7 � d � 0L4EB �2f8<CN?D0x2 − 11y! � 034GB+8/;<7 d 45;>CABP=97TKM=/.QBDCJH<.10�8/0��<=92�BE?D0 �4,;>CQB�=97RK�=/.ABDCNH/.J0~8/0~�>=923BE?D0)H<.N=94s=97�b�->2�? d ;<7>BE?D0 � .106KO0LKOI/?D0~8/0~8<?D;>CABD0 � d � 0L4EB �28<CN?D0

2003 � 0L4EBs=97fKM=/.QBDCJH<.10�890��<=92�BE?D0MH<.N=94�BE?D;>CJ43bW�>�7@0��>=923BDCJ;<7f7Y� 2O8/;<7 d H/2�4�8904,;>.A=<BDC1;<7/4s�>=92�798y ≥ 4

b�@7)��@03?DCA��0�=97OH/2�?W=97M.J0L4 d 2�4

y = 1 � 2 � 3 b�3Cy = 1 � ;<7á2 x2 − 11(1!) = 2003 � 8/;<7 d x2 = 2014

b �@7���@03?DCA��02LCN4 @03KR037>B��>=n02014

7Y� 0L4EB+H/2�4�=97 d 2�?D?�@0RH/2�?[g12LCQB�b!ãq.ä7Y� 2f89;<7 d H/2�46890R45;>.N=<BDCJ;<7/4.1;<?[45�<=n0y = 1

b�3Cy = 2 � ;<7Ý2 x2 − 11(2!) = 2003 � 8/;<7 d x2 = 2025

b 0�B[BE0Ogq;>CN46;<7BE?D;>=>�<0x = 45

bß<7<�97 � 4�C y = 3 � ;<7O2 x2 − 11(3!) = 2003 � 89;<7 d x2 = 2069b �@7~��@03?DCA��0�<=n0

20697Y� 0L4EB�H/2�4�=97 d 2L?E?�@0TH92L?Dg123CAB�b�ãq.�7Y� 2�89;<7 d H92L4R8/0Ý45;>.N=<BDCJ;<7/4�.1;<?[45�<=n0

y = 3b�>�7@0L�<=92�BDC1;<7MHn;<4�4 �0L890�8/;<7 d .J2�4,;>.A=<BDC1;<7~=97<CJ�<=n0

(x, y) = (45, 2)b

����¢

�P0G LBP}60���CQ�<0OBGF/0f;�� d CN2L.ä4,;>.A=<BDC1;<7/4+BD;�BGF/0 �L������� b §3b@©9.A=97989;<7â��2�BGF/0LKM23BDC d 4 ;<7>BD0L4EB �L����� �L£/� � �3£/��� � b

� µ�¸ ��� ¸@Ä@±u²G¸_±�µ������� ��[º6Ä���«�ij�­_±�µ+¸@¼´­_±�²G°�Ã��M«�Ä@±�¸@Ã�±Û bW��;>.Q�<0 �

log2(9 − 2x) = 3 − xb

= ÆL\�Ç/ÌJZ{Æ�Ï?>�¦s4GCJ79��BGF/068/03�97<CABDCJ;<7O;>gnBGF/0�.1;/��2L?DCQBGF<K�gJ=97 d BDCJ;<7 � }606F<2��<0log2(9 − 2x) = 3 − x �

9 − 2x = 23−x �9 − 2x = 8 · 2−x �

9 · 2x − 22x = 8 �(2x)2 − 9 · 2x + 8 = 0 �(2x − 1)(2x − 8) = 0 �

2x = 1;<?

2x = 8 �x = 0

;<?x = 3

b F/0 d ��CN79�@BGF/034,0sCJ7+BGF/0�;<?DCJ��CJ7/23.<0��>=923BDCJ;<7 � }60s�n798�BGF<23BuBGF/0,:62L?D0PI/;�BGF�4,;>.A=<BDC1;<7/43bÞ bW��F/;�}�BGF<23B

(√

5 + 2)13 − (

√5 − 2)

13CN4@?[23BDCJ;<7/2L. � 2L798��9798�CAB[4ä�>2L.A=n09b

= ÆL\�Ç/ÌJZ{Æ�Ï?>ä�L0�Bx = (

√5 + 2)

13 − (

√5 − 2)

13b��YF/037

x3 = (√

5 + 2) − 3(√

5 + 2)23 (

√5 − 2)

13

+ 3(√

5 + 2)13 (

√5 − 2)

23 − (

√5 − 2)

= 4 − 3(√

5 + 2)13 (

√5 − 2)

13

(

(√

5 + 2)13 − (

√5 − 2)

13

)

= 4 − 3(1)(x)b

��037 d 0 � (√5 + 2)13 − (

√5 − 2)

13CJ4@2���0L?D;�;>g

P (x) = x3 + 3x − 4 = (x − 1)(x2 + x + 4)}6F>C d F�F<2�4x = 1

2L4�CAB[4W;<7<.Q:+?D0L23.���03?D;ubu�YF/03?D03gq;<?D0 � (√5+2)13 −(

√5−2)

13 = 1

b��/Õ�Ôu�YF>CN4�H/?D;/I<.J0LK CJ4�4GCJK�CN.N2�?9BD;PH/?D;/I<.J0LK ¢ ;<7�BGF/0 �����>� ��2L?DCQBDCJKO0P��2�BGF/0LKM23BDC d 4 ;<KMHn0�BDCABDCJ;<7 � gq;<?_}6F>C d F�2�4,;>.A=<BDC1;<76}�2�4s��C �<0L7O;<7~BGF/0�H/?D0 d 0�8>CJ79��H92��309b �à b!ãqg

a3 + b3 = 42�798

ab = 2

3� }6F/0L?D0 a

2L798b2�?D0+?D0L23. � �9798 a + b

b= ÆL\�Ç/ÌJZ{Æ�Ï?>W��;�BE0��n?[4EBWBGF<23B

(a + b)3 = a3 + 3a2b + 3ab2 + b3 = a3 + b3 + 3ab(a + b)b

�L0�Bx = a + b

b��YF/037x3 = 4 + 3

(

2

3

)

x � }6F>C d F6:>C10�.18<4x3 − 2x − 4 = 0 �

(x − 2)(x2 + 2x + 2) = 0b

.J0L2L?D. : � x = 2CJ4!BGF/0�;<7<.Q:�?D0L23.�?D;/;�B�b�037 d 0 � a + b = 2

b

�����½ b!Ü�CN798

x � y � 2L798 z4�= d F6BGF<23B�}6F/037M2�7�:�;<790�;>g9BGF/0LK�CN4@2�898/0�8)BE;6BGF/0+H/?D;/8<= d B;>gnBGF/06;�BGF/03?WB�}6; � BGF/0�?D034�=/.QBäCJ4 2

b= ÆL\�Ç/ÌJZ{Æ�Ï?>�©<:�4[:<KOKO0�BE?{: � }60fK�=94GB)F<2��<0 x = y = z

b�F/0L7 0L2 d F'0��>=923BDCJ;<7I90 d ;<KO0L4x + x2 = 2 �

BGF<2�B+CN4 � x + x2 − 2 = 0 � ;<? (x − 1)(x + 2) = 0b�YF>=94 � x = 1

;<?x = −2

b!�CJ7/23.N. : � x = y = z = 1;<?

x = y = z = −2b

å bäãqga � b � 2L798 c

2�?D0sBGF/0sBGF<?D0�0s��0L?D;<4!;>gP (x) = x3 −x2 +x−2 � �n798 a+ b+ c2�798

a2 + b2 + c2b

= ÆL\�Ç/ÌJZ{Æ�Ï?>_ãqga � b � 2L798 c

2L?D0�BGF/0+?D;/;�BD4s;>gnBGF/0���CQ�<037 d =nI<C d � BGF/037x3 − x2 + x − 2 = (x − a)(x − b)(x − c)

= x3 − (a + b + c)x2 + (ab + ac + bc)x − abcb

ß/�>=923BDCN79� d ;/0�� d CJ0L7>B[4P;>gYBGF/0x2� BD0L?DKO4 � }60)450�0+BGF<23B a + b + c = 1

b����<=92L?DCN79�BGF>CJ4 � }60)�30�B a2 + b2 + c2 + 2(ab + ac + bc) = 1b�©9=<Bs0��>=923BDCN79� d ;/0�� d CJ0L7>B[4;>gnBGF/0

x� BD0L?DKO4s��CQ�<034

ab + ac + bc = 1b��+0L7 d 0 �

a2 + b2 + c2 + 2(1) = 1 �a2 + b2 + c2 = −1

bæ b!ãqg

sin x + cos x =

2 +√

3

2� }+CQBGF 0 < x < π

2� �n798 x

b= ÆL\�Ç/ÌJZ{Æ�Ï?>����>=92�?[CJ79�PBGF/0���CQ�<037R0L�<=92�BDC1;<7 � }60��30�B

sin2 x + 2 sin x cos x + cos2 x =2 +

√3

2�

1 + 2 sin x cos x = 1 +

√3

2�

sin 2x =

√3

2�

}6F/0L?D00 < 2x < π

b~�YF/03?D03gq;<?D0 � 2x = π/3;<?

2x = 2π/3b���F/0L7

x = π/6;<?x = π/3

b� b�-9?D;3�<0+BGF<2�BäB�}6; d ;<7/4,0 d =<BDCQ�<06;/8/8OH9;<4�CQBDCQ�<0+CN7>BE0L�30L?[4 d 2L7/79;�B@F<2��<0�2 d ;<KOKO;<7g12 d BD;<?�;�BGF/03?WBGF<2L7

1b

= ÆL\�Ç/ÌJZ{Æ�Ï?>n�>}6; d ;<7/450 d =<BDC �<0P;/8986H9;<4�CQBDCQ�<0sCJ7>BD0��303?[4 d 2L7~I/0@}�?DCQB[BE037)2L42k−1

2L7982k + 1

gq;<?ä4,;<KO0+Hn;<4GCABDC �<0�CJ7>BD0��303?kbW�3=9H9H9;<4,0

pCN4s2 d ;<KMKR;<7�g12 d BE;<?!;>g

2k − 12�7982k + 1

b_��F/0L72k − 1 = mp

2L7982k + 1 = np �

gq;<?�4,;<KO06H9;<4�CQBDCQ�<0+CN7>BE0L�30L?[4m2L798

nbs�3=nI�BE?[2 d BDCN79��BGF/0��n?[4EB@0��>=923BDCJ;<7Og1?D;<K�BGF/04,0 d ;<798O��C �<0L4

2 = (n − m)pb_�YF>=94 � p = 1

;<?p = 2

b�©9=<Bp 6= 2 � 4GCJ7 d 0 2k − 12�798

2k + 12�?D06;/8/8�b���F/0L?D0�gq;<?D0 � p = 1

b

��� �� b6�9?DCN2�79��.J0

ABCF<2�4��<03?[BDC d 034

A(3, 1) � B(5, 7) � 2�798 C(1, y)bM�CJ798f2L.A.

y45;BGF<23B!2L79��.10

CCJ4@2�?[C1�3F�B!2�79��.J09b

= ÆL\�Ç/ÌJZ{Æ�Ï?>BC ⊥ AC �

mBC =−1

mAC�

7 − y

4=

−2

1 − y�

7 − 8y + y2 = −8 �y2 − 8y + 15 = 0 �

(y − 5)(y − 3) = 0 �y = 5

;<?y = 3

b -

6

........................................................................................................................................................................................................................................................................................................................................................................................................................................................................................................................................................................................................

A(3, 1)

B(5, 7)

C(1, y)

................................................................................

� b ã{7jBGF/0'8<CN2���?[2LK BE; BGF/0'?[C1�3F�B � PQ = 8 �TS = 12 � 2L798 QS = 20

báÜ�CJ798QR

4,;�BGF<2�B∠PRT

CN4@2�?DCJ�3F�B�2L79��.10/b= ÆL\�Ç/ÌJZ{Æ�Ï?>ä�L0�B

QR = xb��YF/037

(PT )2 = (PR)2 + (RT )2

= (64 + x2) +(

144 + (20 − x)2)

= 2x2 − 40x + 608b

..

..

..

..

..

..

..

..

..

..

..

..

..

..

..

..

..

..

..

..

..

..

..

..

..

..

..

..

..

..

..

..

..

..

..

..

..

..

..

..

..

..

..

..

..

..

..

..

..

..

..

..

..

..

..

..

..

..

..

..

..

..

..

..

..

..

..

..

..

..

..

..

..

..

..

..

..

..

..

..

..

.

..

.

..

.

.

..

.

..

.

..

.

..

.

..

.

..

.

.

..

.

..

.

..

.

..

.

..

.

..

.

.

..

.

..

.

..

.

..

.

..

.

..

.

.

..

.

..

.

..

.

..

.

..

.

..

.

.

..

.

..

.

..

.

..

.

..

.

..

.

.

..

.

..

................................................................................................................................................................................................................................................................................................................................................................................................................................................................................................................................................................................................................................................................................................................

..

.

.

..

.

.

..

.....................................

......................... . . . . . . . . . . .............

P

Q R S

T

c .J45; � (PT )2 = 202 + 42 = 416b���037 d 0 �

2x2 − 40x + 608 = 416 �x2 − 20x + 96 = 0 �

(x − 8)(x − 12) = 0 �x = 8

;<?x = 12

b ........................................................................................................................................................................................................................................................................................................................................................................................................................................................................................................................................................................

..

.

..

..

.

..

.

..

.

..

.

..

.

..

.

..

.

..

.

..

.

..

.

..

.

..

.

..

.

..

.

..

.

..

.

..

.

..

.

..

.

..

.

..

.

..

.

..

.

..

.

..

.

..

.

..

.

..

.

..

.

..

.

..

.

..

.

..

.

..

.

..

.

..

.

..

.

..

.

..

.

..

.

..

.

..

.

..

.

..

.

..

.

..

.

..

.

..

.

..

.

..

.

..

.

..

.

..

.

..

...............................................................................................................................................................................................................................................

.....................................................................................................................................................................................................................................................................................................................................................................................................................................................................................................................................................................................

.

..

.

.

..

.

..

.

.....................................

......................... . . . . . . . . . . ...............

..............................................

......................... . . . . . . . . . . .............

P

Q R S

T

U

8 8

4

x 20 − x

20

Û�� b c 4,�>=92�?D0�;>g!4GC18/02CN4�H/.N2 d 0�8f}+CQBGFÝ;<790O4�CJ890R;<7Ý2MBD2L79�30L7>BPBD;�2 d CN? d .10R;>g?[2�8>CN=94

545;�BGF<23BnBGF/0@4,�>=92�?D0�.AC1034_;>=<B[4�CJ890äBGF/0 d CJ? d .J0 � 2�798+;<790ä�<03?[BD0� �;>gLBGF/0@45�<=92L?D0.NCJ0L4+;<7RBGF/0 d CN? d .10/b c .NCN790~CJ4+8<?[2�}�7fg1?D;<K¯BGF/0 d 0L7>BE?D0O;>gBGF/0 d CJ? d .J0�BGF<?D;>=n�3F~BGF/0P�<0L?�BE0G �;>gnBGF/0�45�<=92L?D0�BGF<23BäCJ4@79;�B;<7�BGF/0�BD2L79�30L7>B�2L798�79;�B�;<7PBGF/0 d CJ? d .J09bu�YF>CN4�.NCN790 d =<BD4�BGF/0BD2L79�30L7>B�23B�2�H9;>CJ7>B

Tb~ãqg�BGF/06BD2L79�30L7>B�KR0L0�B[4�BGF/0 d CJ? d .J023B

S � �9798~BGF/0+.J0L79�¡BGFM;>gnBGF/0�.NCN790�450���KO0L7>B TSb

.

.

..

.

..

.

.

..

.

..

.

..

.

..

.

..

.

.

..

.

..

.

..

.

..

.

..

.

..

.

.

..

.

..

.

..

.

..

.

..

.

.

..

.

..

.

..

.

..

.

........................................................................................................................................................................................................................................................................................................................... ..............................................................

.

..

.

.

..

.

.

..

.

..

.

.

..

.

.

..

.

..

.

.

.

.

.

..

..

.

..

..

.

..

..

..

.

..

.

..

..

.

..

..

..

..

..

..

..

..

..

..

..

..

..

..

..

..........................................................

...................................................................................................................................................................................................................................................

...........................................................................................................

S T= ÆL\�Ç/ÌJZ{Æ�Ï?> 4TQR2L798 4TSO

2�?D0+4�CNKMCA.J2L?�b��YF>=94 �T Q

2=

T Q + 6

5�

5TQ = 2TQ + 12 �TQ = 4

b��037 d 0 � TS = 6 + 4 = 10

b ............................................................................................................................................................................................................................................................................................................................................................................................................................................................................................................................................................ ..

.

..

.

..

.

..

.

..

.

..

.

..

.

..

.

..

.

..

.

..

.

..

.

..

.

..

.....................................................................................

.......................................................................................................................................................................................... ..

.

..

.

..

.

..

.

..

.

..

.

..

..

..

..

..

..

..

.

..

..

..

..

..

..

..

..

..

..

..

..

.

..

..

..

..

..

..

..

..

..

..

..

.......................................................................

....................................................................................................................................................................................................................................................................................................................................................

......................................................................................................................................................

O

Q

R

S T

2 2

3

4

4 2

5

��� �� ������� � ������#�� � ������ �

��23BGF/03KO2�BDC d 2L._��2�:9F/03K´I/0���2L7OCJ7 �� �� � 2�4���h��������������������3y �Ex"!�#%$��3y�&�x�#�'$ (�)�*l+�-,"��."�/�<x<x�y0�'$ (213$4� 54��#%6�� ��*�./�%!0(���$"��6<b@ã1B d ;<7>BDCJ7<=n034 � }+CQBGF~BGF/0�452LKR0603KOH9F<2�4GCJ4 �2�4@2�7�CN7>BE0L��?[2L.nH92L?[B�;>g87:9<;<=�>@?:ACBED<F�?:AHGJI�K"9�;'FMLNGOAPB2>@?:APBED�F�?:AJGHI�?<QR>@?:S'BED<F�b�YF/0f��2�:9F/03K%ß/8>CABD;<?sCJ4+��F<2�}�7�¨W;/8<CN7T� �!B[BD2�}�2 2L?D.J0�BD;<7'¥!CJ4EBE?DC d BP� d F/;/;>.©�;<2�?D8 � b��YF/0 c 454GCJ4EBD2L7>Bs��2�:9F/03K¯ß/8>CABD;<?WCN4ä§5;/F<7M¨W?[2L7>B�� d �L;>=n�3F>.ACJ7M�q¦s7<C �<0L?[4GCAB{:;>gs�P0¡}©n?[=97/4E}+C d � � bO�YF/0R;�BGF/03?P4EBD2 � KO0LKOI903?[4�2L?D0��32�?D?{: ¤YC d 0O�q¦s7<C �<0L?[4GCAB{:T;>g� 23BD0L?[.1;/; �E� ¥�2�7M��2 d/T CN7/79;<7~� �!B[BD2�}�2 2L?D.J0�BD;<7O¥!CN4GBE?[C d B�� d F/;/;>.�©�;<2�?D8 �G� 2�798~ã{2�7U_2�798/0L?,©9=9?D�3FR�1¦s7<CQ�<03?[4�CQB�:M;>g � 23BD0L?[.1;/; � b

V WYX[Z]\2^ _a`+bdcfe"\2^hg

i4j�k�l m mOjonqp4r�k�sutwvHx<p'soyzj<tJtwv{j}|�r�szs�r/mHk't~lOr<p"szx�k%�2��v{r"��m<�j�yzj�sY�4ku��v��j%s�j�p�t+p k�y]�j�v{rx"|%x<p4t�mOj�'�P�/�� �/�+�/���{� �R�/�<�'�����'j�s�s�r/mHk"t~lOr<p's�v{j"�� k'j�s�x���v4�j�s � j<tJtOjq��x"tOj�p4j�s�j�v{r<p�t���vJl-s�j�sj�p � r<y�EtHj+�4k'j�s�� l m4p4r�k�s�v{j�s�tHjN�4k�tHj�y��4s�x"|ox<p�t�m-x���k"��m l � x"t~lOr<pq�:j%s�s�r/mHk"t~lOr<p's����  x'�4k"j2��v{r"��m��j�y¡j}s�j�vHxf��k"��m l��jf��x<p"s�mOj�s¢�:j<k��fm-x<p'£�k'j�s¤r�¥ � lOjom mOj�s¢�4k � x<p"x'��x¦ x<p'£/m-x/l-s§j<t3¨~vHx<pE�� x/l-sH©��Rª�x<p's�mOj%sp k�y]�j�v{r�s§«%¬�­�¬�®Yj<tN¯°¬�mC� x<p�£/m-x/l-s+��v��j � �j��:j�vHx�mOj�¨�vHx<pE�� x/l-sP¬j<tE��x<p's�mOj%s�p�k<y]�j�v{r�s8±�¬�²�¬�³�j<tE´�¬4mOj8¨�vHx<pE�� x/l-sR��v��j � �j/�:j�vHxNmC� x<p'£/m-x/l-s/���x}v��j���x � t~lOr<p¤s�r�k   x/lOtHjYv{j�yzj�v � lOj�v�µ/j�x<p4¶°·2x<v �¡¸ j�vPvHlOj�v�j<t·2x<v°t~l~pº¹�r/mw��s�tOj%l~p�¬��:jmC��»0p�lO|�j�vHs{lOt��jN�:j�·fr<p4twv��j%x/m ¬E� � x"|�r/l~v�twvHx'�4k�lOt4mOj�sR��v{r"��m��j�y¡j%s��

³ Û å�� bR¼nXGÆ3V�Æ�É�½ËsVYÓ�X�¾�Ë3ÊLË�Ïz¿LÇ�X[Z�À×[]�Á3ÓLÐLX�Ë�Ñ�]8Â/X�Æ�Ó<ÌJZ�Ë�b�@7 4�2LCQB��>=n0'.1034�gq;<?EK�=/.1034

a = x2 − y2 � b = 2xy � c = x2 + y24,;<7>B@=<BDCN.J0L4�Hn;>=9?�BE?D;>=>�<03?�8/0L4�4,;>.A=<BDC1;<7/4P0L7>BDC �0L?D034�8/0).��7@0��>=923BDCJ;<7a2 + b2 = c2

bï2��EBG�ECN.�89034�gq;<?DKM=/.J0L4�450LKOI<.N2�I>.1034�Hn;>=9?OBE?D;>=>�<0L?�8/0L4�45;>.N=<BDCJ;<7/4T037>BDC �03?D0L4T890.��7@0��>=923BDCJ;<7

a2 + ab + b2 = c2�

³ Û å � bR¼nXGÆ3V�Æ�É�½ËsVYÓ�XÄäÔHÅ!Ô = Ô = Ó3É3ÌqXDØL]�Æ�Ó�Ï<Ð�Ó�\�Æ�XGË�]�Ç{Ï�Õ9Ë>b�12 � ß>7�7Y� =<BDCN.ACJ4�2�7>B �<=n0 89034â79;<KRI<?D0L4â7/23BD=9?D0�.J4 �BE?D;>=>�<0L?�.J0L4u.J;<79��=n03=9?[4�89034 d'È;�B�@034�8/0L4�8<C�� @03?D0L7>B[4uBE?[CJ2L79��.1034?D0 d B[2�79��.J0L4s2LH9H/2�?[23CJ4�452L7>B�8<2�7/[email protected]+����=9?D0/b

........................................................................................................................................................................................................................................................................................................................................................................................................................................................................................................................................................................................

.

..

.

..

.

..

..

.

..

.

..

.

..

.

..

.

..

.

..

.

..

..

.

..

.

..

.

..

.

..

.

..

.

..

.

..

..

.

..

.

..

.

..

.

..

.

..

.

..

.

..

..

.

..

.

..

.

..

.

..

.

..

.

..

.

..

.

..

..

.

..

.

..

.

..

.

..

.

..

.

..

.

..

..

.

..

.

.

..

.

..

....................................................................................................................................................................................................................................................................................................................................................................................................................................................................................................................................................

................... . . . . . . . ..........

.

.

.

.

.

.

.

.

............................

. . . . . . . . ............................ ....................................

25

�qI � �9?D;>=>�<03?�=9790�0G �H/?D0L4�4�CJ;<7 �2@�>23.10�=9?[4ä0L7>BDC �0L?D034WH9;>=9?�?D03KOH<.J2 d 03?u.10s79;<KRI<?D025

890ÝBE0�.N.J0 45;<?[BD0á�>=u� ;<7 H/=/CN45450á0379�30L798<?D0L?R=9790'CN7<�n7<CQB @0á8/0 ?D0 d B[2�79��.J0L4�8<2�7/4.1034,�>=n03.N4s0L4EBäCJ7/4 d ?DCQBä=97~BE?DCN2�79��.J0�?D0 d BD2L79��.10 � BE;>=94 �2 d'È;�B�@034s037>BDC103?[4Lb³ Û å � bR¼nXGÆ3V�Æ�É�½ËsVYÓ�XW\°É ½� Ê�Ç3ZQV�Ë�Õ9ËzË Ó�Ø�Í�Ë�Ò�b

ß<4EBG�GCA.�H9;<454GC1I>.10�8�� 2L?E?[2L79�30L?�.1034~79;<KRI<?D0L41 � 2 � 3 � 4 � 5 � 6 � 7 � 8 � 9 � 10 .10.1;<79�+8Y� =97 d 03? d .10�890+g123Ìd ;<7O�<=n0+.N2645;<KOKO0�89068/03=� +�<;>CN4�CN7/4s�<=n0�. d ;<79�<=n034s4,;>CQBä=9779;<KRI<?D0�H9?D03KMCJ0L? �

���" ³ Û æ � bR¼nXGÆ3V�Æ�É�½ËsVYÓ�XW\°É ½� Ê�Ç3ZQV�Ë�Õ9ËzË Ó�Ø�Í�Ë�Ò�b

;<KMKR0f4GBE?[2�B �0��30 � ��2�H9;>. @0L;<7 @0�B[2LCQB+��03?)8/0�45;<7'2L?EK4@0�0��>=u� CA.ä�<;>=/.N2LCQB+BE? �0L48<CN4 d CNH/.ACJ7�@0L0 � 0L7>BE?[2 È � 7�@0L0~0�B�KR;/I>CN.J09b�¥�2�7/4s=9790~89064,034�I<23B[2LCA.N.J0L4 d ;<7>BE?D06.J0L4 c .A.NC @0L4 �CN.u8�@0 d CJ8/2~8/0+�>2L?DCJ0L?!.{� 2L?E?[2L79�30LKO0L7>Bs8906450L4@g12�7>B[2�4�4�CN7/4Lb c =f8�@0�I>=<B � d 0�=� >� d C�gJ=9?D037>B��?D;>=9H�@034!037540

?[2L79� @0�034�@0���23.1034�H/=/CN4_CN.�.1034�?D;>=9H/2�037

105?[2L79� @0�034�@0���23.1034 � H9;>=9?�n7/23.103KR037>B�.1034���?D;>=9Hn03?s0L7

216?[2L79� @0�034 @0���23.1034Lb��L0679;<KOI/?D0)8/0)45;>.18<23B[4 @0�B[2LCQB!.10H/.A=94sH90�BDCQB!79;<KRI<?D0�?D0L798<2�7>B!Hn;<4�4�CJI<.J0L4@890�BD03.N4@2�?D?[2�79�303KR037>BD43b

;<KOI<CJ0L7):M0L7M2��>23CABG�ECN.��³ Û æ Û bR¼nXGÆ3V�Æ�É�½ËsVYÓ�X3¿LÔ���Ó�\�Ì{Ë�X���Ø�Ï�×DÍ/]/U�ÌqÍYË�Ï/É�]��9U!]�� = UPb

¥�2�7/4�=97�BE?[CJ2L79��.10~CN4,; d �0�.10ABC

890�I/2L4,0BC = 1 � .N2MH/?D0LK�C �03?D0M890M8/03=� 8/?D;>CQBE034@H/2�?[23.N. �03.J0L4 �2+.N2�I/2L4,0 d ;>=9H90�.1034 d'È;�B @0L4 @0���23=� 

AB0�B

AC2L=� �Hn;>CN7>BD4

P0�B

Q � .N2�4,0 d ;<798/0�2L=� 6Hn;>CN7>BD4 R0�B

Sb9�L2 d F<2L7>B@�>=n0+.J268<CN4GB[2�7 d 068/0

PQ�2

BC034GB

10�B!�>=n0�.J0+?[2LH9H9;<?[BPQ/RS

0L4EB @0���23.92L=M79;<KOI/?D0+8Y� ;<? 1

2(1 +

√5) � BE?D;>=>�<0L?W.{� 23CJ?D08<=�BE?[2�H �0���0

PQSRb

³ Û æ+Þ bR¼nXGÆ3V�Æ�É�½ËsVYÓ�X�¾�Ë3ÊLË�Ïz¿LÇ�X[Z�À×[]�Á3ÓLÐLX�Ë�Ñ�]8Â/X�Æ�Ó<ÌJZ�Ë�b�@7��<03=<B d ;<7/4EBE?D=/CN?D0OH/2�4 �2RH/2�4�=9790O2L?EKM23BD=9?D0 d =nI>C1�>=n0O2��<0 d 89034�BDCJ�30L4�890K È[email protected];<79��=n0�=9? � 8<CN4,;<7/4 1 K �0�BE?D0 � �>=/C>H903=>�<037>B�4�� 03KRI/; È � BD0L?u2L=� P0G LBE?�@0LK�CAB�@034�.�� =9790890O.�� 2L=<BE?D0RHn;>=9?Pgq;<?EKO0L?P=9790R4GBE?[= d BD=9?D0 d =nI>C1�>=n0�?DCJ��C18/09bäãq.!0L7Tg123=<B

12H9;>=9?�=97H9?D03KMCJ0L? d =nI90f89;<7>B�.�� 2�? È0�BD0fKO0L4G=9?D0

1K�b c d ;<798<CQBDC1;<7�8Y� 0L7<.J0,�<0L?+.J0L4+BDC1�3034~�>=/Cgq;<7>B)8/;>=nI<.J0�=94�2��30 � F>=/CAB6890 d 034 d =nI/0L4~H903=>�<037>B64�� 2�8/2LH<BD0L?�23�n7'8Y� ;/I>BD0L7<CN?+=9790890�=� �C �0LKO0�2�?DKO2�BD=9?D0�8/;<7>B�.{� 2L? È0�BE0�KR034�=9?D0 d 0�B[BD0�gq;>CJ4

2KTb<ãq./0L7�g12L=<B

27Hn;>=9?�=9790BE?D;>CJ4GC �03KR0+2L?EKM23BD=9?D068/0

3KÙ8�� 2L? È0�BE0 � 0�B d b

� =n03.A.10�0L4EBs.J2�H/.A=94+��?[2�798/0M2L?EKM23BD=9?D0 d =nI>C1�>=n0O�>=u� ;<7�H<=/CJ4�4,0 d ;<7/4GBE?[=/CJ?D0�4GC.J2P.1;<79��=n0�=9?_BD;�BD23.10�89034äBDCJ�30L4�8<CN45H9;<7<C1I>.1034!4�=.�O?[2LCQBäH9;>=9?W?D03.AC103?_.N2PBD0L?D?D0 �2�.N2�.A=9790 �4,;>CQB384 000

��K �b b b b b b b b b b b b b b b b b b b b b b b b b b b b b b b b b b b b b b b b b b b b b b b b b b b b b b b b b b b b b b b b b³ Û å�� b+¼nXGÆ3V�Æ�É�ËLÕ�Ñ�Ø�¾�Ë3ÊLË�Ïz¿LÇ�X[Z�À×[]�Á3ÓLÐLX�Ë�Ñ�]8Â/X�Æ�Ó<ÌJZ1Ó/b

�YF/0Mgq;<?DKM=/.N2�4a = x2 − y2 � b = 2xy � c = x2 + y2

2L?D0R��79;�}�7�BD;�I90=94,0�gJ=/.�gq;<?uH/?D;/8<= d CN79��CJ7>BD0��303?u4,;>.A=<BDC1;<7/4_;>g�BGF/0�0L�<=92�BDC1;<7a2+b2 = c2

b c ?D0�BGF/03?D04�CNKMCA.J2L?Wgq;<?EK�=/.J2L4�gq;<?äCN7>BE0L�30L?!45;>.N=<BDCJ;<7/4s;>gnBGF/0�0��>=923BDCJ;<7a2 + ab + b2 = c2

�³ Û å � b+¼nXGÆ3V�Æ�É�ËLÕ�Ñ�ØuÄäÔHÅ!Ô = Ô = Ó3É3ÌqXDØL]�Æ�Ó�Ï<Ð�Ó�\�Æ�XGË�]�Ç{Ï�Õ�Z1Ó/b

�12 � ¥@0�BE03?EK�CJ790�BGF/0�.J0L79�¡BGF<4�;>g<BGF/0P4�CJ89034�;>g<BGF/0�>2�?[C1;>=94ä?[C1�3F�B�BE?[CJ2L79��.1034WCN7�BGF/0�����=9?D0 � ��CQ�<037�BGF<2�B�BGF/0.10379�¡BGF<4s2�?D0�CJ7>BD0��303?[4Lb.....................................................................................................................................................................................................................

..

.

..

..

.

..

.

..

.

..

..

.

..

.

..

.

..

.

..

.

..

.

..

.

..

..

.

..

.

..

.

..

.

..

.

..

.

..

.

..

..

.

..

.

..

.

..

.

..

.

..

.

..

.

..

..

.

..

.

..

.

..

.

..

.

..

.

..

.

..

..

.

..

.

..

.

..

.

..

.

..

.

..

.

..

.

..

..

.

..

.

......................................................................................................................................................................................................................................................................................................................................................................................................................................................................................................................................................................................................................................................................................................................................................................................................................................................................................................................

................... . . . . . . . ..........

.

.

.

.

.

.

.

.

............................

. . . . . . . . ............................ ....................................

25

�qI � Ü�CJ798)2L7)CN7>BE0L�30L?E�D�>23.N=n0L8~0� �H9?D03454GC1;<76CN7~H<.J2 d 0P;>g/BGF/0P7<=9KOI903? 25 4,;+BGF<2�B2�7+CN7<�n7<CQBE0s7<=9KOI903?�;>g/8>C��u0L?D037>B�?D0 d B[2�79��.J0L4_KM2�:�I90��303790L?[2�BE0L8 � 0L2 d F�}+CQBGF+2�?[C1�3F�BBE?DCN2�79��.J0�CJ7/4 d ?DCJI90L8M2L4@45F/;�}�7 � 4G= d F)BGF<2�BWBGF/0�.J0L79�¡BGF<4s;>g9BGF/0+4�CJ89034s;>g9BGF/0��>2�?[C1;>=94?DCJ�3F�BäBE?[CJ2L79��.1034@2�?D0�CJ7>BD0��303?[4Lb

���L�³ Û å � b+¼nXGÆ3V�Æ�É�ËLÕ�Ñ�ØMÌqÍ�ËzË'Ó�ØLÍYË�Ò = ÌJÓ��Wb

ã{4�CAB�Hn;<4�4�CJI<.J0~BD;�2L?E?[2L79�30�BGF/0O7<=9KOI903?[41 � 2 � 3 � 4 � 5 � 6 � 7 � 8 � 9 � 10 CN7�2d CJ? d .J0�45;)BGF<2�BWBGF/0�4G=9KÙ;>gY2L7�:~B�}6;~7903CJ�3F/I9;>=9?[4@CN4@26H/?DCNKR0+7<=9KRI/0L? �

³ Û æ � b+¼nXGÆ3V�Æ�É�ËLÕ�Ñ�ØMÌqÍ�ËzË'Ó�ØLÍYË�Ò = ÌJÓ��Wb�YF/0R4GBE?[2�BE0L��CJ4EB~��2�H9;>.10L;<7�}�2�4�H9?D;>=n8�;>g�F>CJ462L?EK):Y� 4~8>CJ4 d CJH<.NCN790 � BE?[2LCN7<CJ79� �2�798OKR;/I>CN.ACAB{:�b�ã{7R2)I/2�B[BD.1062���2LCN7/4GBäBGF/0�ß�=9?D;<Hn032�7R23.N.AC1034�F/0��>2�?[C10L8�BGF/062�?D?[2�79�30L�KR037>B�;>gYF>CN4@45;>.18>C103?[4Lbäã{7<CQBDCJ23.N. : � BGF/0�4,;>.J8<CJ0L?[4W}603?D0+2L?E?[2L79�30�8~CJ7 540

0��>=92L.9?D;�}�4�CJ7+BGF/0P450 d ;<79862�?D?[2�79�303KR037>B � BGF/0L?D0@}60L?D0 105

0��>=92L.<?D;�}�4�2�798�CJ7+BGF/0�BGF>CN?D8 d 2L4,0 �

2160��>=92L.�?D;�}�43b��YF/0�7<=9KOI903?P;>g_4,;>.J8<CJ0L?[4s}�2L4�BGF/0�4�KO23.N.J0L4EB�7<=9KOI903?sBGF<23B�0L79�2�I>.10L8M2L.A.n4�= d F�2L?E?[2L79�30LKO0L7>B[4Lb��;�} KO2L7�:M4,;>.J8<CJ0L?[4W}60L?D0PBGF/0L?D0��

³ Û æ Û b+¼nXGÆ3V�Æ�É�ËLÕ�Ñ�Ø�¿LÔ���Ó�\�Ì{Ë�X���Ø�Ï�×DÍ/]/U�ÌqÍYË�Ï/É�] �9U!] � = UPb�9?DCN2�79��.J0

ABCCN4!CJ45;<4 d 03.J0L4 � }+CABGF AB = AC

2L798BC = 1

b �@7~BGF/0�4�CJ89034AB

2L798AC

2�?D06H9;>CJ7>B[4P2�798

Q � ?D0L4�Hn0 d BDCQ�<0�.Q: � 4G= d F�BGF<2�B PQ ‖ BC2L798MBGF/08<CN4GB[2�7 d 0�g1?D;<K

PQBD;

BCCJ4

1b �@7�BGF/0�4,0L��KR037>BD4

AP2L798

AQ2L?D0MH9;>CJ7>B[4

R2�798S � ?D0345H90 d BDC �<03. : � 4�= d F)BGF<23B RS ‖ PQ

bãqgPQ/RS = (1 +

√5)/2

�NBGF/0 �3;>.J89037 ?[2�BDC1; �E� �9798âBGF/0'2�?D032á;>g�BGF/0BE?[2�H90���;>CJ8PQSR

b³ Û æ+Þ b+¼nXGÆ3V�Æ�É�ËLÕ�Ñ�Ø�¾�Ë3ÊLË�Ïz¿LÇ�X[Z�À×[]�Á3ÓLÐLX�Ë�Ñ�]8Â/X�Æ�Ó<ÌJZ1Ó/b

¤Y;/8/41KÙCJ7R.J0L79�¡BGFR2L?D0)=9450�8MBE;�I>=/CN.J8�2~?DCJ��C18 d =nI<C d g1?[2�KO0�}6;<?E��b��>}60�.Q�<0?D;/8/4�2�?D0�790�0L890L8�BE;fI<=/CA.18f2 d =nI90O;>g_4�CJ890

1KTb�©<:��<B[BDCJ79�6BD;/�30�BGF/03?

8;>g�BGF/034,0=97<CAB d =nI/0L4 � 2 d =nI<C d g1?[2LKR0¡}6;<?G� d 2�7)I90 d ;<7/4GBE?[= d BE0L8+BGF<23B_F<2�4ä4GC18/0 2

K�b�©<:�=94GCJ79�27;>g>BGF/0�=97<CQB d =nI/0L4 � 2 d =nI>C d g1?[2�KO0�}6;<?E� d 2�7)I90 d ;<7/4GBE?[= d BE0L8+BGF<23B_F<2L4ä4�CJ890 3

K�bß<2 d F�BDCNKR0�BGF/0�=97<CAB d =nI9034�2�?D0 d ;<KOI<CN790�8 � BGF/0L?D062�?D068<=9H<.NC d 2�BE06?D;/8/4s2L.J;<79�BGF/0�0L89�3034�}6F/03?D0�BGF/0 d =nI9034O�<B)BE;/�30�BGF/0L?¡bç�YF/0Ý8<=9H<.NC d 2�BE034RKM2�:âI90T?D0LKO;L�<0L8�J.J0L2��<CN79��;<790+?D;/8)}6F/03?D0�BGF/03?D0P}60L?D0+H9?D0,�>C1;>=94G.Q:)B�}6; � 2�798M?D0L�G=9450�8M03.N4,0¡}6F/0L?D0/b��;�} .N2�?D�30~2 d =nI>C d g1?[2�KO0�}6;<?E� d 2L7�I90 d ?D0323BD0�8�CN7RBGF/0�KO2L7/790L?�8/0L4 d ?DCJI90L82�I/;L�<0PCAg/BGF/0�BD;�BD23./.J0L79�¡BGF~;>g/BGF/0�2��>2LCA.J2�I<.J0P?D;/8/4_}6;>=/.18 d ;<7/790 d B_BGF/0�0L2L?[BGF�BE;�BGF/0KR;/;<7 � 2L454G=9KMCN79��2)8>CJ4EBD2L7 d 0�;>g 384 000��K#I90�B�}60�037~BGF/0LK �

�����V W�XºZ]\2^ �ubfe������'b��]g

³ �+à b+¼nX�Æ3V�Æ�É�Ë�ÕRÑ�ØMÌqÍYËqË Ó�Ø�Í�Ë�Ò = ÌJÓ��WbÜ�C �<0~I/23.N.N4�7<=9KOI903?D0�8

1BD;

52L?D0)H/=<B@CN7>BE;O2�I9;� nb c I<2L.A.YCJ4�8/?[2�}�7�23B@?[2L79�89;<K � CAB[4s7<=9KRI/0L?@?D0 d ;<?D8/0�8 � 2�798�BGF/0)I<2L.A.Y?D0�BD=9?E790L8MBD;~BGF/0)I9;� nb!�YF>CN4�H9?D; d 0L4�4sCN4?D0LH90L2�BE0L8O=97>BDCA.��>�<067<=9KOI903?[4PF<2��<0)I90L0L7O?D0 d ;<?D890L8�b�ãqg�BGF/064�=9KÙ;>g�BGF/06?D0 d ;<?D890L87<=9KRI/0L?[4�CJ4

15 � }6F<23BPCN4�BGF/0RH9?D;/I<2�I>CN.ACAB{:�BGF<2�B�BGF/0R7<=9KRI/0L? 3}�2L4)8<?[2�}�7Ý0L2 d FBDCJKO0 �

= ÆL\�Ç/ÌJZ{Æ�Ï�Ñ�Ø �_Ë�ÏYË3Ê¡ZËLÊLË �LÓ3\{Æ�ÏYÕ9Ë¡]�Ë'ÓLÉ�É�Ë�Ø3]��N¾)b�YF/0�B[2�I>.10+I/03.J;�}�4,F/;�}�4WBGF/0�7<=9KOI903?!;>g<}�2�:/4�;>g�8/?[2�}+CJ79���>�<0+I<2L.A.J4_}+CABGF)24�=9K#;>g

15b_��F/0�BE;�B[2L.n7<=9KOI903?�;>g9}�2�:<4�CJ4

381b_��F/0L?D0�gq;<?D0 � BGF/0�H9?D;/I<2�I>CN.ACAB{:~BGF<2�B0,�<0L?{:O8<?[2�} }�2L4@2

3CJ4 1

381

b©n23.N.N4 �s=9KRI/0L?�;>g � 2�:/4 ©n23.N.N4 �s=9KRI/0L?�;>g � 2�:/4

5 � 5 � 3 � 1 � 1 5!

2!2!= 30 5 � 3 � 3 � 2 � 2 5!

2!2!= 30

5 � 5 � 2 � 2 � 1 5!

2!2!= 30 4 � 4 � 4 � 2 � 1 5!

3!= 20

5 � 4 � 4 � 1 � 1 5!

2!2!= 30 4 � 4 � 3 � 3 � 1 5!

2!2!= 30

5 � 4 � 3 � 2 � 1 5! = 120 4 � 4 � 3 � 2 � 2 5!

2!2!= 30

5 � 4 � 2 � 2 � 2 5!

3!= 20 4 � 3 � 3 � 3 � 2 5!

3!= 20

5 � 3 � 3 � 3 � 1 5!

3!= 20 3 � 3 � 3 � 3 � 3 5!

5!= 1

������ ������������������� �"!#� �$���&%('��)�������*�,+ ��-�.³ �@½ b+¼nX�Æ3V�Æ�É�Ë�ÕRÑ�ØMÌqÍYËqË Ó�Ø�Í�Ë�Ò = ÌJÓ��Wb

;<7/4GC18/0L?YBGF/0@B�}6;�gJ=97 d BDC1;<7/4f(x) = x2 −2ax+1

2�798g(x) = 2b(a−x) �}6F/0L?D0

a � b � x ∈ / b � 0s}+CA.N. d ;<7/4�CJ8903?W0L2 d F)H/2LCN?W;>g d ;<7/4GB[2�7>B[4 a2L798

b2�4ä2�Hn;>CN7>B

(a, b)CJ7)BGF/0

ab� H/.N2�790/bs�L0�B

AI90PBGF/0+450�B!;>g�Hn;>CN7>BD4

(a, b)gq;<?�}6F>C d F)BGF/0���?[2LHnF<4;>g

y = f(x)2L798

y = g(x)8/;MÏYÆ9Ì�CN7>BE03?[4,0 d B�b!Ü�CN798~BGF/0+2�?D032);>g

Ab

= ÆL\�Ç/ÌJZ{Æ�Ï�Ñ�Ø �äÇ3É3ÌJÓ<ÊLÆ�ÄuX[Z1Ò10<Ë�XD] ��Ï>Z�ÊLË�X[ÉGZ�Õ>Ó>Õ}Â�U �E ��]3Æ_Ç<Ë�ÏYÆ�ÉWUäZJX�Ë¡É5]<U�X�Ð<Ë�ÏuÌJZJÏ9Ó/b�YF/0���?[2�H9F<4�;>g

y = f(x)2�798

y = g(x)89;~79;�BWCJ7>BD0L?[450 d B�}6F/0L7~BGF/0�0L�<=92��BDC1;<7

x2 − 2ax + 1 = 2b(a − x)F<2L4�79;�4,;>.A=<BDC1;<7�b+�YF>CN4+F<2LH9H90L7/4PCNg_2L798�;<7<.Q:CNg

b2 + a2 < 1bP��037 d 0 � A CJ4@BGF/0~;<H90L7�8>CJ45�M;>gu?[2�8>CN=94

1d 037>BE?D0�8R2�B!BGF/0~;<?[C1��CN7�b�YF>=94 � BGF/0+2�?D032);>g A CN4

πb

23� ���4��� �,+ ��-6587:9;�<5����=��>?� � � ����@�� A; )!<� ����BC����� A�D)EF.³ � å b+¼nX�Æ3V�Æ�É�Ë�ÕRÑ�ØMÌqÍYËqË Ó�Ø�Í�Ë�Ò = ÌJÓ��Wb

�CN798f26BE?DCN2�79��.J06}6F/;<450)CN7>BE0L�30L?s4�CJ89034P2L?D0)CJ7�2�?[CABGF<KO0�BDC d H/?D;/��?D0L4�4�CJ;<7O}+CQBGF2 d ;<KOKO;<7O8<C���03?D0L7 d 06;>g22L798)}6F>C d FMF<2�4@2�7M2L?D0L26;>g

336b

�����

= ÆL\�Ç/ÌJZ{Æ�Ï�Ñ�Ø �äÇ3É3ÌJÓ<ÊLÆ�ÄuX[Z1Ò10<Ë�XD] ��Ï>Z�ÊLË�X[ÉGZ�Õ>Ó>Õ}Â�U �E ��]3Æ_Ç<Ë�ÏYÆ�ÉWUäZJX�Ë¡É5]<U�X�Ð<Ë�ÏuÌJZJÏ9Ó/b�L0�B

a � a+2 � 2L798 a+4I/0@BGF/0s4�CJ89034W;>g�BGF/0@BE?[CJ2L79��.10/b�©<:)�+0L?D;<7Y� 4�Ü>;<?EK�=/.J2 �}606F<2��<0

336 =√

1

16(a + 6)(a + 2)(a − 2)(3a + 6)

b��037 d 0 � (a + 6)(a + 2)(a − 2)(3a + 6) = 16 · 3362 = 1806336 � }6F>C d FF<2�4�BGF/0+=97<CJ�<=n0�Hn;<4GCABDC �<0+CN7>BE0L�30L?!45;>.N=<BDCJ;<7

a = 26bW�YF/037�BGF/0�BE?[CJ2L79��.10�4GC18/0L4@2�?D0

26 � 28 � 2�798 30b

23� ���4��� �,+ ��-6587:9;�<5����=��>?� � � ����@�� A; )!<� ����BC����� A�D)EF.³ �Pæ b ¼nX�Æ3V�Æ�É�½Ë�VYÓ�X_\oÉ{½Ë"Ê�Ç3ZQV�Ë6Õ9ËzË'Ó�ØLÍYË�Ò�b

��;>CAB!890�=� )79;<KRI<?D0L4s0L7>BDCJ0L?[4@Hn;<4GCABDCAg14a0�B

bbu->2L?EK�Cn.J0L4@79;<KOI/?D034

a � 2a � 3a � . . . � (b − 1)a � ba

d ;<KOI<CJ0L7):O037M2��DBG�GCA.��>=/C�4,;>CJ0L7>B!8<C �<CN4�CJI<.J0L4�H92L?b�

= ÆL\�Ç/ÌJZ{Æ�Ï~VYÓLXÅ_Æ9Ñ/Ë�X,Ì8Æ9ZN\AZ1Ï<É<0�ZN]:�sÇ<ÌqX�Ë�ÒfÆ�ÏuÌ1]���ÂuÔ�3C

(a, b) = 1 � 23.1;<?[4�CN./:M264503=/.J0LKO0L7>B ba�<=/CY034GB�8<C �<CN4�CJI<.J0�H/2�?

bbYãq./:O0L7O289;<7 d

1b�3C(a, b) = b � 23.1;<?[4�BD;>=946.1034)79;<KRI<?D0L4)4,;<7>B68>CQ�>CJ4GC1I>.1034)H/2�? b

b�ãq.W:�0L7�289;<7 dbb�3C(a, b) = k � 2L.J;<?[4 a = ks

0�Bb = kt

2��<0 d(s, t) = 1

bs�32��>=n0L4EBDC1;<7890,�>C1037>B ��� ->2L?EK�C3.J0L4Y79;<KRI<?D0L4s � 2s � . . . � (kt−1)s � kts � d ;<KOI<CJ0L7�4,;<7>B�8<C �<CN4�CJI<.J0L4H92L?

t���f¥�2�7/46.N2f4G=/CABD0f890�79;<KOI/?D0f;<7TBE?D;>=>�<03?[2

ts � 2ts � . . . � (k − 1)ts � kts �89;<7 dk79;<KRI<?D0L4!4,;<7>BW8<C �<CN4�CJI<.J0L4äH92L?

tb c =<BE?D03KR037>Bä8<CQB � CN.�:~23=9?[2PBD;>=�;{;>=9?[4 (a, b)79;<KRI<?D0L4�8>CQ�>CJ4GC1I>.1034@H/2�?

b8<2�7/4�.J2+.ACJ4EBE0/b

� �&��!<� ��� �������� �"!��' ���!���� '<+ � �F� � B6@���� A����<�$+���� ���$-8'<-6E�2 � E � A;>;!<��������23� ������A<2 ���#����� � ���#.³ � � b+¼nX�Æ3V�Æ�É�Ë�ÕRÑ�ØMÌqÍYËqË Ó�Ø�Í�Ë�Ò = ÌJÓ��Wb

��0,�<0L7OH90�;<H<.10 � A � B � C � D � E � F � G � 2L?D06;<7R;<790�4GC18/06;>gu2�?DC �<0L?¡bä�/;O�30�B2 d ?D;<4�4_BGF/0�?DC �<0L?uBGF/0,:~F<2��<0P2P?D;�}6I/;<23B � I>=<B�CAB d 2L7~;<7<.Q:��<B�B�}6;�H90�;<H<.10�2�B_2�BDCNKR0/b�YF/0)BDCNKR034�BGF<23B!}6;>=/.18fI90~?D0��>=/CJ?D0L8�gq;<?@BGF/0~Hn0L;<H/.J0)BD;�?D;�} 2 d ?D;<4�4�CJ798>CQ�>C18>=92L.A.Q:2�?D01 � 2 � 3 � 5 � 10 � 15 � 2L798 20

KMCN7<=<BE034 � ?D0345H90 d BDC �<03. :�bá�+;�}60,�<0L? � }6F/0L7TB�}6;Hn0L;<H/.J0�2L?D0�CN7�BGF/0TI/;<23B � BGF/0fBDCJKO0fCAB+BD2���0L4)BGF/0LK%BE;'?D;�}�2 d ?D;<4�4~CJ4)BGF/0�452LKR02�4PBGF/0~BDCNKR0M790 d 03454�2�?{:�gq;<?sBGF/0O4G.1;�}603?�;>g_BGF/0~B�}6;RBD;�?D;�}�2 d ?D;<454PCJ798>CQ�>C18>=92L.A.Q:ubc 454G=9KMCN79�sBGF<2�BW79;);<790 d 2�7 d ?D;<454�}+CABGF/;>=<B�BGF/0�I9;<2�B � }6F<2�B_CN4WBGF/0PK�CJ7<CNKM=9KçBDCNKR0gq;<?!23.N.n450,�<0L7MH90�;<H<.10PBE;M�30�B!2 d ?D;<4�4!BGF/0�?[CQ�<03? �= ÆL\�Ç/ÌJZ{Æ�Ï�Ñ�Ø �_Ë�ÏYË3Ê¡ZËLÊLË �LÓ3\{Æ�ÏYÕ9Ë¡]�Ë'ÓLÉ�É�Ë�Ø3]��N¾)b

�YF/0�K�CJ7<CNKM=9K BDCNKR0PCN446KMCN7<=<BE034�2�798 d 2�7�I90�2 dGd ;<KOH<.NCN4,F/0L8~2�4!45F/;�}�7~CN7BGF/0�BD2�I<.J0�I90�.1;�} �

����� ?D;<4�4 �nCNKR0 ¤Y0�BD=9?E7 �nCNKR0A � B 2 A 1

F � G 20 B 2

A � B 2 A 1

D � E 10 B 2

A � C 3 A 1

A � B 2 ������ ������������������� �"!#� �$���&%('��)�������*�,+ ��-�.

³ ��� b+¼nX�Æ3V�Æ�É�Ë�ÕRÑ�ØMÌqÍYËqË Ó�Ø�Í�Ë�Ò = ÌJÓ��Wbc 4,0�B

Sd ;<7/4�CN4GB[4P;>g�4�C  �7<=9KRI/0L?[43b � F/0L7M}606B[2���0~23.N.uHn;<4�4�CJI<.J0)4�=nI<4,0�BD4P;>g

Sd ;<7>BD23CJ7<CN79�

50�.103KR037>BD4 � BGF/0~4�=9KM4�;>guBGF/0�0�.103KR037>BD4�;>guBGF/034,0)4�=nI<4,0�BD4�2�?D0 87 �

92 � 98 � 99 � 104 � 2L798 110 � ?D0L4�Hn0 d BDCQ�<0�.Q:ub@¥@0�BD0L?DKMCN790�BGF/0+4�C  67<=9KRI/0L?[4@CN7 Sb

= ÆL\�Ç/ÌJZ{Æ�Ï�Ñ�Ø�Uä\N\{Ë�Ï ��É ��Ó�X[ÓL]�ÐLXDÓ>Õ9Ë S�`�É�ÌDÇ<Õ9Ë�ÏuÌ1]1�WÓ0>X[Z{Õ�Ð<Ë = Ë�×EÆ�ÏYÕ>ÓLXEØ = ×DÍYÆ/ÆL\N]�>Æ�ÏYÕ9Æ�Ï<]��N¾)b

�L0�BS = {a � b � c � d � e � f} b Ü�?D;<KÂBGF/0OCJ7<gq;<?DKO2�BDC1;<7ÝH/?D;L�>C18/0�8 � }60 d 2�7d ?D0L2�BE0�BGF/0�gq;>.N.J;�}+CJ79�P450�B�;>gY0��>=923BDCJ;<7/4 �

a + b + c + d + e = 87 � � � �a + b + c + d + f = 92 � � � �a + b + c + e + f = 98 � � � �a + b + d + e + f = 99 � � £ �a + c + d + e + f = 104 � � ¢ �b + c + d + e + f = 110

b � � �©<:�4G=nI>BE?[2 d BDCJ79��0L2 d F�0��>=923BDCJ;<7~g1?D;<K BGF/0P.J2L4GB � }60 d 2�7�?D0�.J2�BE0+0L2 d F��>2�?[CJ2�I<.J0PBD; a�

f = a + 23 � e = a + 18 � d = a + 12 � c = a + 11 � b = a + 6b

�P;�} }60 d 2L7�?D0�}�?[CABD0O;<790O;>g_BGF/0O0L�<=92�BDC1;<7/4�� � � � � � � BE;�8/0�BD0L?DKMCN790 a � 2�798�BGF/0L7;/I>B[2LCN7~BGF/0�?D034GB�;>g9BGF/0�7<=9KOI903?[4Lb�Ü>;<?!0� L2LKOH<.10 � }60 d 2L7M?D0�}�?[CABD060L�<=92�BDC1;<7O� � � 2L4(a + 6) + (a + 11) + (a + 12) + (a + 18) + (a + 23) = 110 �}6F>C d F d 2L7OI90+4�CNKOH<.NCA��0L8)BE; �

5a + 70 = 110 �g1?D;<K }6F>C d Ff}60��30�Ba = 8

b��YF/037b = 14 � c = 19 � d = 20 � e = 26 � 2L798

f = 31b_��F>=94 � S = {8 � 14 � 19 � 20 � 26 � 31} b23� ������� �$+���-&587 9;�<5����=� >?� � � ����@�� A )!<� ����BC����� A)D)E��393!���� �� �,� +*A � � '<-������4��� !<-������ A3>;!����<'<587

�<��!#����#��������-8'���7�#�����<� � A�>;!����<'<587 A?>;E��#'���- �!���� '<+ � ��� � B6@<��� A �F�#�,+ ��� ���,-8'#-4E�2 � E � A?>;!#��������23� ������A2 ���#����� � �<'?.³ ��� bR¼nXGÆ3V�Æ�É�½ËsVYÓ�XW\°É ½� Ê�Ç3ZQV�Ë6Õ9ËqË Ó�Ø�Í�Ë�Ò�b

�9?D;>=>�<0L?_BD;>=94!.1034@0L7>BDCJ0L?[4�H9;<4�CQBDCNg14xH9;>=9?W.1034,�>=n03.N4

x(x + 60)034GB_=97 d 2L?E?�@0H92L?Dg123CAB�b

����£

oGb = ÆL\[Ç<ÌJZ�Æ�Ï)VYÓ�XÅ�Æ9Ñ/Ë�X,Ì8Æ9ZA\NZJÏ/É#0�ZJ] �sÇ/ÌqXGË�ÒfÆ�ÏYÌ1] ��Âub-<;<4,;<7/4

x = n − 30 � 2L.J;<?[4 d F/0L? d F/03? x > 0BE0�.ä�>=n0

x(x + 60)4,;>CQB�=97d 2L?E?�@0�H/2�?[g12LCQBW?D0,�<CJ0L7>B �2 d F/03? d F/0L?

n > 30BE0�.9�<=n0

(n − 30)(n +30) = n2 − 9004,;>CQB_=97 d 2�?D?�@0�H92L?Dg123CAB�bn�P;<KMKR;<7/4k2d 0 d 2�?D?�@0�H92L?Dg123CABä�<=n0�.{� ;<7~?D0 d F/03? d F/09b c CJ7/4GC �

n2 − 900 = k2;>=RI>C1037

n2 − k2 = (n − k)(n + k) = 900 = 223252b�P;>=94�890,�<;<7/4PKO23CJ7>BD0L7/2L7>B�4 @03H92L?D0L?

900037�890�=� Mg12 d BE0�=9?[4

a0�B

bBE0�.J4��>=n0

n − k = a0�B

n + k = bb �@7'23=9?[2�2LCN7/4�C@�>=n0

n =a + b

2

0�Bk =

b − a

2

b-/=/CJ45�<=n0�79;>=94ä�<;>=/.1;<7/4s�<=n0n0�B

k4,;>CJ0L7>B!89034�037>BDC103?[4 � CA.9g12L=<B��<=n0 a

0�Bb4,;>CJ0L7>B890PK È0LKO0PH/2�?[CAB�@0/bn-/=/CJ45�<=n0�=97�8/0L4!890�=� �g12 d BE0�=9?[4�890

9008/;>CAB È0�BE?D0�H923CJ? � .1034�8/03=� .10P450L?D;<7>B�bn�L0PH/?D;/I<. �0LKO0P?D0,�<CJ0L7>BW89;<7 d �2�BE?D;>=>�<0L?W890�=� �g12 d BD03=9?[4

r0�B

s8/0

225�>=/C89;<7/7903?D;<7>B

n = r + s > 300�B

k = r − s > 0b �@?�.1034!H/2LCN?D0L4!8<C�� @03?D0L7>BD0L4�8/0�g12 d �BE0�=9?[4s890

22545;<7>B

(1, 225) � (3, 75) � (5, 45) � (9, 25)0�B

(15, 15)bnß>.A.1034@89;<7/79037>B

n ∈ {226 � 78 � 50 � 34} ;>=RI>C1037 x ∈ {196 � 48 � 20 � 4} bo�oGb = ÆL\[Ç<ÌJZ�Æ�Ï·Ñ�Ø �äÇLÉ�Ì1Ó>ÊLÆ ÄuX[ZJÒ10/Ë�XD] �uÏ<Z{ÊLË�X�É�Z{Õ>Ó<Õ Â�U �� ��] Æ_Ç>Ë�Ï�Æ�É UäZ1XGË�É�]U�X[Ð>Ë�ÏYÌJZ1Ï/Ó9b

�L0�BkI/0@2sHn;<4GCABDC �<0�CJ7>BD0��303?�bu��F/0�0L�<=92�BDC1;<7

x2+60x−k2 = 0F<2L4�Hn;<4GCABDC �<0CJ7>BD0��303?s4,;>.A=<BDC1;<7/4sCNg�2L798f;<7<. :OCNg √

302 + k2CN4�2L7RCJ7>BD0��303?�b��YF/037O}60)K�=94GB@�97982L.A.P->:�BGF<2��3;<?D032�7TBE?[CJH<.1034�;>gäBGF/0�gq;<?EK

(30, k,√

302 + k2)b'��F>CJ46CJ4~0L�<=/C �>2L.J0L7>BBE;R�9798<CN79�)2L.A.�H9?[CJK�CABDC �<0f->:�BGF<2��3;<?D032�7RBE?[CJH<.1034

(a, b, c)}6F/0L?D0

a | 30;<?

b | 30b�YF/034,0�BE?[CJH<.1034!2L?D0

(3, 4, 5) � (5, 12, 13) � (8, 15, 17) � 2�798 (15, 112, 113)b!��037 d 0 �

x = 20 � x = 48 � x = 4 � 2L798 x = 196 � ?D0L4�Hn0 d BDCQ�<0�.Q:ub23� ��� ��� �,+ ��- 587 93! ��� �� �,� +*A � � '<-�� � � ��� !<-������ A1>;!����<'<587 �<��!#��� �<��������-8' ��7 �<� ���<� � A

>;!����<'<587 A;>;EF.³ � � b+¼nX�Æ3V�Æ�É�Ë�ÕRÑ�ØMÌqÍYËqË Ó�Ø�Í�Ë�Ò = ÌJÓ��Wb

¥@0�BD0L?DKMCN790+BGF/0+.J2L?D�30L4EB�H9;<4�CQBDCQ�<0�CJ7>BD0��303?ngq;<?_}6F>C d F

2002n CN4s2�g12 d BD;<?�;>g2002!

b � F<2�B�F<2LH9H90L7/4@CAg2002

CJ4@?D03H/.N2 d 0�8OIL:2003

;<?2004

�= ÆL\�Ç/ÌJZ{Æ�ÏfÑ�Ø�Å!ÇLÉGZ�Ä�ÆL\{Ë3Ê�]�ÐLXDÓ>Õ9Ë)S�`�É3ÌDÇ>Õ9Ë�ÏYÌ1]�Æ_Ç�XEÏ/Ó/Ñ�Ø = ÆnÇ/ÌqÍ = Ë�×EÆ�ÏYÕ>ÓLXEØ = ×DÍYÆ/ÆL\N]Æ_Ç�XEÏ/Ó/Ñ�ØL]�Æ�Âub

�L0�BF = 2002!

bÝ�3CN7 d 02002 = 2 · 7 · 11 · 13 � CN7 2002!

BGF/03?D0M}+CA.N.äI90b2002/13c = 154

KM=/.QBDCJH<.1034);>g132�798 b2002/132c = 11

KM=/.QBDCJH<.1034);>g1322�798 b2002/133c = 0

KM=/.QBDCJH<.1034�;>g133

b��YF/03?D06}+CA.N.�I90~KR;<?D0~K�=/.ABDCNH/.J0L4P;>g11 �

7 � 2L798 2b��YF/03?D03gq;<?D0 � BGF/0~.N2�?D�3034GBsCJ7>BD0��303? n

4G= d FRBGF<2�B2002n 8>CQ�>C18/0L4 2002!

CN4n = 154 + 11 = 165

b_��F>=94 � n = 165gq;<?

2002b�3CJ7 d 0

2003CJ4�2+H/?DCNKR0�7<=9KRI/0L? � CJ7 2003!

BGF/0L?D0�}+CN.A.nI/0+;<7<.Q:�;<790�KM=/.QBDCJH<.10;>g2003

b_�YF>=94 � ;<7<. : 20031}+CA.N.�8>CQ�>C18/0 �L����� �{b_��F>=94 � n = 1

gq;<?2003

b�3CJ7 d 02004 = 22 ·3 ·167 � CJ7 2004!

BGF/03?D0�2�?D0 b2004/167c = 12K�=/.ABDCNH/.J0L4;>g

167 � 2�798)4�CN7 d 0 1672 > 2004 � BGF/0L?D0P2�?D0 0KM=/.QBDCJH<.1034�;>g

1672b���F/0L?D0�gq;<?D0 � BGF/0.J2L?D�30L4EBäCJ7>BD0��303?

n4�= d F)BGF<2�B

2004n 8<C �<CJ89034 2004!CJ4

n = 12b

23� ������� �$+���- 587 9;�<5����=� >?� � � ����@�� A4 )!<� ����BC����� A D(E��(' ��-&23� �,��� �� � ' � ' A���� '#-�� ���&��� !<-������ A �' @#� �$- �<� �#��������-8'���7�<� ���<� � A��<����-����<A� ��:.

����¢³ � Û bz¼nXGÆ3V�Æ�É�ËLÕ�Ñ�Ø}Å_Æ9Ñ/Ë�X,Ì�ËâÆ�X�ËLÈPÆ/Æ/Õ�]NÆ_ÇLXEÏ9Ó<Ñ�Ø = ÆnÇ<ÌqÍ = Ë�×EÆ�ÏYÕ>ÓLXEØ = ×DÍYÆ/ÆL\N]Æ_Ç�XEÏ/Ó/Ñ�ØL]�Æ�Âub

�L0�BkI90f2�gq;>=9?E�58>C1��CQB+CJ7>BD0��303?�b ¥@0�BE03?EK�CJ790�2L.A.!Hn;<4�4�CJI<.J0M�>2L.A=n0L4~;>g

kgq;<?}6F>C d F

k20030L798<4�CJ7OBGF/0)gq;>=9?�8<CJ��CAB[4

2003b � F<2�B�F<2LH9H90L7/4�CNg

2003CJ4�?D0LH<.J2 d 0L8IL:

2002;<?

2004�

= ÆL\�Ç/ÌJZ{Æ�Ï�Ñ�Ø �_Ë�ÏYË3Ê¡ZËLÊLË �LÓ3\{Æ�ÏYÕ9Ë¡]�Ë'ÓLÉ�É�Ë�Ø3]��N¾)b�L;/;/��CN79��2�B!BGF/0)H/?D;/I<.J0LKÙKR;/8>=/.1;

10 � }6064,0L0+BGF<23B k2003 ≡ 3 (mod 10)b�YF>=94 � k ≡ 7 (mod 10)

b� 0f790� 3B679;�BDC d 0RBGF<2�B72 ≡ 49 (mod 100) � 73 ≡ 43 (mod 100) � 2L798

74 ≡ 1 (mod 100)b)�L0�B

k ≡ 10a + 7 (mod 100) � }6F/03?D0 aCJ4�2~4�CN79��.10)79;<79�790���2�BDCQ�<068>C1��CQB�b��YF/037 � =94GCJ79��BGF/0�©9CJ79;<K�CJ23.9�YF/0L;<?D0LK � }606�30�B

k2003 ≡ (10a + 7)2003 ≡ 72003 + 2003 · 10a · 72002

≡ 43 + 3 · 10a · 49 ≡ 43 + 70a (mod 100)b

�YF/037 � 4GCJ7 d 0 k200303798/4@CN7~BGF/068>C1��CQBD4

2003 � }60�F<2��<043 + 70a ≡ 3 (mod 100) �

70a ≡ 60 (mod 100) �7a ≡ 6 (mod 10) �a ≡ 8 (mod 10)

b��037 d 0 � k ≡ 87 (mod 100)

b � CQBGF62�.ACAB[BD.J0PF/0�.JH6g1?D;<K#��2�H<.10 � }60��9798�BGF<2�Bk ≡ 587 (mod 1000)

2�798k ≡ 587 (mod 10000)

b��F>=94 � BGF/0f;<7<.Q: � gq;>=9?E�8<CJ��CAB ��7<=9KOI903?!CN4k = 0587

b� F/0L72003

CN4M?D0LH<.J2 d 0L8�}+CQBGF2002

;<?2004 � BGF/03?D0�CN4M79; 7<=9KOI903?)BGF<2�B452�BDCJ4G��034!BGF/0 d ;<79��?[=n0L7 d 06KO;/8<=/.J;

10b_��F>=94 � BGF/03?D0�2�?D0+79;�4,;>.A=<BDC1;<7/43b³ �+Þ b+¼nX�Æ3V�Æ�É�Ë�ÕRÑ�ØMÌqÍYËqË Ó�Ø�Í�Ë�Ò = ÌJÓ��Wb

c3 × 3

KM2���C d 45�<=92L?D0 d ;<7/4�CN4GB[4s;>g�7<CJ790�8<CN4GBDCN7 d B_�>23.N=n034 � 4�= d F6BGF<23B�032 d F�;>gBGF/0+?D;�}�4 � d ;>.N=9KM7/4 � 2�798�8<CN2��3;<7/23.J4@F<2��<0+2 d ;<7/4GB[2�7>BW4�=9K�b@©�0�.1;�} CN4�2L7M0� L2LKOH<.10;>gY23 × 3

KM2���C d 4,�>=92�?D0/b�3=9H9H9;<4,0~BGF<23B�2

3 × 3KM2���C d 45�<=92L?D0RF<2L4+2 d ;<7/4GB[2�7>B�4�=9KÂ;>g

Tbf�L0�B�BGF/0KMCJ898>.10�0L7>BE?{:O;>gnBGF>CN4@4,�>=92�?D06I/0

Eb@-9?D;3�<0�BGF<2�B

T = 3Eb

2 9 4

7 5 3

6 1 8

�����

= ÆL\�Ç/ÌJZ{Æ�ÏfÑ�Ø�Å!ÇLÉGZ�Ä�ÆL\{Ë3Ê�]�ÐLXDÓ>Õ9Ë)S�`�É3ÌDÇ>Õ9Ë�ÏYÌ1]�Æ_Ç�XEÏ/Ó/Ñ�Ø = ÆnÇ/ÌqÍ = Ë�×EÆ�ÏYÕ>ÓLXEØ = ×DÍYÆ/ÆL\N]Æ_Ç�XEÏ/Ó/Ñ�ØL]�Æ�Âub

�L0�BWBGF/0+KO2���C d 45�<=92L?D06I/0a1 a2 a3

a4 a5 a6

a7 a8 a9�YF/037a5 = E

b �3CJ7 d 0fBGF/0�4G=9K ;>g�BGF/0�03.J0LKO0L7>B[4OF/;<?[CA��;<7>B[2L.A.Q: � �<03?[BDC d 23.N. : � ;<?8<CN2��3;<7/23.N. :�CN4s0��>=92L./BE;T � BGF/037

3T = (a1 + a5 + a9) + (a2 + a5 + a8) + (a3 + a5 + a7)

= (a1 + a2 + a3) + 3a5 + (a9 + a8 + a7)

= T + 3E + T = 3E + 2T �2�798OF/037 d 0 � T = 3E

b23� ���6��� �$+���-C587&9;�<5����=�)>?� � � ����@�� A) )!<� ����BC����� A(D(E����! ��� '#+�� ��� � B6@<��� A �F�#�,+ ��� ���,-8'#- E�2 � E � A

>;!<��������23� ������A<2 ���#����� � �<'?.³ �+à b�¼nXGÆ3V�Æ�É�ËLÕ Ñ�Ød¿�Æ�É�½Ë �/Ç�ZJÉ���½� Ó�����Æ�Ó�XEXGË�XGÆ�] ��Ï>Z�ÊLË�X[ÉGZ�ÌJÓ/Ì}¼nÆL\AZ�Ì;ËL×DÏ<Z{×[Ó Õ9ËÂ/Ó/ÌJÓ3\�Ç�Ï9Ø3ÓL]8Æ�ÓLX�×EË¡\{Æ�Ï9Ó3] = VYÓ�Z1Ïub

ã{7TBE?[CJ2L79��.10ABC � 4�=9H/Hn;<450OBGF<2�B tan A � tan B � tan C

2�?D0OCJ7�F<2L?EKO;<7<C dH9?D;/��?D03454GC1;<7�bW��F/;�}�BGF<23Ba2 � b2 � c2

gq;<?DK#2L7O2L?DCQBGF<KR0�BDC d H9?D;/��?D03454GC1;<7�b%¾�Æ9Ì{Ë�>

x � y � z 2L?D0�CN7+F<2L?EKO;<7<C d H/?D;/��?D0L4�4�CJ;<7PCAg 1

x� 1

y� 1z

gq;<?EK�2L7�2�?[CABGF<KO0�BDC dH9?D;/��?D03454GC1;<7�b �= ÆL\�Ç/ÌJZ{Æ�Ï�Ñ�Ø �äÇ3É3ÌJÓ<ÊLÆ�ÄuX[Z1Ò10<Ë�XD] ��Ï>Z�ÊLË�X[ÉGZ�Õ>Ó>Õ}Â�U �E ��]3Æ_Ç<Ë�ÏYÆ�ÉWUäZJX�Ë¡É5]<U�X�Ð<Ë�ÏuÌJZJÏ9Ó/b

�L0�BABC

I/0O2�7�2 d =<BE0L��2L79��.10L8fBE?DCN2�79��.J09bT�L0�BAHa � BHb � 2�798 CHc

I90BGF/0�2L.QBDCABD=n8/0L4 d ;<?E?D0345H9;<798<CN79�PBD;�4�CJ89034a � b � 2L798 c � ?D0L4�Hn0 d BDCQ�<0�.Q:ub�3CJ7 d 0

tan A � tan B � tan C2�?D0�CN7jF<2L?EKO;<7<C d H/?D;/��?D0L4�4�CJ;<7 � BGF/03?D0�0G �CN4GB[4

d ∈ / 4�= d F)BGF<23B �1

tan A=

1

tan B− d � � � �

1

tan C=

1

tan B+ d

b � � �ã{7O2�898>CABDCJ;<7 � }60�F<2��<0�BGF/0�gq;>.N.J;�}+CJ79�P?D03.N23BDCJ;<7/4 �

1

tan A=

AHb

BHb

=AHc

CHc

� � � �1

tan B=

BHc

CHc

=BHa

AHa

� � £ �1

tan C=

CHa

AHa

=CHb

BHb

b � ¢ �

��� �

Ü>?D;<K#� � � 2�798O� £ �E� � £ � 2L798O� ¢ �G� � � � 2�798M� ¢ �G� ?D0L4�Hn0 d BDCQ�<0�.Q: � }60��30�B �1

tan A+

1

tan B=

AHc + BHc

CHc

=c

CHc

� � � �1

tan B+

1

tan C=

BHa + CHa

AHa

=a

AHa

� ��� �1

tan A+

1

tan C=

AHb + CHb

BHb

=b

BHb

b � � �Ü>?D;<K#� � � 2�798O� � �E� � � � 2L798O��� �G� 2�798M� � �G� � � � 2�798O� � �E� ?D0345H90 d BDC �<03. : � }606;/I�BD23CJ7

2

tan B=

c

CHc

+ d � � �2

tan B=

a

AHa

− d � � �L� �2

tan B=

b

BHb

b � ��� ��L0�B

k =2

tan B

b!Ü�?D;<KÙ0L�<=92�BDC1;<7/4s� �G� � �L� � 2L798O� ��� �E� CQBägq;>.A.1;�}�4äBGF<23B �c2 = (k − d)2 · (CHc)

2 � � ��� �a2 = (k + d)2 · (AHa)2 � � ��� �b2 = k2 · (BHb)

2 b � ��£ ��L0�B∆I90PBGF/0�2L?D0L26;>g 4ABC

bW�3CN7 d 0∆ =

c · CHc

2=

a · AHa

2=

b · BHb

2�

}606F<2��<0 �(CHc)

2 =4∆2

c2� � ��¢ �

(AHa)2 =4∆2

a2� � ��� �

(BHb)2 =

4∆2

b2

b � � � � ;<KOI<CN7<CJ79�+� ��� � }+CABGF�� ��¢ �E� � ��� � }+CABGF�� ��� �E� 2L798O� ��£ � }+CQBGFM� � � �E� }60�;/I>B[2LCN7 �

c2 = 2∆(k − d) = 2k∆ − 2d∆ �b2 = 2∆k �a2 = 2∆(k + d) = 2k∆ + 2d∆

b�YF/034,0+0L�<=92�BDC1;<7/4�45F/;�} BGF<2�B

c2 � b2 � 2L798 a22L?D0PCJ7~2�?[CABGF<KO0�BDC d H/?D;/��?D0L4�4�CJ;<7}+CABGF d ;<KOKO;<7O8<C���03?D0L7 d 0

2d∆b

��� �_a`+bdcfe"\2^ b�� �§Z]\�V b�� �§Z

� ��"��Ý��"~(��������� @����� "�*��������Y*����Â&���� ��� ���"!,&)&��03?D0�CN4W2�H9?D;/I>.103K }6F/0L?D0sCABuCJ4WH9;<454GC1I>.10�BE;�890�BE03?EK�CJ790sBGF/0�2�7/4D}60L?�}+CQBGF/;>=<B8<CN4 d ;L�<03?DCN79�+45;<KR0�;>g9BGF/0�?D0L23.N. :~CJ7>BD0L?D034GBDCN79��BGF>CJ79��4�BGF<2�B!2�?D0�F<2�H/Hn037<CJ79�<b

#�#�x4)�y����#� �����>� ¦ T ��037<C1;<?s��23BGF/03KO2�BDC d 4 F<2L.A.10379�30 �c 4,�>=92�?D0XABD

;>g/4�CJ890�.J0L79�¡BGF1CJ4ä8/?[2�}�76CJ7/4GC18/0�2 d CN? d .10�}+CABGF68<CN2�KO0�BD0L?

XY;>g<.10379�¡BGF

2b���F/0PH9;>CJ7>B

A.AC1034!;<7�BGF/0 d CN? d =9KMgq03?D0L7 d 0�;>g<BGF/0 d CN? d .10/b c 79;�BGF/0L?4,�>=92�?D0

Y CBECN4�8/?[2�}�7�b � F<23B!CJ4@BGF/06?[23BDCJ;O;>gYBGF/062�?D032�;>gu4,�>=92�?D0

XABDBD;BGF/0�2�?D032);>g�4,�>=92�?D0

Y CBE�

� CABGFOBGF>CJ4�B{:/H90O;>gWH/?D;/I<.J0LK �12O�30L;<KR0�BE?DC d 23._;<790)}+CQBGF�.J0L79�¡BGF<4+��CQ�<037 �G� K~:�n?[4EB�CJ7/4EBDCJ7 d B�45;<KR0�BDCJKO0L4�CJ4�BE;�=94,0 d ;/;<?D8<CN7/23BD0L4 � I/0 d 2L=9450P�<=/CQBE0�;>gNBE037�BGF/0�2�7/4D}60L?d 2L7�I90MF>CAB�=9H9;<7��<=/C d ��. :fCN7�BGF>CJ4�}�2�:�b6��F>CJ4PCJ4P79;�B�H92L?[BDC d =/.J2L?D. :�03.J0���2L7>B � I<=<B�CQB}6;<?G��4 �.Lx�y !����Nx�$%$-/=<B_BGF/0 d CJ? d .J0PCJ7)BGF/0 d ;/;<?D8>CJ7/2�BE0�H/.N2�790�}+CQBGF)CAB[4 d 0L7>BE?D0 � O � 2�B (0, 0) � X 2�B(−1, 0) � 2L798 Y

23B(1, 0)

bãqg�}60 ;{;>CJ7OBE;

A�Q}6F>C d F�CN4M;<7�BGF/0d CJ? d .J0 �E� BGF/037 OA = OX = 1 � 4�CN7 d 0MBGF/0,:2�?D0!?[2�8>CNC � 2L798 AX = 1 � I/0 d 2L=9450äBGF/0!4,�>=92�?D0F<2�4~4GC18/0�.J0L79�¡BGF

1b'�YF/03?D03gq;<?D0 � 4XAO

CN40��>=/CN.N23BD0L?[23.�BGF>=94 � ∠OXA = 60◦ b� CABGFáBGF>CN4�H/CJ0 d 0 ;>g�CN7<gq;<?EKM23BDCJ;<7e2�798;>=9?���79;�}+.10L89�30O;>g

30�60�90BE?DCN2�79��.J0L4 � }60d 2L7á890�BE03?EK�CJ790�BGF<23B�BGF/0 d ;/;<?D8<CN7/23BD0L4M;>g

A2�?D0 (−1

2,

√3

2

) bäã{7~;�BGF/03?Y}6;<?D8/4 � BD;)�30�B�g1?D;<KXBE;

A}60��3; 1

2

=97<CAB@BD;OBGF/0~?[C1�3F�B�2L798 √3

2=97<CAB[4@=9H�b

..

..

.

..

..

.

..

..

.

..

..

.

..

..

..

..

..

.

..

..

..

..

..

.

..

..

..

.

..

.

..

..

..

.

..

.

..

.

..

..

..

..

..

..

..

..

..

..

..

..

..

..

..

..

..

..

..

..

..

..

..

..

..

..

..

..

..

..

..

..

..

..

..

..

..

......................................................................................................................................

.......................................................................................................................................................................................................................................................................................................................................................................................................................................................................................................................................................................................................

.........................................................................................................................................................................................................................................................................

..

..

..

..

..

..

..

..

..

..

..

..

..

..

..

..

..

..

..

..

..

..

..

..

..

..

..

..

..

..

..

..

..

..

..

..

..

..

..

..

..

..

..

..

..

..

..

..

..

..

..

..

..

..

..

..

..

..

..

..

..

..

..

..

..

..

..

..

..

..

..

..

..

..

..

..

..

..

..

..

..

..

..

..

..

..

..

..

....................................................................................................................................................................................................................................................................................................................................................................................................................................................................................................................................................

..................................................................................................................................................................................................................................................................................................................................................................................................................................................................................................................................................

.....................................................................................................................................................................................................................................................................................

..

..

..

..

.

..

..

..

..

..

..

..

..

..

.

..

..

..

..

..

..

..

..

..

.

..

..

..

..

..

..

..

..

..

..

.

..

..

..

..

..

..

..

..

..

.

..

..

A

B C

D

EO

X Y

�3CJ7 d 0XA

2L798AB

2�?D0�Hn03?EH90L798>C d =/.N2�? � BGF/0376BD;)�3;�g1?D;<K ABD;

B � }60�KM=94EB�3; 1

2

=97<CQBP8/;�}�7f2�798 √3

2

=97<CQBsBD;OBGF/0�?[C1�3F�B�b6� � 0 d 2L7 ;1=94EBDCNgA:OBGF>CJ4�=94�CN79�)4�CNKMCA.J2L?BE?DCN2�79��.J0L4�;<?��<0 d BE;<?[4�;<?�H90L?DHn03798<C d =/.J2L?�4G.1;<H90L43b � �YF/03?D03gq;<?D0 � BGF/0 d ;/;<?D8<CN7/23BD0L4�;>gB2�?D0 (−1+

√3

2,

√3−1

2

) b���;�} � ��79;�}+CJ79��BGF/0 d ;/;<?D8>CJ7/2�BE034�;>g B2L798

Y � }60 d 2�7890�BE03?EK�CJ790�BGF/0�.10379�¡BGFO;>gnBGF/0�8<CN2��3;<7/23.BY

;>g�4,�>=92�?D0Y CBE

BY =

(

−1 +√

3

2− 1

)2

+

(√3 − 1

2− 0

)2

=

(√3 − 3

2

)2

+

(√3 − 1

2

)2

=

12 − 6√

3

4+

4 − 2√

3

4

=

4 − 2√

3 =√

3 − 1

���"

©9=<BBY =

√2 Y C � 2L798)BGF/0�2�?D032�;>g�45�<=92L?D0 Y CBE

CJ4Y C2

b!�+0L7 d 0 � BGF/0+2L?D0L2;>gnBGF/0�45�<=92L?D0+CJ4(√

3 − 1√2

)2

=4 − 2

√3

2= 2 −

√3b

�YF>=94 � BGF/0+2�7/4D}60L?_BE;6BGF/06�<=n034GBDCJ;<7�CN4 1 : (2 −√

3)b

�YF<2�B,� 4�2��3;/;/8)4GB[2�?�B���}60<� �<0�23B_.J0L2L4GBä�3;�BW2L7�2L7/4E}603?�BE;�BGF/0�H/?D;/I<.J0LK�b@©9=<B �CJ4!BGF/03?D0�26KO;<?D060�.10L��2�7>B_}�2�: ��Ã�034 �.Lx�y !����Nx�$���3CJ7 d 0

XYCN4M2�8>CJ2LKR0�BE03? � CQB~4�=nI�BE03798/4O2T?DCJ�3F�B~2�79��.J0T2L7�:�}6F/03?D0Ý;<7 BGF/0d CJ? d .J09bW�3CN7 d 0

∠XAB = 90◦ BGF/037MCAg9}60�0� 3BE03798 ABBE;�KO0�0�BWBGF/0 d CN? d .10 � CQB!KM=94EBKR0L0�B@BGF/0 d CN? d .10~23B@BGF/0M;<H9H9;<4�CQBE0�0L798�;>g�BGF/0O8>CJ2LKR0�BE03?@BGF<?D;>=n�3F

X � 7/2�KO03. : Ybã{7R;�BGF/03?_}6;<?D8<4 � ABY

CN4@264EBE?[2LCJ�3F�Bä.NCN790���L0�BrI90)BGF/0~?[2�8>CN=94�;>g�BGF/0 d CJ? d .J0��A}60��79;�}ÙF/03?D0�BGF<23B

r = 1 �E� .10�B pI/0�BGF/0O4GC18/0.10379�¡BGF ;>gs45�<=92L?D0

XABD�A}60���79;�} F/0L?D0BGF<23B

p = 1 �E� 2L798�.10�B qI/0sBGF/0�4GC18/0�.J0L79�¡BGF);>g4,�>=92�?D0

Y CBEbR�3CN7 d 0

ABYCN4+2R4EBE?[2LCJ�3F�B.NCN790+}+CQBGF

AB = p2L798

BY =√

2q � BGF/037 �=94�CN79�_BGF/0P->:�BGF<2��3;<?D032�7���F/0�;<?D03K CN7�BE?[CJ2L79��.10XAY � }60��30�B

p2 + (p +√

2q)2 = (2r)2 �p2 + p2 + 2

√2pq + 2q2 = 4r2 �

p2 +√

2pq + q2 = 2r2 b

.

.

..

..

..

.

..

..

.

..

..

.

..

..

..

..

..

..

..

.

..

..

..

..

..

.

..

..

.

..

..

..

.

..

.

..

.

..

..

..

.

..

..

..

..

..

..

..

..

..

..

..

..

..

..

..

..

..

..

..

..

..

..

..

..

..

..

..

..

..

..

..

..

..

..

..

..

.........................................................................................................................

....................................................................................................................................................................................................................................

....

...

...

....

...

...

...

...

..

..

..

...

..

..

..

...

..

..

.

..

..

.

..

..

..

.

..

..

..

.

..

.

..

..

.

..

..

..

.

..

..

.

..

..

..

..

..

..

...

..

..

..

..

...

..

...

....

...

...

...

....

...

.............................................................................................................................................................................................................................................................................................................................................................................................................................................................................................................

..

..

..

..

..

..

..

..

..

..

..

..

..

..

..

..

..

..

..

..

..

..

..

..

..

..

..

..

..

..

..

..

..

..

..

..

..

..

..

..

..

..

..

..

..

..

..

..

..

..

..

..

..

..

..

..

..

..

..

..

..

..

..

..

..

..

..

..

..

..

..

..

..

..

..

..

..

..

..

..

..

..

..

..

..

..

..

....................................................................................................................................................................................................................................................................................................................................................................................................................................................................................................................................................

...................................................................................................................................................................................................................................................................................................................................................................................................................................................................................................

......................................................................................................................................................................................................................................................................................

..

..

..

..

..

..

.

..

..

..

..

..

..

..

..

..

.

..

..

..

..

..

..

..

..

..

.

..

..

..

..

..

..

..

..

..

..

.

..

..

..

..

..

..

..

..

..

A

B C

D

E

X Y

� 0 d 2L7+4,;>. �<0!BGF>CN4�gq;<?q � =94�CN79� r = 1

2�798p = 1 � 2�798P�n798�2���23CJ7�BGF<23B�BGF/0s89034�CN?D0�8?[23BDCJ;~CJ4

1 : (2 −√

3)b

� 03.A. � BGF<23B,� 4W2PI>CAB�I90�B[BE03?�bäã1BuCJ4_KO;<?D0�03.J0���2L7>B � 2�798�KR;<?D0��30L7903?[2L.�BD;/; � 4�CN7 d 0}60@8<CJ8/7Y� B�=94,0WBGF/0!g12 d B9BGF<2�BnBGF/0�?[2�8<CA=94�2�798�BGF/0�4GC18/0!.J0L79�¡BGF�;>gXABD

2L?D0@0��>=92L.�b©9=<B_}�2LCQB��'BGF/0L?D0>� 4@KR;<?D0 � � 0>� ?D0+4GBDCA.N.nK�CJ4�4�CN79��45;<KR0�BGF>CJ79��H/?D0�B[B{:M790L2�B�b� 0���79;�}�BGF<2�BABY

CJ4@2�4EBE?[2LCJ�3F�Bä.NCN7909b@¤Y;�B[23BD0PBGF>CJ4�.NCN790�IL:45◦ d ;>=97>BD0L?E�d .1; d ��}+CJ450�2�I/;>=<B

Bb_�3CJ7 d 0

∠ABX = 45◦ � BGF/0P790�} .ACJ790�H92L45450L4_BGF<?D;>=n�3F X2L798

Bb��3CJ7 d 0

∠CBY = 45◦ � BGF/037MBGF/0)790¡}ç.NCN79062L.N4,;MH/2�4�4,034@BGF<?D;>=n�3F Cb�ã{7�;�BGF/0L?}6;<?D8/4 � XBC

CN4@264EBE?[2LCJ�3F�Bä.NCN7909b� F<23B+8/;/0L4�BGF>CJ4�KR032�7 � � 0�.N. � 4�CN7 d 0 ∠XCY = 90◦ � BGF/0OHn;>CN7>B CKM=94EB.NCJ0O;<7�BGF/0 d CJ? d .J0M2L4�}603.A. � c 798�BGF>CN4�}6;<?G��4PCJ7��303790L?[23.�� CAg�}60M8<?[2�} BGF/0M45�<=92L?D0

XABD}+CQBGF62�7�:~�J?D0L2L4,;<7/2�I<.J0 � 4�CJ890�.10379�¡BGF)2�798 A

;<76BGF/0 d CN? d =9KMgq03?D0L7 d 0+;>g/BGF/0d CJ? d .J0 � 2L798MCAgn}60+=9450 B2�798

Y2�4!BGF/0)8>CJ2��3;<7/2L.Y;>gY2)790¡}ç45�<=92L?D0

Y CBE � BGF/0L7C}+CA.N.�23.Q}�2�:/[email protected]);<7MBGF/0 d CN? d =9KMgq03?D0L7 d 09b���0L2�B � c 798MBGF<2�B,� 4�79;�B�4,;<KO0�BGF>CN79�P}60����=9?D0L8R;>=<Bäg1?D;<K#;>=9?�;<?[C1��CN7/2L.n45;>.N=<BDCJ;<7�b

��£<�_��bfe�X]W��%g _]W}`�W��}b��

�wµP­_±��qÃe±�µ+¸ ��²�T¸s¿n¸sÄ+°_¸� ���+­s¿�± à�������! #"%$'&)(+*,"

ã{7�BGF/0~.N2�4EB@B�}6; d ;>.N=9KM7/4+� �����3£4� ��� � � �������3£4���L��� �>�L�>��� � }60~2�7/23.Q:���0�84,0L�<=n037 d 034!IL:60G L2�K�CJ7<CN79��BGF/0�4,0L�<=n037 d 034ä;>g/8<C���03?D0L7 d 0L4 � 450 d ;<79868<C���03?D0L7 d 0L4 � 2L7984,;M;<7�bs�P;�} }60P}+CJ45F~BD;M2LH9H<.Q:O;>=9?äC18/0L2L4�BE;�4,0L�<=n037 d 034@BGF<2�B�2�?D0�79;�B��30L7903?[23BD0�8IL:MHn;>. :/79;<K�CJ23.J43b

;<7/4GC18/0L?nBGF/0s4,0L�<=n037 d 0 {an} }6F/0L?D0 an = 2n � � =n034GBDCJ;<7�� � g1?D;<K���2�? d F�� 4F/;<KR0¡}6;<?G� � b � 0��9798'BGF<2�B � gq;<?�BGF>CJ4MH/2�?�BDC d =/.N2�?)4,0L�<=n037 d 0 � 0L2 d F 4,0L�<=n037 d 0T;>g8<C���03?D0L7 d 0L4@CN4s0��>=92L./BE;6BGF/06;<?DCJ��CJ7/23.n4,0L�<=n037 d 0/btn 1dn 2dn

11

2 12

4 24

8 48

16� 0 d 2L7Ý0G LBD0L798fBGF/0R450��>=n0L7 d 0L46032�4GCN. :�b � 0RF<2��<01d4 = t4 = 16 �

BGF/0L7 �4�CN7 d 01d4 = t5 − t4 � }60)450�0+BGF<2�B t5 = 1d4 + t4 = 16 + 16 = 32

b��P;�} BGF<2�B}60+��79;�}�2+790¡} BD0L?DK � }60 d 2�7~?D0LH90L2�B_BGF/0�H9?D; d 0L4�4äBD;)�9798~2�4�KM2�7�:6BD0L?DKO4�2L4W}60.NCJ��09bä�YF/064�2�KO0�H9?D; d 0L4�4 d 2�7RI90+=94,0L8R;<7�BGF/0�Ü�C1I/;<7/2 dEd CY450��>=n0L7 d 0 � 1 � 1 � 2 � 3 � 5 �8 � 13 � . . .

� � =n0L4EBDC1;<7�� £ g1?D;<K´��2�? d F�� 4�F/;<KR0¡}6;<?G� � bã{7O�303790L?[23. � CAg�2�7�:�;>g9BGF/0�4,0L�<=n037 d 034s;>gY8>C��u0L?D037 d 034 d 2�7MI90+0� 3BE0379890L8 � BGF/0L7}60 d 2�7�0G LBD0L798OBGF/0M;<?[C1��CN7/2L.u4,0L�<=n037 d 0/bO©9=<B@CJ7f45;<KR0 d 2L4,034�BGF/0)B[2�I>.10�;>g_8<C���03?G�0L7 d 0L4s89;/03457Y� BW:>C10�.18�2L7�:MH/23B[BD0L?D7/4Lb � F<23B�8/;6}60�89;)BGF/037 � ;<7/4GC18/0L?uBGF/0�4,0L�<=n037 d 0 {bn} }6F/0L?D0 bn = 5n � � =n0L4EBDC1;<7�� � g1?D;<K#��2�? d F�� 4F/;<KR0¡}6;<?G� � b ;<7/4EBE?D= d BDCN79�PBGF/0PBD2�I<.J06;>gY8<C���03?D0L7 d 0L4 � }60��30�B

tn 1dn 2dn 3dn 4dn

14

5 1620 64

25 80 256100 320

125 400500

625ãqg!}60f8/;<7Y� B�4,0L0�2�7�:ÝH92�B[BE03?E7/4~F/0L?D0 � }60 d 2L7 d ;<7/4GC18/0L?PBGF/0ÝÕ�Z �_Ë�X�Ë�ÏY×EËTÌJÓ/ÑL\{Ë�Æ�ÎÕ9Ë,V_ÌqÍ2b@�9; d ;<7/4GBE?[= d B!BGF>CJ4 � }60 d ;<7/4�CJ8903?!BGF/064,0L�<=n037 d 0 t0 � 1d0 � 2d0 � 3d0 � . . . �}6F>C d FRCN7�;>=9? d 2L4,0)CN4

1 � 4 � 16 � 64 � 256 � . . .b~ãqg_79;�BGF>CN79�)2�H/Hn032�?[4P23B�BGF>CJ4�.J0,�<03. �}60 d 2L7 d ;<7/4GBE?[= d B!BGF/0)8<C���03?D0L7 d 0�B[2�I>.10);>g�8/0LH>BGF

3g1?D;<K�BGF/0)8>C��u0L?D037 d 0+BD2�I<.J0);>g8903H<BGF

2=94GCJ79�PBGF/0�4�2�KO0�KR0�BGF/;/8�bsãqgn:<;>= d ;<7>BDCN7<=n0+BGF/0�H9?D; d 0L4�4!:<;>=�}+CN.A.YI/0�.J0�8BE;)BGF/0+Hn;�}603?[4�;>g

2b

��£/�

�P;�}ç}60 d 2�7 d ;<KOI<CN790)BGF/034,0~KR0�BGF/;/8/4s}+CABGFf;>=9?@CJ7��<03?[4�CJ;<7fgq;<?DKM=/.N2�g1?D;<KBGF/0���2�:~CJ4�4�=n0�� d ;<?D?D0 d BD0�8~BD;�?D03KR;3�<0�2+B{:/H9; � �tn+k =

n∑

i=0

(

n

i

)

idkb

m��"�'���<y~���0 � 2 � 9 � 31 � 97 � 291 � . . . ;<7/4EBE?D= d BDCN79�PBGF/0�8<C���03?D0L7 d 0�BD2�I<.J0 � }60��30�B

tn 1dn 2dn 3dn 4dn 5dn

02

2 57 10

9 15 1922 29 36

31 44 5566 84

97 128194

291�YF>CN4WCJ4�7Y� B�KM= d F)F/0�.JH�b��L0�B,� 4 d ;<7/4EBE?D= d BuBGF/0P8<C���03?D0L7 d 0sBD2�I<.J0P;>gn8/0LH>BGF2b_��B[2�?�BDCJ79�}+CABGF)BGF/0+4,0L�<=n037 d 0 {sn} = {0 � 2 � 5 � 10 � 19 � 36 � . . .} � }606�30�B

sn 1d′n 2d′

n 3d′n

02

2 13 1

5 25 2

10 49 4

19 817

36� 0�450�0�BGF<2�Bidn = 2n gq;<? i ≥ 2

bW¦s4GCJ79�P;>=9?�CJ7��<03?[4�CJ;<7)gq;<?EK�=/.J2�}+CABGFk = 0 � }60�30�B

sn =

n∑

i=0

(

n

i

)

id0 = 0 + 2n +

n∑

i=2

(

n

i

)

=n∑

i=0

(

n

i

)

+ n − 1 = 2n + n − 1b

��037 d 0 � 2LH9H<.Q:>CJ79��BGF/0+CN7��<0L?[4GC1;<7�gq;<?DKM=/.N2+BD;)BGF/0�;<?DCJ��CJ7/23.�450��>=n0L7 d 0 � }60�;/I>B[2LCN7tn =

n∑

i=0

(

n

i

)

(

2i + i − 1) b

Ã�;>=�4,F/;>=/.J8R79;�}�F<2��<0~2+�>2�?[C10�B�:�;>gYBE;/;>.N4�gq;<?@2�B[BD2 d ��CN79�6450��>=n0L7 d 0L43bP¦s7>BDCN.790� 3BäBDCNKR0 � F<2LH9H�:MH/?D;/I<.J0LK#45;>.Q�>CJ79� �

��£/������ � d � � ����� �%����� ���

�ç&�����

� ������� &)&6(��/&� ��=9?��n?[4EBM4,0�BO;>g+H/?D;/I<.J0LKM4Rgq;<?~BGF>CN4RCN454G=n0 d ;<KR034Rg1?D;<K c ?D�30L7>BDCN7/2nb���:BGF<2�79��4ä�3;PBE; F<?DCN4ä�LKM2L.A. � 2�7/2�8<CN2�7+�9032�K#�L032�8/0L?�BE;PBGF/0 £/����� ãD� � � gq;<? d ;>.A.10 d BG�CJ79��BGF/0LKÙgq;<?@;>=9?�=94,0 � 2L798MBE; c .JI903?[BD;��P0�B[BE0�.u2L798T��=<�+->2L.N2 d CJ;<4sgq;<?!BE?[2L7/4�.N23BDCN79�BGF/0LK�CN7>BE;Oß<79��.ACJ45Fub

��������� � »"!~ª �#� ª �$� ª�³ � �#% !�³ � �#� � � ¶'&e¶)(â³ � �*� +� $.&�/{����$-, � ��+/. !'& /6&�/{���r:�<54��y/k��:#��"�Û b �YF/0f7/23BD=9?[23.@7<=9KOI903?[4�2�?D0O}�?[CAB[BD0L7�CN7'4G= dGd 0L4�4�CJ;<7 � gq;<?DKMCN79��2�4,0L�<=n037 d 0�;>g8<CJ��CAB[4Lb�¥@0�BD0L?DKMCN790+F/;�}�KM2�7�:~7<=9KO0L?[C d 2L. d F<2�?[2 d BE03?[4�2�?D0�CJ76BGF/0�7/2�BD=9?[2L./7<=9KRI/0L?BGF<23B d ;<7>BE?DCJI<=<BD0L4�BD;)BGF>CN4s450��>=n0L7 d 0�BGF/068>C1��CQB!23BäHn;<4GCABDCJ;<7

102000b�P;�BD0 � �YF/0P7/2�BD=9?[2L./7<=9KOI903?�BGF<23B d ;<7>BE?[C1I>=<BE034WBD;+BGF/0P4,0L�<=n037 d 0�BGF/0P8<CJ��CAB_2�BHn;<4GCABDCJ;<7

10F<2�4

27<=9KR03?DC d 23. d F<2L?[2 d BD0L?[4PI/0 d 2L=9450)CQB@CN4

10 �BGF/0)7/2�BD=9?[2L.u7<=9KRI/0L?BGF<23B d ;<7>BE?[C1I>=<BE034�BD;�BGF/0�450��>=n0L7 d 0OBGF/0f8<CJ��CAB+2�B+Hn;<4GCABDCJ;<7

102F<2L4

27<=9KR03?DC d 23.d F<2�?[2 d BE03?[4�I90 d 23=94,0�CABäCN4

55b

Þ bW¨�CQ�<037~2�BE?[CJ2L79��.10ABC

}+CQBGF)4GC18/0AB

��?D0323BD0L?�BGF<2�7BC � .10�B M

I90�BGF/0PKMCJ89�Hn;>CN7>B�;>gAC � 2�798�.J0�B L I/0�BGF/0sH9;>CJ7>Bu2�Bn}6F>C d F�BGF/0sI<CN4,0 d BE;<?u;>g ∠B

d =<B[4_4GC18/0AC

bc 4EBE?[2LCJ�3F�B�.ACJ790�CN4�8/?[2�}�7MBGF<?D;>=n�3FM

H/2�?[23.N.J03.9BE;AB � d =<B[BDCJ79��BGF/0)I<CN4,0 d BE;<? BL23B

D � 2�798f2�79;�BGF/03?�4GBE?[23C1�3F�Bs.NCN790~CJ4�8/?[2�}�7�BGF<?D;>=n�3F LH/2�?[23.N.J03.YBE;

BC � d =<B[BDCJ79�BGF/0�KR0L8<CN2�7BM

2�BEbW��F/;�}�BGF<23B

EDCN4@Hn03?EH90L798>C d =/.N2�?WBE;

BLb

à b c I9;<2L?D8�F<2�432?D;�}�4P2L798

10d ;>.N=9KM7/4LbM->2L=/.�}�?DCQBE034�CJ7�032 d F�4,�>=92�?D0�;>gWF>CJ4I9;<2L?D8f03CQBGF/0L?

1;<? −1

b6��2�B[BGF/0�} � }+CABGF�->2L=/.{� 4PI9;<2L?D8RCN7�4�CJ�3F�B � d F/;/;<450L4�;<790�;<?KR;<?D0 d ;>.A=9KO7/4�g1?D;<K F>CJ4_;�}�7�I/;<2�?D8�2�798 � CN7PBGF/0L450 d ;>.A=9KO7/4 � 790���2�BE034_23.N.�;>gn->2L=/.{� 47<=9KRI/0L?[46�JH/=<B[BDCN79�1}6F/03?D0�->2L=/.WF<2�4 −1 � 2�798 −1

}6F/03?D0�->2L=/.WF<2L41 � bfã{7fBGF/0?D0L4EB�;>gnBGF/0 d ;>.N=9KM7/4�F/0+H/=<B[4�BGF/0�4�2�KO0+7<=9KRI/0L?[4s2L4+->23=/.�� 4Lb��23B[BGF/0¡} }+CN7/4sCNg�F/0 d 2�7��30�Bs032 d FO?D;�} ;>g�F>CN4PI/;<2�?D8MBD;RI90~8>C��u0L?D037>B@g1?D;<K0L2 d F�?D;�}ç;>g_->23=/.�� 4�I/;<2�?D8ub �!BGF/0L?�}+CJ450��NBGF<2�BäCN4 � CNg�2�7�:�?D;�} ;>g_��2�B[BGF/0�}�� 4sI9;<2L?D8CJ4!BGF/0�4�2�KO0+2�4@2L7�:M?D;�}ç;>gW->23=/.�� 4�I/;<2�?D8 �G� ->23=/./}+CN7/4LbãqgäBGF/0,:ÝI9;�BGF�H/.N2�:�BD;TH90L?[gq0 d BDCJ;<7 � 8/0�BD0L?DKMCN790M}6F>C d FÝ;>gWBGF/0LK%F<2L4��<C d BE;<?{:��=92�?[2L7>BE0L0�8ub

½ bs¥@0�BD0L?DKMCN7906F/;�} KO2L7�:MH923CJ?[4s;>gY7/23BD=9?[23.�7<=9KRI/0L?[4(a, b)

BGF/03?D0�2�?D0�4G= d F~BGF<2�B4620

CJ4@2�KM=/.QBDCJH<.10�;>ga � 4620 CJ4@2�K�=/.ABDCNH/.J06;>g b � 2L798 b

CJ4@2�KM=/.QBDCJH<.10�;>gab

��£/�å b c d ;<KOH<=<BE03?�H/?D;/��?[2�K �30L7903?[23BD0L4�2M450��>=n0L7 d 0O;>g_7<=9KOI903?[4�IL:RBGF/0~gq;>.N.J;�}+CJ79�?D=/.J09bu�YF/0@�n?[4EBu7<=9KRI/0L?uCN4u}�?[CAB[BD0L7�IL: 2LKMCA.1;ubWÜ�?D;<K�BGF<23BuHn;>CN7>B�;<7 � BGF/0sH9?D;/��?[2LK89;/034uCJ7>BD0��303?u8<C �<CN4�CJ;<7�;>gLBGF/0!.J2L4GBY�30L7903?[23BD0�8�7<=9KOI903?uIL:

18 � ;/I>B[2LCN7<CJ79�!2L7PCJ7>BD0��303?�<=n;�BDCJ0L7>B�2L798�?D03KO23CJ798/0L?¡b��YF/0s4G=9K ;>g�BGF>CN4�CJ7>BD0��303?��>=n;�BDC1037>B�2L798�BGF>CN4_?D0LKM2LCN798903?I90 d ;<KO0L4�BGF/0â790G LB��30L7903?[23BD0�8�7<=9KRI/0L?¡bwÜ>;<?�0� L2LKOH<.10 � CNg 2�K�CN.J;�� 4�7<=9KRI/0L?CJ45291 � BGF/0 d ;<KMH/=<BD0L?f8/;/0L4 5291 = 293 ∗ 18 + 17 � 2L798��303790L?[2�BE034fBGF/07<=9KRI/0L?

310 = 293 + 17b!��F/06790G LB@�303790L?[2�BE0L8O7<=9KRI/0L?W}+CN.A.YI/0

21 � ��CQ�<037MBGF<2�B310 = 17 ∗ 18 + 4

2�79817 + 4 = 21 � 0�B d bÜ<;<?�0,�<03?{:fCJ7<CQBDCJ23.�7<=9KRI/0L?sBGF<2�B 2LKMCA.1; d F/;/;<4,034 � BGF/03?D0 d ;<KO0L4�2MH9;>CJ7>B�2�B}6F>C d FOBGF/0 d ;<KOH<=<BE03?��303790L?[2�BE034�BGF/0~4�2�KO0~7<=9KOI903?�;L�<03?s2�798f;L�<03?�b~¥@0�BD0L?DKMCN790}6F>C d F�7<=9KRI/0L?WBGF>CN4!}+CA.N.nI90 � CNg 2�K�CN.J;�� 4�CN7<CABDCN2L.n7<=9KOI903?äCJ4 2110

bæ � c 7�0��>=/CN.N23BD0L?[23.uBE?DCN2�79��.J06}+CABGF�2L?D0L2O0L�<=923.�BD;

9CJ4PKO2�890M;>=<B�;>gWH/2�H90L?¡bMã1B�CN4gq;>.18/0�8�CN7~B�}6;�23.1;<79��264EBE?[2LCJ�3F�Bä.NCN790�H/2�4�4�CN79��BGF<?D;>=n�3F~BGF/0 d 0L7>BE?D06;>gnBGF/0�BE?[CJ2L79��.102�798�79;�B�H/2�4�4�CN79�6BGF<?D;>=n�3F�2�7�:�;>g�BGF/06�<0L?�BDC d 0L4+;>g�BGF/0~BE?DCN2�79��.J09b)�YF/0M?D034�=/.QB�CJ4�2gq;>=9?G�G4�CJ890L8RHn;>. :<�3;<7Rgq;<?EKO0�8�IL:MBGF/0~;L�<03?D.N2�Hf;>gYBGF/0�B�}6;OH<C10 d 034P� d ?D0323BD0�8�IL:OBGF/0gq;>.18 � 2L798�BGF<?D0L0�BE?DCN2�79��.J0L4!}+CQBGFM79;M;L�<03?D.N2�H�b�¥@0�BD0L?DKMCN790+BGF/0�K�CJ7<CNKM=9K#Hn;<4�4�CJI<.J02�?D032);>gnBGF/0�gq;>=9?G�G4�CJ890L8MH9;>.Q:<�3;<7 d ?D0323BD0�8OIL:)BGF/06;L�<03?D.N2�H�b

c 4PBGF/0M450 d ;<798�H9?D;/I>.103K 4,0�B�gq;<?sBGF>CJ4�7<=9KRI/0L? � }60O��C �<0~BGF/0 ������� Ü>;<?EK ;>gBGF/0����sã c .1I<2�7<CN2�7á��23BGF/03KO2�BDC d 2L. ��. :/KMH/CN2�8Tgq;<?)�PC1�3F�� d F/;/;>.N4 � �YF>CN?D8j¤Y;>=9798ub�YF<2L79��4s2���2LCN7O�3;)BE; F<?DCN4��LKM2L.A. � 2L7/2�8>CJ2L7M�/0L2LK �L0L2�8903?WBD;)BGF/0 £/����� ãD� � � gq;<?d ;>.A.10 d BDCJ79��BGF/03KTb�"����� ¶� � ª �*� ª ³ � �#% !~³ � �#� � � ¶ &e¶)(á³ � �$� +

� & � % � » % ® � % &/&e¶�®� (?/7� ����� ���h��'#���� ���� �������

Û bW�12 � -9?D;L�<0PBGF/0+CN790��>=92L.ACAB{:(1 + x1)(1 + x2) · · · (1 + xn)

1 + x1x2 · · · xn

≤ 2n−1 � ∀x1 � x2 � . . . � xn ∈ [1, +∞)b

�qI � � F/0L7O89;/034!BGF/060L�<=923.NCQB�:MF/;>.18.�Þ b ;<7/4�CJ8903?)BGF/0T4,0L�<=n037 d 0x1 � x2 � . . . � xn � . . .

4�= d F'BGF<23Bx1 =

√22L798

xn+1 =√

2 + xngq;<?ä2L.A.

n > 1b��CJ798

�J2 � limn→∞

xn ��1I � lim

n→∞4n(2 − xn)

bà b@-9?D;L�<0PBGF<23B � CAg 0 < a < b <

π

2� BGF/037

�J2 � a

b<

sin a

sin b�

�1I � sin a

sin b<

π

2

a

b

b

��£�£½ b_�J2 � -9?D;L�<0�BGF<23Bugq;<?�2�7�: d ;<7��<0�  n

� �3;<7 � }6F/03?D0 n > 4 � BGF/0�2�?[CABGF<KO0�BDC d KR032�76;>gBGF/0+.J0L79�¡BGF<4s;>gnCAB[4@4GC18/0L4�CN4@79;���?D0L2�BE03?WBGF<2L7~BGF/0+2�?[CABGF<KO0�BDC d KO0L2L7O;>g9BGF/0+.J0L79�¡BGF<4;>g�CAB[4s8<CN2��3;<7/23.J43b�1I � ¥@;/034s0��>=92L.ACAB{:OF/;>.J8�gq;<?ä2L7�: d 2L4,0��å b��L0�Ba � b � c I/0@BGF/0�4�CJ89034ä;>g<2@BE?DCN2�79��.J0 � 2�798+.J0�B α � β � γ I/0@BGF/0�2�79��.J0L4ä;<H/Hn;<4GCABD0BGF/0�4�CJ89034

a � b � c � ?D0L4�Hn0 d BDCQ�<0�.Q:ub�J2 � -9?D;L�<0PBGF<2�B γ = 2αCAgY2�798O;<7<. :~CNg

c2 = a(a + b)b

�qI � �CN798�2L.A.�BE?DCN2�79��.J0L4�4G= d FfBGF<2�B a � b � c2�?D0O7/2�BD=9?[2L.W7<=9KOI903?[4 � b

CN4�2RH/?DCNKR0 �2�798γ = 2α

b

³ � �#% !�³ � �#� �O® � &â³ ��! �#� �#� &âªe® � ª � � ª ¶ � ª +��Þ ����� � Þ ����Û� /��,(�� �'(_�_� ,*"%$'&�(�!,+#$'& /1��� � ���,&�!,��& � � !,��/{��� /���/7�'/{��� � ���� ���� ���

� �o�,x:) ��#�$��� ���������k'�-����� $������Y� $:!��/�'6Û bMÜ>;>=9?s=97<CABP8>CJ4 d 4P2�?D0�H/2 d ��0L8�BD;/�30�BGF/03?2�4)4,F/;�}�7TCJ7TBGF/0��n��=9?D0L43b��YF/0�8>CJ4 d 4)2L?D00L7 d .J;<4,0L8�IL:�2 d =9?{�<0R;>gWKMCN7<CJKM2L.�.J0L79�¡BGFubã{7+}6F>C d F6;<790�;>g>BGF/0 d ;<7<�n��=9?[23BDCJ;<7/4WCN4�BGF/00L7 d .J;<4�CN79� d =9?{�<0�4,F/;<?�BE03? � .

..

..

.

..

.

..

..

.

..

..

..

..

..

..

..

..

..........................

.........................................................................................................................

................................................

..

..

.

..

..

.

..

.

..

..

..

..

..

..

..

..

.......................................................................

...

...

..

.

..

..

..

..

..

..

...

..................................................................................................... ..

..

.

..

..

.

..

.

..

..

..

..

..

..

..

..

.......................................................................

...

...

..

.

..

..

..

..

..

..

...

.....................................................................................................

.

..

..

.

..

.

..

..

.

..

..

..

..

..

..

..

............................

.........................................................................................................................

................................................

.......................................................................

.

.

..

.

..

.

.

..

.

..

.

..

.

..

.

.

..

.

..

.

..

.

..

.

.

..

.

..

.

..

.

..

.

.

..

.

..

.

..

.

..

.

.

..

.

..

.

..

.

.

.......................................................................

.

.

..

.

..

.

..

.

.

..

.

..

.

..

.

..

.

.

..

.

..

.

..

.

..

.

.

..

.

..

.

..

.

..

.

.

..

.

..

.

..

.

..

.

.

..

.

..

.

. ........................................................

.........................................................................................................................

................................................ .

..

..

.

..

.

..

..

.

..

..

..

..

..

..

..

..

..........................

.........................................................................................................................

................................................

.

..

.

..

..

..

.

..

.

..

..

..

..

..

..

..

.......................................................................

...

...

..

..

..

.

..

..

..

..

...

...

....................................................................................................

..

.

..

..

..

.

..

.

..

..

..

..

..

..

..

..

.....................................................................

...

...

..

..

..

.

..

..

..

..

...

...

...................................................................................................

.......................................................................

..

..

..

..

..

..

..

..

..

..

..

..

..

..

..

..

..

..

..

..

..

..

..

..

..

..

..

..

..

..

..

..

..

..

..

.

.......................................................................

.......................................................................

Þ b@¥@0�BD0L?DKMCN790�?D032L.x2L798

y4�= d F)BGF<2�B

5x2 − 5y2 − 24xy + 11y + 3x = 0b

à b@¥@0�BD0L?DKMCN790�23.N.nH9;<4�CQBDCQ�<0�CN7>BE0L�30L?[4m2�798

n4G= d F)BGF<23B

m2 − n2 = 270b

½ b c 7<=9KOI903?6;>g d ?D;<4�4,�G4,F<2LHn0L8ÝH/CJ0 d 0L4 � 2�464,F/;�}�7 �2�?D0~H/.N2 d 0�8f;<7f2�78 × 8

d F/034545I9;<2L?D8�CN7f4�= d F�26}�2�:BGF<23B@BGF/0~45�<=92L?D0L4�;>guBGF/0~H/CJ0 d 0L4P2L798RBGF/0~45�<=92L?D0L4�;>gBGF/0 d F/034545I9;<2L?D8�2L?D0)2L.AC1��790L8�2L798OBGF/0)H/CJ0 d 0L4P8/;R79;�B;L�<03?D.N2�Hâ032 d F ;�BGF/0L?¡b � 0�4�2�:ÝBGF<23B�BGF/0ÝI/;<2�?D8áF<2L4I90L0L7M�/.N.J0�8~CAgY79;MKO;<?D0�H<C10 d 034 d 2�7OI90+H/.N2 d 0�8O;<7~BGF/0I9;<2L?D8~4�23BDCN4�gA:>CJ79�@BGF/0 d ;<798<CQBDC1;<7/4!2�I9;3�<09b�¥@0�BE03?EK�CJ790BGF/0�4�KO23.N.J0L4EBuHn;<4�4�CJI<.J0s7<=9KOI903?_;>g>H/CJ0 d 0L4u}+CQBGF�}6F>C d FBGF/06I9;<2L?D8 d 2�7OI90��9.A.10L8�b..........................................................................................................................................................................................................................................................................

.

..

.

..

.

..

.

..

..

.

..

.

..

.

..

.

..

.

..

.

..

.

..

.

..

.

..

.

..

.

..

.

..

.

..

.

..

.

..

.

..

.

..

..

.

..

.

..

.

..

.

..

.

..

.

..

.

....................................................................................................................................................................................................................................................................................................................................................................

............................................................................................................................................................................................................................................................................................................................................................................................................................................................

.

..

.

..

.

..

.

..

.

..

.

..

.

..

.

..

.

..

.

..

.

..

.

..

.

..

.

..

.

..

.

..

.

..

..

.

..

.

..

.

..

.

..

.

..

.

..

.

..

.

..

.

..

.

..

.

..

.

..

.

..

.

..

.

..

.

..

.

..

..

.

..

.

..

.

..

.

..

.

..

.

..

.

..

.

..

.

..

.

..

.

..

.

..

.

..

.

..

.

..

.

..

.

..

..

.

..

.

..

.

..

.

..

.

..

.

..

.

..

.

..

.

..

.

..

.

..

.

..

.

..

.

..

.

..

.

..

.

..

..

.

..

.

..

.

..

.

..

.

..

.

..

.

..

.

..

.

..

.

..

.

..

.

..

.

..

.

..

.

..

.

..

.

..

.

��£/¢³ � �#% !�³ � �#� �O® � &â³ ��! �#� �#� &âªe® � ª � � ª ¶ � ª +��Þ ����� � Þ ����Û

� /��,(�� �'(_�_� ,*"%$'&�(�!,+#$'& /1��� � ���,&�!,��& � � !,��/{��� /���/7�'/{��� �� /7��$-, ���� ������'):#o! �'#o* �� ����� $ �k"� �u��� $������¡�-$:!����'6

Û b!�L0�BABC

I90@2�?[C1�3F�B�BE?[CJ2L79��.10ä}+CABGF+F�:<Hn;�BD0L7<=9450AB

2�798�2L.QBDCABD=n8/0CF � }6F/03?D0

F.NCJ0L4�;<7

ABb��YF/0 d CN? d .10)BGF<?D;>=n�3F

Fd 037>BE?D0�8f23B

B2�798f2L79;�BGF/0L? d CN? d .10�;>g�BGF/0452LKR0+?[2�8>CN=94 d 037>BE?D0�8M2�B

ACJ7>BD0L?[450 d B�;<7~BGF/0+4�CJ890

CBb@¥@0�BE03?EK�CJ790

FB : BCb

Þ b �>}6; 79;<79�GCN7>BE03?[4,0 d BDCJ79� d =9?{�<0L4�F<2��<0�0L�<=92�BDC1;<7/4y = ax2 + bx + c

2L798y = dx2 + ex + f � }6F/03?D0 ad < 0

b�-9?D;L�<0~BGF<23B�BGF/0L?D0O0� LCJ4EBD4+2O4GBE?[23C1�3F�B�.ACJ790F<2��>CJ79��79;�Hn;>CN7>BD4�CJ7 d ;<KOKO;<7)}+CABGF6BGF/0�B�}6; d =9?{�<0L43bà b��YF/0�H9;<4�CQBDCQ�<0PCJ7>BD0��303?[4

a � b � 2L798 c4�23BDCN4�gA: 1

a+

1

b+

1

c< 1

bs-9?D;3�<0�BGF<2�B1

a+

1

b+

1

c≤ 41

42

b½ bÝã{7�BGF/0�}60�0L��. :Ý��B[23BD0T�L;�B[BE03?{: � 2�450��>=n0L7 d 0�;>g�450,�<0L7T7<=9KRI/0L?[4�CJ4�H/C d ��0�8T2�B?[2�798/;<KTb+ß<2 d FR7<=9KRI/0L?@KO2�:fI/0)2�7�:�;>g

0 � 1 � 2 � 3 � 4 � 5 � 6 � 7 � 8 � 9 b6¥@0�BD0L?DKMCN790BGF/0�H9?D;/I<2�I>CN.ACAB{:�BGF<23BYBGF/0�4,0L�<=n037 d 0sCN4 d ;<KOH9;<4,0L86;>g/;<7<.Q:��<�<0�8<C���03?D0L7>B�7<=9KOI903?[4Lbå b@¥@0�BD0L?DKMCN790

n ∈ �4G= d F~BGF<2�B

n2 + 28<C �<CJ89034

2 + 2001nb

�P;�}�}60�BD=9?D7áBE;�BGF/0Ý4,;>.A=<BDC1;<7/4)}60�F<2��<0Ý?D0 d 0�CQ�<0L8 BE;áH/?D;/I<.J0LKM4�;>g�BGF/0��B�bu-<0�BD0L?[45I<=9?D� ;<7>BE034GB[4 �� �>¢��>�" ��£ ��CQ�<037�CJ7f��2�: � �L����� �����>�'������� ������� � bÛ b)��F/0L?D0�2�?D0n��.N2�4�4,034+032 d F�I>C1��0379;>=n�3FRBE;fF/;>.18f2L.A.uBGF/06}�23BD0L?¡bOã{7<CQBDCJ23.N. : � 2L.A.��.J2L45450L4 d ;<7>BD23CJ7fBGF/0O452LKR0M2�KO;>=97>B+;>g�}�23BD0L?¡b�ã1BPCN4�23.N.J;�}60�8�BD;�Hn;>=9?�g1?D;<KÂ2L7�:��.J2L454�BE;Ý2�7�:';�BGF/03?)��.N2�4�4~2L4~K�= d F�}�23BD0L?�2L4)CN7ÝBGF/0f4,0 d ;<798'��.N2�4�4LbjÜ<;<?P}6F>C d F�>2L.A=n0L4s;>g

nCJ4�CQB!H9;<454GC1I>.10PBD; d ;>.N.J0 d Bä2L.A.<}�23BD0L?WCJ7>BD;M;<790���.J2L454<�

= ÆL\�Ç/ÌJZ{Æ�Ï Ñ�Ø@¼/Z{Ë�XEXGË Æ�Æ�XEÏ/É ��Ì�Ë,Z1Ï<]fË Ó�ZJÉ�Æ�Ï/É�� �LÓ���Ì[Ì�Ë¡]���XDÓLÏY×EË�]�Ó>Õ>Ó�V_Ì{Ë�Õ Ñ�Ø�ÌqÍ�ËË�Õ�Z�Ì�Æ�X¡b�YF/0sCJ7>BD0��303?[4

ngq;<?�}6F>C d F+CQB�CN4WH9;<454GC1I>.10�BE;+H/=<B�23.N.LBGF/0@}�2�BE03?uCJ7>BD;6;<790���.N2�4�42�?D0PBGF/;<4,0+4�= d F)BGF<2�Bn = 2k gq;<?ä4,;<KO0+79;<79��790L��23BDC �<0+CJ7>BD0��303? k

b�L0�BG1 � G2 � . . . � Gn

8/0L79;�BD0~BGF/0M��.J2L45450L43b � 06}+CA.N.�8/0L79;�BD0OIL:(i, j)

BGF/0;<Hn03?[23BDCJ;<7)}6F>C d F d ;<7/4�CN4GB[4s;>gYH9;>=9?DCN79��}�2�BE03?äg1?D;<KGiBE;

Gjbãqg

n = 1 � BGF/037OBGF/0~��.J2L454 G1

d ;<7>BD23CJ7/4�23.N.nBGF/0+}�23BD0L?¡b+ãqgn = 2 � BGF/0L7OBGF/0;<Hn03?[23BDCJ;<7

(1, 2)H<=<BD4�23.N.�BGF/0)}�23BD0L?�CJ7>BD;

G2

bfãqgn = 2k }6F/03?D0 k > 1 � BGF/0L7BGF/064,0L�<=n037 d 0

(1, 2) � (3, 4) � . . . � (2k − 1, 2k)��C �<0L4s2 d ;<7<����=9?[2�BDC1;<7~}+CABGF

2k−1��.J2L45450L4 d ;<7>BD23CJ7<CN79�P0��>=92L./2�KO;>=97>BD4�;>g<}�23BD0L?¡b�¤Y0LH90L2�BDCJ79�sBGF>CJ4!H9?D; d 0L4�4 � }60P?D032 d F2 d ;<7<����=9?[2�BDC1;<7T}6F/0L?D0f2L.A.äBGF/0R}�2�BE03?+CJ4)CN7'2�4GCJ79��.J0���.J2L454 � }6F>C d F�H/?D;L�<034)BGF<2�Bn = 2k CJ4@2�H9;<454GC1I>.10��>23.N=n0�gq;<? n

b

��£ �

�P;�} d ;<7/4GC18/0L?s2�7�:n ≥ 2

gq;<?�}6F>C d FOBGF/03?D0~CJ4�2��97<CABD0~4,0L�<=n037 d 0M;>g_;<Hn03?G�23BDCJ;<7/46IL:�}6F>C d F�2L.A.�BGF/0~}�2�BE03? d 2�7TI90MH/=<B�CJ7>BD;T;<790O��.J2L454 � 452�: Gnb ;<7/4GC18/0L?2�7�:M4G= d FM450��>=n0L7 d 0 � 2�798�.J0�B N

I90PBGF/0�7<=9KRI/0L?�;>gu;<Hn03?[23BDCJ;<7/4�CJ7~BGF/0�450��>=n0L7 d 09b�L0�BxI/0~BGF/0~CJ7<CQBDCJ23.�2�KO;>=97>B�;>gu}�23BD0L?sCJ7�032 d F���.J2L4543b)��F>=94 � BGF/0)BE;�B[2L.�2LKR;>=97>B;>gn}�2�BE03?äCJ4

nxb � CQBGF/;>=<B�2L7�:M.J;<454s;>gu�30L7903?[2L.ACAB{: � }60�KM2�:M2L454G=9KR0�BGF<2�BäBGF/0+.J2L4GB;<Hn03?[23BDCJ;<7�CN7)BGF/0+4,0L�<=n037 d 0�CJ4

(n − 1, n)b�§G=94EB!I90�gq;<?D0PBGF>CJ4@;<Hn03?[23BDCJ;<7 � I/;�BGF Gn2�798

Gn−1

d ;<7>B[2LCN790�8�2L7+2LKR;>=97>Bu;>g3}�23BD0L?Y0��>=92L.3BD;nx/2 � 2�798+0,�<03?{:+;�BGF/0L?u��.N2�4�4}�2�4s03KOH>B�:ub� 0TH/?D; d 0L0�8áIL:â89034 d 0L798>CJ79�fCJ798>= d BDC1;<7�b��L0�BiI90�2�7 CJ7>BD0��303?~4G= d F'BGF<2�B

0 < i ≤ NbW�3=9H/Hn;<450PBGF<23Bä23gNBE03?

i;<H90L?[2�BDC1;<7/4 � gq;<?!032 d F j ∈ {1 � 2 � . . . � n} � BGF/02�KO;>=97>B�;>g�}�2�BE03? d ;<7>BD23CJ790L8OCJ7

GjCN4�;>gYBGF/06gq;<?EK

pjnx/2aj � }6F/03?D0 aj2L798

pj2�?D0+79;<79��790L��23BDC �<0+CJ7>BD0��303?[4Lb�L0�B(`, m)

I/0)BGF/0i��� ;<Hn03?[23BDCJ;<7�bMÜ<;<?�032 d F���.N2�4�4

Gj;�BGF/03?@BGF<2�7

G`2L798

Gm � BGF/0�2LKR;>=97>B�;>g/}�23BD0L?WCN7 GjCJ4!BGF/0+4�2�KO0�I90�gq;<?D0�BGF/0

i��� ;<Hn03?[23BDCJ;<7M2L4@2LgNBD0L?CAB�be©<:�;>=9?+CN798<= d BDCJ;<7'F�:/H9;�BGF/0L4GCJ4 � BGF/0f2LKR;>=97>B6;>gä}�2�BE03?+CN7 Gm

2LgNBD0L?PBGF/0i���;<Hn03?[23BDCJ;<7�CN4

pmnx/2am �BGF/03?D03gq;<?D0 � BGF/0+2LKR;>=97>B!I90�gq;<?D0PBGF/0 i

��� ;<H90L?[2�BDC1;<76}�2L4pmnx/2am+1 � }6F>C d F�CJ4!;>g>BGF/0�8/0L4GCJ?D0L8�gq;<?EK�b���F/0P2LKR;>=97>BW;>g�}�23BD0L?�CN7 G`

2LgNBD0L?BGF/0i��� ;<H90L?[2�BDC1;<7�CN4

p`nx/2a` �BGF/03?D03gq;<?D0 � BGF/0O2�KO;>=97>B+I/03gq;<?D0~BGF>CJ4�;<Hn03?[23BDCJ;<7}�2�4 ( p`

2a`+

pm

2am+1

)

nx � }6F>C d F d 2�7OI/060G �H/?D0L4�4,0L8�CN7~BGF/068/0L4GCJ?D0L8�gq;<?EK�b�YF>=94 � CNKOKO0�8>CJ2�BE0�.Q:Ý2LgNBD0L?PBGF/0 (i − 1)��� ;<Hn03?[23BDCJ;<7ÝCN7TBGF/0f4,0L�<=n037 d 0 � gq;<?0L2 d F

j ∈ {1 � 2 � . . . � n} � BGF/0T2LKR;>=97>B�;>g�}�23BD0L? d ;<7>B[2LCN790�8'CJ7 GjCJ4O;>gsBGF/0gq;<?EK

pjnx/2aj � }6F/03?D0 aj2�798

pj2L?D0O79;<79�G790���2�BDCQ�<0�CJ7>BD0��303?[4LbM�YF>CN4603798/4�BGF/0CJ798>= d BDC1;<7M4GBD0LH�bã1B_gq;>.A.1;�}�4�BGF<23B�BGF/0�CJ7<CQBDCJ23.<2�KO;>=97>Bä;>g>}�23BD0L?�CJ7

GnCN4

x = pnx/2a � }6F/03?D0a2�798

p2L?D0)79;<79�G790���2�BDCQ�<06CN7>BE0L�30L?[43bs�YF>=94 � n = 2a/p � g1?D;<K�}6F>C d F�}60)8/0�8>= d 0BGF<23B

pCN4@26H9;�}60L?!;>g

22�798M45;~CN4

nb

Þ b���F/06Hn;>CN7>BCCN4�;<7MBGF/06450���KO0L7>B

ABb c 4GBE?[23C1�3F�B!.ACJ790+BGF<?D;>=n�3F

CCN7>BE03?[4,0 d BD4BGF/0 d CN? d .10P}+CQBGFO8>CJ2LKR0�BE03?

AB23B

E2L798

F � BGF/0 d CJ? d .J0P}+CABGFM8<CN2�KO0�BD0L? AC2���2LCN723B

M � 2�798)BGF/0 d CN? d .10�}+CQBGF�8<CN2�KO0�BD0L? BC2���23CJ7�2�B

Nb�-9?D;L�<0�BGF<23B

MF = ENb

= ÆL\{ÊLË�ÕjÑ�Ø[Ë�ZNÐ9Ç>Ë¡\äU�ÒfË�Ï<Ð9ÇLÓ3\qÂ�Æ>Ê�ÓLÉ�]�Â/Ó3\JÓ �3ZJÐ/Ç<Ë�XDÓ3]¡Ë'Ó3\A\�Æ�XG×[Ó3] = VYÓ�Z1Ï � ¼/Z{Ë�XEXGËÆ�Æ�XEÏ/É ��Ì�Ë,Z1Ï<]¡Ë'Ó3ZNÉ�Æ�Ï<É � �LÓ ��Ì[Ì{Ë�] ��XDÓLÏY×EË � Â/Í9X[ZNÉ3Ì{Æ3VYÍYË�X�¿LÔ�Æ�X[Ó<Õ�\{Ë�ØL]YÆ�X[ZNÉ3Ì{ÆL\J] ��Ä �

�LÆ�É5Í<Z{Æ = Ë,Z1Ò�Z1Ø3ÓL]RÄYÓ>ÈsÓLÉ5Ó0�ZN]:¿,Ó�VYÓ�Ï � Ó�Ï�Õ�Æ�Æ9Ñ = Ë�X 0/Ë�Ø3] �>ËLÆ�Ï<ZJÓL]�¾N¿5] � = UPÔ �~Ë@Ð3Z{ÊLËÌqÍYË6ÈsX[Z�Ì�Ë �,Ç�VTÆ�Î/U�ÒfË�Ï<Ð9ÇLÓ3\8Â�Æ>Ê�ÓLÉ3bã{7TBGF/0��n��=9?D0 � BGF/0�.ACJ790 AM

CN4~0G LBD0L798/0�8BE;�KO0�0�BPBGF/0 d CJ? d .J0�}+CQBGFT8<CN2�KO0�BD0L?AB

2�BM ′ b�3CJ7 d 0

AC2L798

AB2�?D0�8<CN2�KO0�BD0L?[4 � ∠AMC

2L798∠AM ′B

2�?D0�?[C1�3F�B_2L79��.1034Lb���F/0�B�}6; d F/;<?D8/4EF2�798

BM ′ � I9;�BGFâHn03?EH90L798>C d =/.N2�?~BE; AM ′ � 2�?D0H92L?[2L.A.10�.<BE;~0L2 d F�;�BGF/0L?¡b!��037 d 0 � EFM ′BCJ4!2�7CJ45;<4 d 03.J0L4äBE?[2�H90���;>CJ8�b�3CJ7 d 0

BCCN4s2)8>CJ2LKR0�BE03? � ∠BNC = 90◦ b�YF/0 ?DCJ�3F�B�BE?DCN2�79��.J0L4

MFM ′ 2�798 NEB2�?D0d ;<79��?[=n0L7>B � 2L798OF/037 d 0 � MF = EN

b

.

.

..

.

..

..

.

..

.

..

..

.

..

..

..

..

..

..

..

..

.

..

..

..

.

..

.

..

.

..

.

..

..

.

..

..

..

..

.

..

..

..

..

..

..

..

..

..

..

..

..

..

..

..

..

..

..

..

..

..

..

..

..

..

..........................................................................................................

...........................................................................................................................................................................................................................................................................................................................................................................................................................................................................................................

..........................................................................................................................................................................................................................

.

..

.

..

.

..

..

..

..

..

..

.

..

..

..

..

..

..

..

..

..

..

..

..

..

...................................................................................................................

..

...

..

..

..

..

..

.

..

..

.

..

.

..

.

..

.

..

.

...

..

..

...

..

.................................................................................................................................................................. .

.

..

..

..

..

.

..

..

..

..

..

.

..

.

..

.

..

.

..

..

..

..

..

..

..

..

..

..

..

..

..

..

..

..

..

..

..

..................................................................

.......................................................................................................................................................................................................................................................................................................

.......................................................................................................................................................................................................................................................................................................................................................................................................................................................

..............................................................................................................................................................................................................................................................................................................................................................................................................................................................................

.

..

..

..

..

.

..

..

..

..

..

.

..

..

..

..

..

.

..

..

..

..

..

.

..

..

..

..

..

.

..

..

..

..

.

..

..

..

..

..

.

..

..

..

.......................................................................................................................................................................................................................................................................................................................................................................................

A BC

E

F

M

M ′

N

��£ �½ b@�YF/064�CJ89034�;>g�2~F/0LH>BD2��3;<7

A1A2A3A4A5A6A7

F<2��<0)0��>=92L.Y.J0L79�¡BGFubPÜ�?D;<K¯2Hn;>CN7>BOCN7/4�CJ890 � Hn03?EH90L798>C d =/.N2�?[4+2L?D0M8/?D;<H/Hn0L8�BE;OBGF/0M4GC18/0L4 A1A2 � A2A3 � . . . �

A7A1 � KO0�0�BDCJ79��BGF/03K � 2L798M79;�B_BGF/03CN?!0� 3BE037/4�CJ;<7/4 � 23B H1 � H2 � . . . � H7 � ?D0345H90 d �BDCQ�<0�.Q:ub@-9?D;L�<0PBGF<23BA1H1 + A2H2 + · · · + A7H7 = H1A2 + H2A3 + · · · + H7A1

b= ÆL\{ÊLË�Õ Ñ�Ø Ë�ZNÐ9Ç>Ë¡\�U�ÒfË�Ï/Ð/Ç�Ó�\MÂ�Æ>Ê�ÓLÉ�] Â/Ó3\JÓ �LZNÐ9Ç>Ë�X[ÓL]�Ë Ó�\N\{Æ�X�×�ÓL] = VYÓ�Z1Ï �Â/Í9X[ZNÉ3Ì{Æ3VYÍYË�XN¿LÔNÆ�X[Ó<Õ�\{Ë�ØL]�Æ�X[ZNÉ3Ì{ÆL\J] �8Ä � ÓLÏYÕ �3Æ�É,Í>Z�Æ = Ë,Z1Ò�Z1Ø3ÓL]�Ä�Ó<ÈsÓ3É,ÓF0�ZJ]�¿,Ó�VYÓLÏ�Ô�~ËPÐ3Z{ÊLË = Ë,Z1Ò�Z1Ø3Ó�É ÉPÈsX[Z�Ì{Ë �5Ç�VWb

......................................................................................................................................................................................................................................................................................................................................................................................................................................................................................................................................................................................................................................................................................................................................................................................................................................................................................................................

.................................................................................................................................................................................................................................................................................................................................................................

.

..

..

.

..

.

..

..

..

.

.

..

..

..

.

..

.

..

..

.

..

.

..

..

.

..

..

.

..

.

..

..

.

..

.

..

..

..

.

.

..

..

..

..

........

........

........

........

........

........

........

........

........

........

........

........

........

........

........

........

..

.

..

..

.

.

..

..

.

..

.

.

..

..

.

.

..

.

..

..

.

.

..

..

.

..

.

.

..

..

.

.

..

..

.

..

.

.

..

..

.

..

.

.

..

..

..

..

..

.

..

.

........

........

........

........

........

........

........

........

........

........

........

........

........

........

........

........

........

........

........

........

........

........

..

..

..

..

..

..

..

..

..

..

..

..

..

..

..

..

..

..

..

..

..

..

..

..

..

..

..

..

..

..

..

..

..

..

..

..

..

..

..

..

..

.

..

..

..

..

..

..

..

..

..

..

..................................................................................................................................................

............................................................................................................................................

.....................................................................................................................................................

...............................................................................................................................................................................................

.............................................................................................................................................................................................................

..

..

..

..................................................................................................................................................................................................................

.

.

..

.

..

.

..

.

..

.

..

.

..

.

..

.

..

.

..

.

..

.

..

.

..

.

..

.

.

..

.

..

.

..

.

..

.

..

.

..

.

..

.

..

.

..

.

..

.

..

.

..

.

..

.

..

.

..

.

..

.

..

.

..

.

.

..

.

..

.

..

.

..

.

..

.

..

.

..

.

..

.

..

.

..

.

..

.

..

.

..

.

..

.

..

.

..

.

..

.

..

.

..

.

.

..

.

..

.

..

.

..

.

..

.

..

.

..

.

..

.

..

.

A1

A2

A3

A4

A5

A6

A7

H1

H2

H3

H4

H5

H6

H7

O

....................................

....................................

....................................

....................................

....................................

....................................

.

.

.

.

.

.

.

.

............................

� 0O.J0�Ba8/0L79;�BD0MBGF/0M.10379�¡BGFÝ;>gWBGF/0O4�CJ890346;>gWBGF/0RF/0LH>BD2��3;<7 � 2L798f}60OH/=<B

A8 = A1

b!Ü>;<?i = 1 � 2 � . . . � 7 � 4GCJ7 d 0 OHi ⊥ AiAi+1 � }606F<2��<0

OA2i − OA2

i+1 = AiH2i − HiA

2i+1

= (AiHi + HiAi+1)(AiHi − HiAi+1)

= a(AiHi − HiAi+1)b

�YF>=94 �7∑

i=1

(OA2i − OA2

i+1) = a

7∑

i=1

(AiHi − HiAi+1)b

�3CJ7 d 0)BGF/0�4G=9K ;<7RBGF/0~.J03gNBs4�CJ890)CJ4�0L�<=923.uBD;O��0L?D; � BGF/0�4G=9K ;<7RBGF/0�?[C1�3F�B�4GC18/0~CN42L.N4,;6��0L?D;ub ;<7/4,0L�<=n037>BD.Q: �7∑

i=1

AiHi =

7∑

i=1

HiAi+1 �}6F>C d F~CJ4!BGF/0�89034�CN?D0�8�?D0L4G=/.AB�bå b~��F/0L?D0M2�?D0

2NH90�;<H<.10�23B�2MH92L?[B{:�bRß>2 d F���79;�}�4�2�B�.1032�4EB

N;�BGF/0L?[43bf-9?D;L�<0BGF<23B�;<790 d 2�7�2L. }�2�:<4 d F/;/;<450)gq;>=9?@Hn0L;<H/.J0~2L798�H/.N2 d 06BGF/0LK¯23Bs2~?D;>=9798RB[2�I>.10)4,;BGF<23B�032 d F�Hn03?[4,;<7O��79;�}�4sI9;�BGF�790�C1�3F/I/;>=9?[4Lb

��£ �

= ÆL\�Ç/ÌJZ{Æ�Ï�Ñ�Øu¼/Z{Ë�XEXGË¡Æ�Æ�XEÏ/É ��Ì�Ë,Z1Ï<]�Ë Ó�ZJÉ�Æ�Ï/É�� �LÓ���Ì[Ì�Ë¡] ��X[Ó�Ï�×GË�b� 0'2L454G=9KR0TBGF<23B�BGF/0'?D0�.J2�BDC1;<7 �

A��79;�}�4

B��CN4�4[:/KMKR0�BE?DC d b � 0�2L.N4,;2�4�4�=9KO06BGF<23B

N ≥ 2 � 4GCJ7 d 06BGF/0L?D0)CJ4�79;�450�B�;>g�gq;>=9?�8>CJ4EBDCJ7 d BsH90L?[45;<7/4�CNg N = 0;<?N = 1

b�CN?[4GB d ;<7/4�CJ8903?nBGF/0 d 2L4,0N = 2

b��L0�BA � B � C � D I/0!BGF/0@Hn0L;<H/.J09b_ãqg�BGF/0,:�2L.A.��79;�}�;<790)2�79;�BGF/03? � BGF/0L7f0L2 d FRHn03?[4,;<7�2�B�BGF/06B[2�I>.10+}+CA.N.���79;�}�F>CN4P790�C1�3F/I/;>=9?[4 �79;�KO2�B[BE03?s}6F<23BP4,0323BDCN79�M2L?E?[2L79�30LKO0L7>BPCJ4�=94,0L8�b �!BGF/0L?�}+CJ450 � }60OKM2�:�4G=9H9H9;<4,0BGF<23B

A8/;/0L4@79;�B���79;�}

Cb��YF/037

A2�798

CKM=94EB@032 d FM��79;�}

B2L798

D � 2L798�BGF/04,0323BDCN79��2�?D?[2�79�303KR037>BA�B�C�DCN4@2645;>.N=<BDCJ;<7�b�P;�}#4�=9H/Hn;<450

N ≥ 3b ;<7/4GC18/0L?�BGF/0O4GCJKMH/.J0R��?[2LHnF�}6F/;<450~�<03?[BDC d 034�2�?D0BGF/0+H90�;<H<.10 � B�}6;~;>g/}6F>C d F~2L?D0 ;{;>CN790�8�IL:~2�7M0L89�30PCNgn2�798�;<7<.Q:)CAg9BGF/0,:���79;�} 0L2 d F;�BGF/0L?¡b��YF/0+H9?D;/I>.103K�CJ4äBE;~H/?D;L�<0PBGF<23B_BGF>CN4s��?[2�H9F d ;<7>B[2LCN7/4@26�>=92�8<?DCA.J2�BE03?[2L.��ABGF<23BCJ4 � 2 4

��d CN? d =/CAB � b c�dEd ;<?D8>CJ79�äBE; � � �q0G �0L? d CN4,0 � ��� � H�b �: �G� 0,�<03?{:+4�CNKOH<.10s��?[2LHnF�}+CQBGF2N

�<0L?�BDC d 0L4@2�798M2�Bä.J0L2L4GB N

2(1 +

√8N − 3)

0�8/�30L4 d ;<7>B[2LCN7/4@2)�>=92�8<?DCA.J2�BE03?[2L.�b�L0�BAI/0�2�7�: ;<790T;>g@BGF/0R�<0L?�BDC d 0L4 � 2L798'.10�B d(A)

I90fCAB[4M890L��?D0�0�CN7'BGF/0��?[2�H9Fub�Ü�?D;<K BGF/0���CQ�<037~CN7<gq;<?EKM23BDCJ;<7 � }60+F<2��<0 d(A) ≥ Nb_�3CJ7 d 0�2�7�:~0�8/�30

ABCJ4 d ;>=97>BD0�86CN7~I9;�BGFd(A)

2�798d(B) � }60�8/0�8>= d 0�BGF<23B�BGF/0sBE;�B[2L.<7<=9KRI/0L?W;>gn0�8/�30L4CJ4s2�B!.J0L2L4GB

N2bsã1B!CN4�032�4[:~BE;6�<0L?[CNgA:~BGF<23B

N2 ≥ N

2(1 +

√8N − 3)

gq;<?N ≥ 3

b�YF/0 d ;<7 d .N=94GC1;<7~gq;>.N.J;�}�4Lb����&P��#���$ ��� ��� �/b9�L;L��@2�4E� � Â�Æ�Ò�ÑLZ1Ï/Ó/Ì{Æ�X[Z1Ó�\�¼nX�Æ9ÑL\{Ë�ÒMÉäÓ�Ï�Õ ���<Ë�X�×�ZNÉ�Ë¡É � ��;<?[BGF/�,��;>.A.J2L798 � �" � bæ b@-9?D;3�<0�BGF<2�BW2�7�:)79;<79��790L��23BDC �<0+0,�<0L7)CJ7>BD0��303? d 2L7�I90P=97<CJ�<=n0�.Q:~?D03H9?D034,037>BE0L8�2L4(x + y)2 + 3x + y

}6F/03?D0x2L798

y2L?D0�79;<79�G790���2�BDCQ�<0+CN7>BE0L�30L?[43b

= ÆL\{ÊLË�ÕâÑ�Ø[¼/Z�Ë�XEX�Ë]Æ�Æ�XEÏ/É ��Ì�Ë,Z1Ï<]zË'Ó3ZNÉ�Æ�Ï<É � �LÓ ��Ì[Ì{Ë�] ��XDÓLÏY×EË � ÓLÏYÕ Â/Í9X[ZNÉ3Ì{Æ3VYÍYË�X�¿LÔÆ�XDÓ>Õ�\�Ë�Ø3]8Æ�X[ZJÉ�Ì�ÆL\N]���ÄäÔ �~ËPÐ3Z{ÊLË¡Æ�Æ�XEÏ/É ��Ì�Ë,Z1Ï�É É@É�ÆL\�Ç/ÌJZ{Æ�Ï/]YÓ>Õ>Ó�V_Ì{Ë�ÕRÑ�ØMÌqÍ�Ë6ËLÕ�Z�Ì{Æ�X¡b

�L0�BnI/0+2�79;<79�G790���2�BDCQ�<0�0,�<0L7~CJ7>BD0��303? � 2L798~.10�B x

2L798yI/0+79;<79��790L��23BDC �<0CJ7>BD0��303?[4s4�= d F)BGF<2�B

(x + y)2 + 3x + y = nã1B�CN460L2L4[:fBE;�450�0�BGF<2�Bn = 0

CNgä2�798Ý;<7<. :�CNgx = y = 0

bTÜ>?D;<KÂ79;�}¯;<7 � }602�4�4�=9KO0�BGF<2�Bn ≥ 2

b�L0�BaI/0�BGF/0)H9;<4�CQBDCQ�<0+CJ7>BD0��303?@4�= d F�BGF<2�B

a2 ≤ n < (a + 1)2b��P;�BD0�BGF<2�B

a2 + a − 22�798

a2 + a2L?D0�B�}6; d ;<7/450 d =<BDC �<0+0,�<0L7~CN7>BE0L�30L?[43b ;<7/4,0L�<=n037>BD.Q: � BGF/0gq;>.N.J;�}+CJ79��B�}6; d 2�450L4s0G >F<23=94GB!23.N.nH9;<454GC1I>CN.ACABDCJ0L4 �

pE�"6���$�ba2 + a ≤ n < (a + 1)2

bãqgx + y ≥ a + 1 � BGF/0L7

n = (x + y)2 + 3x + y ≥ (a + 1)2 > n �2 d ;<7>BE?[2�8>C d BDCJ;<7�b!ãqgx + y ≤ a − 1 � BGF/0L7

n = (x + y)2 + 3x + y ≤ (a − 1)2 + 3(a − 1)

= a2 + a − 2 < a2 + a ≤ n �

��£�

2 d ;<7>BE?[2�8>C d BDCJ;<7�b��YF/03?D03gq;<?D0 � x + y = ab��YF/037

n = (x + y)2 + 3x + y = a2 + a + 2xb

��037 d 0 � x = 1

2(n − a2 − a)

2L798y = a − x = 1

2(a2 + 3a − n)

b�P;�}ç.J0�Bx2�798

yI90~8/03�9790�8�IL:MBGF/0L450~0� �H9?D03454GC1;<7/43b��3CN7 d 0

nCN4P0,�<037�2L798

a2 ≡ a (mod 2) � I9;�BGF x2L798

y2L?D0+CJ7>BD0��303?[4LbW�3CN7 d 0

a ≥ 1 � }60�F<2��<0a2 + 3a ≥ a2 + 2a + 1 = (a + 1)2 > n �g1?D;<K�}6F>C d F)}606890L8<= d 0+BGF<2�B

y > 0b!�3CJ7 d 0

n ≥ a2 + a�Ngq;<?äBGF>CN4 d 2L4,0 �G� }60�2L.N4,;F<2��<0

x > 0b���F>=94 � BGF/0)H923CJ? (x, y)

��C �<0L4�2)=97<CJ�<=n0~?D03H9?D034,037>BD2�BDC1;<7�;>gnCJ7OBGF/089034�CN?D0�8�gq;<?DKTb

pE�"6�� �9ba2 ≤ n ≤ a2 + a − 2

bãqgx + y ≤ a − 2 � BGF/0L7

n = (x + y)2 + 3x + y ≤ (a − 2)2 + 3(a − 2)

= a2 − a − 2 < a2 ≤ n �2 d ;<7>BE?[2�8>C d BDCJ;<7�b!ãqgx + y ≥ a � BGF/037

n = (x + y)2 + 3x + y ≥ a2 + a + 2x ≥ a2 + a > n �2 d ;<7>BE?[2�8>C d BDCJ;<7�b��YF/03?D03gq;<?D0 � x + y = a − 1b_��F/0L7

n = (x + y)2 + 3x + y = (a − 1)2 + a − 1 + 2x = a2 − a + 2xb

��037 d 0 � }60�F<2��<0 x = 1

2(n − a2 + a)

2�798y = a − 1 − x = 1

2(a2 + a − n − 2)

b�P;�}�.J0�Bx2�798

yI/0O8/03�9790�8�IL:�BGF/034,0M0� �H9?D03454GC1;<7/43b c 4PCJ7 2�450 � � I/;�BGF

x2�798

y2L?D0fCJ7>BD0��303?[4Lbe�3CN7 d 0

a2 ≤ n2L798

a > 0 � }60�F<2��<0 x > 0be�3CJ7 d 0

n ≤ a2 + a − 2�Ngq;<?sBGF>CN4 d 2�450 �E� }60�2L.N4,;fF<2��<0 y > 0

b~�YF>=94 � BGF/0MH/2LCN? (x, y)��CQ�<034s2+=97<C1�>=n0�?D0LH/?D0L450L7>B[23BDCJ;<7R;>gnCJ7~BGF/068/0L4GCJ?D0L8�gq;<?DKTb

� b�ã{7fBE?DCN2�79��.J0ABC � BGF/0M4�CJ89034+452�BDCJ4GgA: AB + AC = 2BC

b�-9?D;3�<0~BGF<23B�BGF/0I<CN4,0 d BE;<?�;>g∠A

CN4sH90L?DHn03798<C d =/.J2L?äBE;)BGF/0+.NCN790�4,0L��KR037>B ;{;>CN7<CJ79�PBGF/0+CN7 d 037>BE?D062L798d CJ? d =9K d 037>BE?D0);>gABC

b= ÆL\{ÊLË�Õ'Ñ�Ø Ë�Z{×DÍYË,\�Æ�Ó/ÌJÓ3ZA\N\{Ë�]§Å_ÆnÇ>Ë�Ï<] ��X[Ó�Ï�×GË � Â/Í/X[ZJÉ�Ì�Æ3VYÍ�Ë�X+¿LÔ�Æ�X[Ó<Õ�\{Ë�ØL]§Æ�X[ZJÉ�Ì�ÆL\N]��Ä � Ó�Ï�Õ �3Æ�É,Í>Z�Æ = Ë¡ZJÒ�ZqØ3Ó3]8Ä�Ó<ÈsÓ3É,ÓF0�ZJ]�¿,Ó�VYÓLÏ�Ô �~Ë�Ð3Z{ÊLË = Ë,Z1Ò�Z1Ø3Ó�É ÉsÓ�X�Ð9ÇLÒfË�ÏYÌDb

� 0+2L454G=9KR0�BGF<2�BAB 6= AC

b��L0�BI2�798

OI/0BGF/0�CJ7 d 0L7>BE?D0+2�798 d CN? d =9K d 0L7>BE?D0�;>g 4ABC � ?D0L4�Hn0 d �BDCQ�<0�.Q: � 2�798�.10�B D

I90MBGF/0RCN7>BE03?[4,0 d BDC1;<7�;>gAI

}+CABGFBC

b_�3CN7 d 0BI

2L798CI

I<CN4,0 d B∠ABD

2�798∠ACD �?D0L4�Hn0 d BDCQ�<0�.Q: � }606F<2��<0

AI

ID=

AB

BD=

AC

CD

=AB + AC

BD + CD=

AB + AC

BC= 2

b

..

.

..

.

..

.

..

.

..

.

..

..

..

..

..

..

..

..

..

..

..

.

..

..

.

..

.

..

.

..

..

..

..

..

..

.

..

..

..

..

..

..

..

..

..

..

..

..

..

..

..

..

..

..

..

..

..

..

.................................................................................................

.....................................................................................................................................................................................................................................................................................................................................................................................................................................................

.................................................................................................................................................................................................

..

..

..

..

..

..

..

..

..

..

..

..

..

..

..

..

..

..

..

..

..

..

..

..

..

..

..

..

..

..

..

..

..

..

..

..

.

..

..

..

..

..

..

..

..

..

..

..

..

..

..

..

..

..

..

..

..

..

..

..

..

..

..

..

..

..

..

..

..

..

..

..

..

..

..

..

..

..

..

..

..

.

..

..

..

..

..

..

..

..

..

..

..

..

..

..

..................................................................................................................................................................................................................................................................................................................................................................................................................................................................................................................................................................................................................................................................................................................................................................................................................................................................................................

...................................

..

..

..

..

..

..

..

..

..

..

..

..

..

..

..

..

..

..

..

................................................................

.........

...........

.

.

.

.

.

.

.

.

.

..

.

.

A

B CD

I

M

O

��¢L�

��037 d 0 � AB = 2BD2�798

AI = 2IDb ;<7/450��>=n0L7>BD. : �

AD = 3 IDb � � ��L0�B

MI90WBGF/0!4,0 d ;<798�CJ7>BD0L?[450 d BDCJ;<7�;>g

AD}+CABGF@BGF/0 d CN? d =9K d CN? d .10�;>g 4ABC

b�YF/037∠MBD = ∠MBC = ∠MAC = ∠MAB

b�3CJ7 d 0

∠BMD = ∠BMA � }60��30�B 4MBD ∼ 4MABb!ã1Bägq;>.N.J;�}�4!BGF<2�B

DM

BM=

BM

AM=

BD

AB=

1

2

b�YF>=94 �

DM

AM=

DM

BM· BM

AM=(

1

2

)2

=1

4

b��037 d 0 � AM = 4DM � g1?D;<K }6F>C d F6}60�;/I>B[2LCN7

AD = 3DMb � � �Ü>?D;<K�� � � 2�798�� � � }60+�30�B ID = DM

b���F>=94 � AI = 2ID = ID + DM = IMb�YF/03?D03gq;<?D0 � OI ⊥ AM

b���F>CJ4�CJKMH/.AC1034!BGF<23BAI ⊥ OI

b��/Õ�Ô_��0�0sBGF/0�4,;>.A=<BDC1;<7�BE;M-9?D;/I<.J0LK ��� � � .N23BD0L?uCJ7+BGF>CJ4WCN454G=n0 � 0L4�Hn0 d CJ23.N. :+BGF/00�8>CABD;<?DCN2L. d ;<KOKO0L7>B[4@gq;>.A.1;�}+CN79��BGF/0+4,;>.A=<BDC1;<7�b �

� b�Ü>;>=9?PHn0L89034GBE?[CJ2L7/4P}603?D0OKO;L�>CJ79��23B�=97<CAgq;<?EK´�<0�.1; d CQBDC1034+23.1;<79�~gq;>=9?P4EBE?[2LCJ�3F�B?D;<2�8<4@CN7O�30L7903?[2L.nHn;<4GCABDCJ;<7/4Lb��>}6;M;>gnBGF/03K#KO0�B�032 d FM;�BGF/03?!2L4ä}603.A.n2�4!BGF/0�;�BGF/0L?B�}6;ub@-9?D;L�<0PBGF<23BWBGF/0�;�BGF/0L?_B�}6;�23.J45;�KO0�B�b= ÆL\{ÊLË�ÕâÑ�Ø[¼/Z�Ë�XEX�Ë]Æ�Æ�XEÏ/É ��Ì�Ë,Z1Ï<]zË'Ó3ZNÉ�Æ�Ï<É � �LÓ ��Ì[Ì{Ë�] ��XDÓLÏY×EË � ÓLÏYÕ Â/Í9X[ZNÉ3Ì{Æ3VYÍYË�X�¿LÔÆ�XDÓ>Õ�\�Ë�Ø3]8Æ�X[ZJÉ�Ì�ÆL\N]���ÄäÔ �~ËPÐ3Z{ÊLË¡Æ�XDÓ>Õ�\�Ë�ØEÉ É�ÈsX[Z�Ì�Ë �,Ç�VWb

..

..

..

..

..

..

..

..

..

..

..

..

..

..

..

..

..

..

..

..

..

..

..

..

..

..

..

..

..

..

..

..

..

..

..

..

..

..

..

..

..

..

..

..

..

..

..

..

..

..

..

..

..

..

..

..

..

..

..

..

..

..

..

..

..

..

..

..

..

..

..

..

..

..

..

..

..

..

..

..

..

..

..

..

..

..

..

..

..

..

..

..

..

..

..

..

..

..

..

..

..

..

..

..

..

..

..

..

..

..

..

..

..

..

..

..

..

..

..

..

..

..

..

..

..

..

..

..

..

..

..

..

..

..

..

..

..

..

..

..

..

..

..

..

..

..

..

..

..

..

..

..

..

..

.......................

......................................

.....................................

......................................

.....................................

......................................

.....................................

.............................................................................................................................................................................................................................................................................................................................................................................................................................................................................................................................................................................................................................................................................

.....................................................................................................................................................................................................................................................................................................................................................................................................................................................................................................................................................................................P (t = t1)

Q(t = t4)

R(t = 0) S(t = t3)X

Y (t = t2)

(t = t5)

(t = t6)

t3x |t5 − t3|x

|t4 − t1|w |t6 − t4|wt1u

(t2 − t1)u |t2 − t4|v

|t4 − t5|v-����������

A

� ���������B

?

����������C

6����������

D¥@0L79;�BD0�BGF/0+Hn0L89034GBE?[CJ2L7/4sIL:A � B � C � D b � CABGF/;>=<BW.1;<4�4s;>gY�30L7903?[2L.ACAB{: � }60KO2�:�2�4�4�=9KO06BGF<23B�BGF/0~�9?[4GBsH/2LCN?@BD;RKR0L0�B!}�2�4

A2�798

D � 2L798RBGF<2�B@BGF/0,:�KR0�B�2�BBDCJKO00b6�P0G LBs4G=9H9H9;<4,0�BGF<2�B

A2�798

CKR0�Bs2�B

t12�798MBGF<23B

A2L798

BKO0�Bs2�B

t2 �}6F/0L?D0t2 > t1 > 0

bä�3=9H/Hn;<450�gJ=9?[BGF/03?_BGF<23BB2�798

DKR0�B!2�B

t32�798)BGF<23B

B2L798

CKO0�B!2�B

t4b � 0�}�2L7>BWBE;~4,F/;�}�BGF<2�B

C2L798

D2L.N4,;~KR0�B�b�YF/0f?D;<2�8<4~2L.J;<79�M}6F>C d F

C2�798

D}60L?D0RBE?[2��<03.A.NCN79��KO0�0�B62�B62�H9;>CJ7>B

Xb�3=9H9H9;<4,0sBGF<23B

D?D0L2 d F/0�8

X23BuBDCNKR0

t52L798

C?D0L2 d F/0�8

X23BuBDCNKR0

t6b�¥!CN4GB[2�7 d 0L4

��¢��

2�?D0)2�4�45F/;�}�7RCJ7MBGF/0)�n��=9?D0 � 2L454G=9KMCN79�+BGF<23B t4CJ4PI/0�B�}60L0L7

t22L798

t3b � 0~F<2��<02L.N4,;�890379;�BE0L8OIL:

u � v � w � 2L798 xBGF/0�?D0L4�Hn0 d BDCQ�<0+45H90�0L8/4s;>g

A � B � C � 2�798 Db¦s4�CN79���'037903.N2L=94��3�YF/0L;<?D0LK�;<7 4Y RS

}+CQBGF+BE?[2�7/4[�<03?[4523.PQX � }60�F<2��<0

RX

XS· SQ

QY· QP

PR= −1 =

t5

t3 − t5· t4 − t3

t2 − t4· t1 − t2

−t1

b � � �¦s4�CN79�)�'0L790�.J23=94�����F/0�;<?D03KÙ;<7 4PRX

}+CQBGF)BE?[2�7/4[�<03?[4523.Y QS � }60�F<2��<0

PY

Y R· RS

SX· XQ

QP= −1 =

t2 − t1

−t2· t3

t5 − t3· t4 − t6

t1 − t4

b � � �ß/�>=923BDCJ;<7�� � � ��CQ�<034 t1t2t3 + t1t3t5 + t2t4t5 = t1t2t5 + t1t3t4 + t2t4t5 �2�798�0L�<=92�BDC1;<7�� � � ��CQ�<034 t1t2t3 + t1t3t6 + t2t4t5 = t1t2t5 + t1t3t4 + t2t3t6

b�3=nI>BE?[2 d BDCJ79��:<CJ03.J8/4(t1t3 − t2t3)(t5 − t6) = 0

b�©9=<Bt1 6= t2 �

}6F/0L7 d 0 � t6 = t5 �CJKMH/. :<CN79�PBGF<2�BC2L798

DKR0L0�B�b

� � -���������� �� ������������� �������� ��������������� ��!"�����#�$ %&�t ��� ����� �$!'!� %&�(���� ���) �"�����������*�+ �(%�,

�+ �����.-) ���/�����.�-��0���� �$ �+�!� %&�(���� �����x

���!y��1#�2�����)����!��(�$���3 ����2%&��4��) �/�����

xy 5 ��+ �����,������768 ��+ + ��-9�� �����. �xyt 5 ���� ������):( ���$�0�� ����&����!����$�$� ��&%;��4��(�) �. =<��� 8 ���%>4�� + ��� �?6; �+ ������ 0�$���3 � �����

+ ���@ �������xy 5 ��+ �����, �����"������$�(��������!� ������ (�����#� 8 �����.����!��(�$���3 ����� �

xyt 5 ���� ����. ���"�$���3 � ������+ �������A -��0����!��(�$���3 �����%&��� �2 (�2�7��%;�)�$ %&�) 8 ���!B����+ 6= 8 �����( �#�� (�����2 �

xyt5 ���� ��$�' ���$���3�7���3���C �D�������.�� �����E� 8 �����@����!����$�$� ����� �

xyt5 ���� ��$�"*��2������+ �����L1

,L2

,L3

,� ���!L4

��1��$�����!� ������B�����B� 4��(�F � 8 ���$%B (� ���G,G�� ����F�� ( �"� 8 �����(�7�=+ ���(�" �����(��7�����(,�-' ���F�����0�������� *�+ �0��H����(���$ ����IJ�� (6L3

���!L4

� A ���0�����$���;+ �����L1

,L2

,� ���!L3

%0<��$��*��F�$����+ ��� ��D,��� ���$�&�� ����K�� ( �) �%&�����0������%�� ��. ��B �����(��7���� �����2:( %B + ��+ 6G,

L1

,L2

,� ���!L4

��$�/������+ ��� �� � A ���(�$� 8 �����,� �+ + 8 ��<��2+ ������+ �' �0������� �%;�B��+ ������;L)��-9-��B�����$�=���� �

L3

���!L4

� ��������)*��B�� �� (+ + �(+?,G*���� �<��7�=���� �"-���<�+ !F %;��+ 6&���� �����!��(�$���3 ����

3 ���!

4-/�(�$�/-' �+ M� ���=���N�� �� (+ + �(+���� �!��)O (�@�����=�� �%;�0��������!�P�,�-/�� ���& �)�����. �+ + ��-���!

8 ���%Q�����F�����*�+ �(%R�$�$ ���(%&�(���(�JO S@���;-� 6����K�7���;���� �/ �����T�$������ !����)�����F�����UV���3�$ �����0� 8L3

���!L4

�����$�������;�����$�����$�����$!� �� ���F��+ ������/ �xyt5 ���� ��$��� P A ������ 8 ���$��,

L3

���!L4

���$���3�7���3��� A ��<��(,����!��(�$���3 ����

3 ���!

4%;���D��� W

Û�� b c B T CJ79� c ?�BGF>=9?,� 4 ;>=9?�B � 2n��7<C1�3F�B[4P��2�BGF/0L?D0L8R23B!BGF/0O¤Y;>=9798O�/2�I<.J09bPß<2 d FF<2�4s2�B�KO;<4GB

n − 103790LK�C1034s2�KO;<79�PBGF/06;�BGF/03?[4Lb�-9?D;3�<0�BGF<23B��'0L?[.NCN7�BGF/0P}+CQ�G2�?D8d 2L7f890,�>CJ450~2~450L2�BDCJ79��2�?D?[2�79�303KR037>B�4�= d FMBGF<2�Bs79;���7<C1�3F�B!}+CA.N.uI90)790� 3B�BE;O2�7�:�;>gF>CJ4s0L7903KMCJ0L43b

= ÆL\�Ç/ÌJZ{Æ�Ï�Ñ�Øu¼/Z{Ë�XEXGË¡Æ�Æ�XEÏ/É ��Ì�Ë,Z1Ï<]�Ë Ó�ZJÉ�Æ�Ï/É�� �LÓ���Ì[Ì�Ë¡] ��X[Ó�Ï�×GË�b� 0�2�4�4�=9KO0)BGF<2�BsBGF/0�?D03.N23BDCJ;<7 �

ACJ4P79;�B�2L7�03790LK):�;>g

B�+CJ4P4[:/KMKR0�BE?DC d b

;<7/4GC18/0L?!BGF/0)4�CNKOH<.10~��?[2LHnF�}6F/;<450+�<0L?�BDC d 0L4�2L?D0�BGF/0���7<C1�3F�B[4 � B�}6;R;>gY}6F>C d FR2�?D0;{;>CJ790L8 IL:�2L7 0L89�30�CNg@2�798';<7<. :ÝCNg!BGF/0,:�2�?D0f79;�B60L7903KMCJ0L4~;>g�032 d F�;�BGF/0L?¡b ��F/0H9?D;/I>.103K�CJ4!BGF/037�BD;�H/?D;L�<0�BGF<2�BWBGF>CJ4s��?[2LHnF d ;<7>BD23CJ7/4@2~�+2LKMCA.ABD;<7<CJ2L7 d CJ? d =/CQB�b�L0�BK1 � K2 � . . . � K2n

I/0+BGF/06��7<CJ�3F�BD4 � 2L798 � gq;<? i = 1 � 2 � . . . � 2n � .J0�B diI90)BGF/0M8/0���?D0L0M;>gKib�Ü>?D;<KÙBGF/0�4GB[23BD0LKO0L7>B�;>g�BGF/0�H/?D;/I<.J0LK � }60�F<2��<0 di ≥ ngq;<?�032 d F

ib_��F>=94 � min

i{di} ≥ n

b©9=<B � 2 dEd ;<?D8>CJ79�+BD; ��� �10� �03? d CJ450 ��� ��2 � H�b � � �G� 0,�<0L?{:R4GCJKMH/.J0~��?[2LHnF�}+CQBGF k�<0L?�BDC d 0L4�4�23BDCN4�gA:>CJ79�

mini

{di} ≥ k

2

�ABGF>CJ4+CN46��79;�}�7�2�4�¥!CJ?[2 d � 4 d ;<798<CQBDC1;<7 � F<2L4�2�+2LKMCA.ABD;<7<CJ2L7 d CJ? d =/CQB�b��YF/0 d ;<7 d .N=94GC1;<7~gq;>.N.J;�}�4Lb����&P��#���$ ��� ��� �/b9�L;L��@2�4E� � Â�Æ�Ò�ÑLZ1Ï/Ó/Ì{Æ�X[Z1Ó�\�¼nX�Æ9ÑL\{Ë�ÒMÉäÓ�Ï�Õ ���<Ë�X�×�ZNÉ�Ë¡É � ��;<?[BGF/�,��;>.A.J2L798 � �" � b

��¢��ÛPÛ b ;<7/4EBE?D= d BP2R450�B�;>g d CN? d .1034P}+CQBGF�79;<79�E��03?D;�?[2�8>CNCW4G= d FfBGF<2�B+0G L2 d BD. :T;<790O;>gBGF/0LK#H/2�4�4,034!BGF<?D;>=n�3FO032 d F�H9;>CJ7>B�;>gnBGF<?D0L0���8<CNKR037/4�CJ;<7/2L.n4�H92 d 0/bÂ�Æ�ÒOÒfË�ÏYÌWÑ�Øu¼/Z{Ë�XEXGË¡Æ�Æ�XEÏ<É ��Ì{Ë¡ZJÏ/]�Ë Ó�ZJÉ�Æ�Ï/É�� �LÓ���Ì[Ì�Ë¡] ��XDÓLÏY×EË>b

�YF>CN4sH9?D;/I>.103K � }+CQBGF~CAB[4s45;>.N=<BDCJ;<7 � 2�H/Hn032�?[4s2�4�ßL �03? d CJ450 ��� ¨ÝCJ7�-�b��+23.JKO;<4 �¼nX�Æ9ÑL\{Ë�ÒMÉ@Î�Æ�X+Ë Ó<ÌqÍYË�ÒOÓ/ÌJZ{×�Z1ÓLÏ/É6Å/ÆnÇ�Ï<Ð�Ó�Ï�Õ ��\�Õ � � c_c � �" : �� bÛ Þ b@¥@;/0L4!BGF/03?D060G �CN4GBä26H9;<4�CQBDCQ�<0PCJ7>BD0��303?n4�= d F)BGF<23B

27n + 84n + 110n + 133n = 144n �= ÆL\{ÊLË�ÕÑ�Ø ¼/Z�Ë�XEX�Ë Æ�Æ�XEÏ<É ��Ì{Ë¡ZJÏ/] Ë Ó�ZJÉ�Æ�Ï/É�� �LÓ���Ì[Ì�Ë¡] ��XDÓLÏY×EË � Â/Í/X[ZJÉ�Ì�Æ3VYÍ�Ë�X ¿LÔÆ�XDÓ>Õ�\�Ë�Ø3]Æ�X[ZNÉ3Ì{ÆL\J] �8Ä � �+Ô ¿LÔ = ÒfËLË�Ï 0�]�ÁLÓ�\�Ì[Ñ/Æ�ÒOÒfË,\J]@ÌqÍYËu¾�Ë�ÌqÍ�Ë�X[\JÓ�Ï�Õ�ÉLÔ �~Ë6Ð3Z{ÊLËÆ�Æ�XEÏ/É ��Ì�Ë,Z1Ï�É ÉsÉ�ÆL\[Ç<ÌJZ�Æ�Ïub

�L0�Bf(x) =

(

27

144

)x

+

(

84

144

)x

+

(

110

144

)x

+

(

133

144

)x

890��n790L8f;<7(0, ∞)

b�ã1B�CJ4P032�4[:MBD;R450�0�BGF<2�B�BGF/0)gJ=97 d BDCJ;<7fCJ4P8/0 d ?D032�4GCJ79�<b���F/0L7BGF/0R0L�<=92�BDC1;<7

f(x) = 1F<2�4+23B�KR;<4EB�;<790M45;>.N=<BDCJ;<7fCJ7�H9;<4�CQBDCQ�<0�?D0L23.W7<=9KOI903?[4Lb�3CJ7 d 0

f(5) = 1 � CABägq;>.A.1;�}�4äBGF<23B n = 5CN4!BGF/06;<7<. :M45;>.N=<BDCJ;<7M;>gnBGF/0�H/?D;/I<.J0LK�b

Û ½ b@-9?D;L�<0PBGF<23B∞∑

i,j=1

aiaj

i + j≤ π

∞∑

k=1

a2k

b= ÆL\�Ç/ÌJZ{Æ�Ï�Ñ�Øu¼/Z{Ë�XEXGË¡Æ�Æ�XEÏ/É ��Ì�Ë,Z1Ï<]�Ë Ó�ZJÉ�Æ�Ï/É�� �LÓ���Ì[Ì�Ë¡] ��X[Ó�Ï�×GË�b

�YF>CN4sCN4s2)4�Hn0 d CJ23. d 2L4,0(p = 2)

;>gY��F/0�;<?D03K ����¢ CJ7 � � � }6F>C d FM4GB[23BD0L4 � �L0�Bp > 1

2�798p′ =

p

p − 1

b!ãqg ∞∑

n=1

apn ≤ A

2L798 ∞∑

n=1

bp′

n ≤ B � BGF/0L7∞∑

m=1

∞∑

n=1

ambn

m + n≤ π

sin(π/p)A

1p B

1p′b

�';<?D0�;3�<0L? � BGF/0 d ;<7/4EBD2L7>B π

sin(π/p)

CJ4!BGF/0�I9034GB!H9;<454GC1I>.10/b����&P��#���$ ��� ��� ¨�bJ��b �+2L?D8�: � §3bJßub ��CQB[BD.10¡}6;/;/8 � ¨sb - @;>.Q:>2 � Ç{Ï�Ë�Ê�ÇLÓ3\AZ�ÌJZ{Ë�É � 2�KOI/?[C18/�30��23BGF/03KO2�BDC d 2L.���C1I<?[2�?{: � �� �¢�� � H�b ��� � �>����� bÛ æ b@¥@0 d ;<KOH9;<4,0

2352 + 9722CJ7>BD;)B�}6;~g12 d BE;<?[43b

= ÆL\{ÊLË�Õ Ñ�Ø Ë�ZNÐ9Ç>Ë¡\�U�ÒfË�Ï/Ð/Ç�Ó�\MÂ�Æ>Ê�ÓLÉ�] Â/Ó3\JÓ �LZNÐ9Ç>Ë�X[ÓL]�Ë Ó�\N\{Æ�X�×�ÓL] = VYÓ�Z1Ï �Â/Í9X[ZNÉ3Ì{Æ3VYÍYË�X�¿LÔ3Æ�XDÓ>Õ�\�Ë�Ø3]�Æ�X[ZJÉ�Ì�ÆL\N] �8Ä � ÓLÏYÕ �/Õ9ÈsÓ�XGÕ �LÔ �~Ô ��Ó�Ï<ÐL]���ZA\JÎ1X[Z�Õ �LÓ9ÇLX[Z�Ë�X��Ï>Z�ÊLË�X[ÉGZ�Ì1ØL] ��Ó/Ì{Ë�X[\{Æ/Æ�]��N¾)Ô��~ËPÐ�Z�ÊLË)ÌqÍ�Ë6ÈsX[Z�Ì�Ë �,Ç�VÝÆ�Î/U�ÒfË�Ï/Ð/Ç�Ó�\8Â�Æ>Ê�ÓLÉ3Ô

�3CJ7 d 02352 + 9722 = 55225 + 944784 = 1000009 = 10002 + 32 �

��¢��

}60�2LH9H<.Q:)BGF/0+CJ89037>BDCAB{:(ab + cd)2 + (ad − bc)2 = (a2 + c2)(b2 + d2) �

}+CABGFa = 17 � b = 58 � c = 2 � d = 7 � BD;M;/I>B[2LCN7

10002 + 32 = (172 + 22)(582 + 72) = 293 · 3413 �

}6F/0L7 d 0 �2352 + 9722 = 293 · 3413

bÛ � b!��BD=n89037>BD4@CJ7M264 d F/;/;>.��3;�gq;<?äC d 0 d ?D0L2LK�CJ7O��?D;>=9H94�;>gu2�B!.J0L2L4GBWB�}6;ubs��;~B�}6;4GBD=n8/0L7>B[4�}+CA.N.W�3;�BE;/�30�BGF/0L?PKR;<?D0�BGF<2L7Ý;<7 d 0/b c gNBD0L?

k > 1��?D;>=9H/4)F<2��<0��3;<790 �0,�<0L?{:)B�}6;�4EBD=n89037>BD4sF<2��<06�3;<790PBE;/�30�BGF/0L?!0G L2 d BD. :M;<7 d 0/b@-9?D;L�<0PBGF<2�BWBGF/0+7<=9KRI/0L?;>gY4GBD=n8/0L7>B[4@CN7~BGF/0�4 d F/;/;>./CJ4@2�B!KR;<4EB

kb

= ÆL\�Ç/ÌJZ{Æ�Ï�Ñ�Øu¼/Z{Ë�XEXGË¡Æ�Æ�XEÏ/É ��Ì�Ë,Z1Ï<]�Ë Ó�ZJÉ�Æ�Ï/É�� �LÓ���Ì[Ì�Ë¡] ��X[Ó�Ï�×GË�bÜ�CN?[4GB!79;�BE0+BGF<2�BäBGF/06H<=nI<.ACJ45F/0�8O4EBD2�BE03KR037>B@;>g�BGF>CJ4sH/?D;/I<.J0LK�}�2L4�;/IL�<CJ;>=94�. :}�?D;<79� � 4�CN7 d 0�CABä2�45��0�8~gq;<?äH9?D;/;>g9BGF<23B � BGF/0+7<=9KRI/0L?�;>g�4GBD=n8/0L7>B[4@CN7~BGF/0+4 d F/;/;>.9CN4Ó/Ì�\�Ë�ÓLÉ�Ì

k��b � 0+H9?D;3�<0�BGF/0 d ;<?E?D0 d BE0L8)�<0L?[4GC1;<7�2�4@4GB[23BD0�8�2�I/;L�<0/b�YF/06��C �<0L7 d ;<798<CQBDC1;<7/4sF<2��<0�BGF/0+gq;>.A.1;�}+CN79� d ;<7/4,0L�<=n037 d 034 �

�12 � c 7�:MB�}6;���?D;>=9H/4�F<2��<0~23B@KR;<4EB�;<790)4GBD=n8/0L7>B@CJ7 d ;<KMKR;<7�b�� �!BGF/03?�}+CN4,0 � B�}6;4EBD=n89037>BD4f}6;>=/.18 I90 BE;/�30�BGF/0L?�CJ7 B�}6; 8>C��u0L?D037>B���?D;>=9H94 � d ;<7>BE?[2L?{: BE; BGF/0F�:<Hn;�BGF/034�CN4Lb ��1I � ��;j4GCJ79��.J0'��?D;>=9H d ;<7>BD23CJ7/4�23.N.sBGF/0�4GBD=n8/0L7>B[4Lb¯� �!BGF/03?�}+CN4,0 � }60 d ;>=/.J8j79;�BF<2��<0R2L79;�BGF/0L?���?D;>=9H�;>g!2�BP.J0L2L4GB�B�}6;�4GBD=n8/0L7>B[4 � 2L4�?D0��>=/CJ?D0L8 � }6F>C d F�}6;>=/.18d ;<7>BE?[2�8>C d B��J2 � b �� d � ß>2 d F�4GBD=n8/0L7>B+I/03.J;<79��4+BD;�2�BP.J0L2L4GB�B�}6;���?D;>=9H943b��{ãqg�45;<KR0M4GBD=n8/0L7>B�}603?D0OCN7;<7<. :T;<790���?D;>=9H � BGF/037�0�CABGF/03?�BGF<2�B+��?D;>=9H�}6;>=/.J8 d ;<7>B[2LCN7Ý23.N.�BGF/0O4GBD=n8/0L7>B[4 �d ;<7>BE?[2�8>C d BDCN79�M�1I �E� ;<?�03.N4,0)BGF/0M4GBD=n8/0L7>B@}6;>=/.18�79;�BPI/0~BE;/�30�BGF/0L?@}+CABGF�0,�<03?{:;�BGF/03?!4EBD=n89037>B�0� L2 d BD.Q:M;<7 d 0 � d ;<7>BE?[2�8<C d BDCJ79��BGF/06F�:<Hn;�BGF/034�CN4Lb ��L0�BSI/0OBGF/0R450�B+;>g�23.N.�BGF/0�4EBD=n89037>BD4 � 2L798T.J0�B n = |S| b�ãqg x ∈ S � }60890379;�BE06IL:

d(x)BGF/0�7<=9KOI903?�;>gu��?D;>=9H94 d ;<7>B[2LCN7<CJ79�

xb�L0�B

aI90)2~4EBD=n89037>B�b)©<:�� d � 2�I9;3�<0 � BGF/03?D0~2L?D0�B�}6;R8<CN4GBDCN7 d B���?D;>=9H/4 G

2L798G′ 4�= d FRBGF<23B a ∈ G

2L798a ∈ G′ bO�3CN7 d 0 |G| ≥ 2

2L798 |G′| ≥ 2 � BGF/03?D0R0G �CN4GB4GBD=n8/0L7>B[4b ∈ G

2�798c ∈ G′ 4G= d F6BGF<2�B b 6= a

2L798c 6= a

b��';<?D0L;L�<03? � IL:��12 �G� }60F<2��<0b /∈ G′ 2�798 c /∈ G

b��YF/03?D0)CJ4�2���?D;>=9HG′′ }6F>C d F d ;<7>B[2LCN7/4 b

2L798c�J4�CN7 d 0

b2�798

cKM=94EBsI90+BD;/�30�BGF/03?s;<7 d 0 � bs¦s4�CN79�6�12 �G� }60)F<2��<0 a /∈ G′′ b�ã1B�gq;>.A.1;�}�4�BGF<2�B

d(a) < kb� 06F<2��<0

(x,G)x∈G

1 =∑

x

G

x∈G

1

=∑

x

d(x)b

��¢�£

�@7�BGF/0�;�BGF/0L?!F<2�798 �∑

(x,G)x∈G

1 =∑

G

x

x∈G

1

=∑

G

|G| b�YF/03?D03gq;<?D0 �

x

d(x) =∑

G

|G| b � � ��L0�B

xI90O2O4GBD=n8/0L7>B � 2L798�.J0�B G

I90O2R��?D;>=9HT79;�B d ;<7>B[2LCN7<CJ79�xbfÜ<;<?P0L2 d F4GBD=n8/0L7>B

a ∈ G � BGF/0L?D0�0G �CN4GB[4�2M��?D;>=9H Ga}6F>C d F d ;<7>BD23CJ7/4

a2�798

xb~��;�BE06BGF<2�B

G 6= Ga � 4�CN7 d 0 x /∈ Gb!�';<?D0�;3�<0L? � CAg b ∈ G

2L798b 6= a � BGF/0���?D;>=9H94 Ga

2�798Gb2�?D0+8<CN4GBDCN7 d B!�J4�CN7 d 0

GCJ4äBGF/0P=97<C1�>=n0���?D;>=9H d ;<7>BD23CJ7<CN79�+I/;�BGF

a2�798

b� b!ã1BWgq;>.A.1;�}�4BGF<23Bd(x) ≥ |G| b � � ��P;�}�4�=9H/Hn;<450 � gq;<?W2 d ;<7>BE?[2�8>C d BDCJ;<7 � BGF<23B n > k

b��YF/037 � g1?D;<K�� � �G� gq;<?ä0L2 d FH923CJ?(x, G)

4�= d F)BGF<23Bx 6∈ G � }606F<2��<0

0 < k − d(x) ≤ k − |G| < n − |G| b�YF/037|G|

n − |G| <d(x)

k − d(x)

b � � ��3=9KOK�CJ79�)2L.A.YBGF/0)CJ790L�<=923.NCQBDC1034�� � � ;/I�BD23CJ790L8Rgq;<?s2L.A.YBGF/0~H923CJ?[4 (x, G)

4�= d FBGF<23Bx 6∈ G � }60��30�B

(x,G)x6∈G

|G|n − |G| <

(x,G)x6∈G

d(x)

k − d(x)

b � £ ��L0�B

GI/0@2���?D;>=9H�b���F/0@7<=9KRI/0L?u;>g�4GBD=n8/0L7>B[4�79;�BuI90�.1;<79��CN79�äBE;

GCJ4

n−|G| b�YF/03?D03gq;<?D0 �∑

(x,G)x6∈G

|G|n − |G| =

G

(

x

x6∈G

|G|n − |G|

)

=∑

G

(

n − |G|) |G|n − |G| =

G

|G| bc .J45; �

(x,G)x6∈G

d(x)

k − d(x)=

x

(

Gx6∈G

d(x)

k − d(x)

)

=∑

x

(

k − d(x)) d(x)

k − d(x)=∑

x

d(x)b

��¢�¢

ã1B!gq;>.A.1;�}�4äBGF<2�B�� £ � KO2�:OI/0�?D0¡}�?DCQB[BE037O2L4∑

G

|G| <∑

x

d(x) �}6F>C d F d ;<7>BE?[2�8<C d BD4s� � � b��YF/037 n ≤ k � 2L4 d .J23CJKO0�8ub�������'#���b ��F/0�CN790��>=92L.ACAB{:'CJ4)BGF/0TI9034GB)Hn;<4�4�CJI<.J0 � CJ7�BGF/0�gq;>.N.J;�}+CJ79�f450L7/4509b �L0�BS = {x1 � x2 � . . . � xn} b ãqg G1 = {x2 � . . . xn} � 2L798 Gi = {x1 � xi}

gq;<?i = 2 � . . . � n � BGF/0L7~BGF/0 d ;<798<CQBDC1;<7/4s;>gnBGF/0+H9?D;/I>.103K#2L?D0�452�BDCJ4G��0L8�2L798

k = nb

Û � b � 0 d F/;/;<4,02p−1

4G=nI/450�B[4!g1?D;<K�2�4,0�B�}+CQBGFp03.J0LKO0L7>B[4�4�= d F�BGF<23BW2�7�:6BGF<?D0L0F<2��<0�2 d ;<KMKR;<7M03.J0LKO0L7>B�b@-9?D;L�<0PBGF<23BWBGF/0,:�2L.A.�F<2��<0�2 d ;<KOKO;<7O03.J0LKO0L7>B�b

= ÆL\�Ç/ÌJZ{Æ�Ï�Ñ�Øu¼/Z{Ë�XEXGË¡Æ�Æ�XEÏ/É ��Ì�Ë,Z1Ï<]�Ë Ó�ZJÉ�Æ�Ï/É�� �LÓ���Ì[Ì�Ë¡] ��X[Ó�Ï�×GË�br:���¡���nb!�L0�B

n ≥ 1I90�2�7�CN7>BE0L�30L? � 2L798�.10�B S

I90�2�4,0�BY}+CQBGFn0�.103KR037>BD43b��L0�B

A1 �A2 � . . . � Ak

I/0P8>CJ4EBDCJ7 d B�4�=nI<4,0�BD4ä;>gS4G= d F�BGF<23B � gq;<?�2L.A. i 2�798 j � BGF/0sCJ7>BD0L?[450 d BDCJ;<7

Ai ∩ AjCJ4@79;�B�03KOH>B�:ub_��F/0L7

k ≤ 2n−1b

¼nX�Æ/Æ�Î!Æ�Î�\�Ë�ÒOÒOÓ9b~Ü>;<?i = 1 � . . . � k � .10�B Bi = S\Ai

b+��F/0L7B1 � B2 � . . . � Bk2�?D0R8<CN4GBDCN7 d B�4G=nI/450�B[46;>g

Sb�ãqg

Bi = Ajgq;<?P4,;<KO0

i2�798

j � BGF/037�}60M�9798�BGF<2�BAi∩Aj = Ai∩(S\Ai) = ∅ � }6F>C d F d ;<7>BE?[2�8<C d BD4uBGF/0sF�:<Hn;�BGF/034�CN4Lb_ã1BYgq;>.A.1;�}�4YBGF<2�BA1 � A2 � . . . � Ak � B1 � B2 � . . . � Bk

2L?D0)8<CN4GBDCN7 d B�4�=nI<4,0�BD4�;>gSb!��F>=94 � 2k ≤ 2n �BGF<23BäCN4 � k ≤ 2n−1

b� 0~79;�} H9?D;3�<06BGF/0�89034�CN?D0�8R?D0L4G=/.AB�IL:OCJ798>= d BDC1;<7Rgq;<?

p ≥ 3b+�{ãqg

p = 1;<?

p = 2 � BGF/0L7fBGF/0 d ;<798<CQBDC1;<7fBGF<2�B�2L7�:fBGF<?D0L0O4�=nI<4,0�BD4�F<2��<0O2 d ;<KMKR;<7T0�.103KR037>BCJ4��>2 d =n;>=94G.Q:�BE?[=n0 � 4GCJ7 d 0+}60 d 2L7 d F/;/;<4,0);<7<. : 2p−1 < 34G=nI/450�B[4Lb@��F/0)?D0L4G=/.AB�CN4g12L.N4,0�gq;<?_BGF/0L450 d 2�450L43b �ãqg

p = 3 � .J0�B S = {x1 � x2 � x3} b � 0 d F/;/;<450)gq;>=9?�450�B[4 A1 � A2 � A3 � A42�KO;<79� ∅ � {x1} � {x2} � {x3} � {x1 � x2} � {x1 � x3} � {x2 � x3} � {x1 � x2 � x3} �4�= d F~BGF<23BäBGF/0�CN7>BE03?[4,0 d BDC1;<7R;>gY2�7�:�BGF<?D0L0);>g�BGF/03K#CJ4s79;<79��0LKMH<B{:�b .1032�?[.Q: � 79;<790;>guBGF/06gq;>=9? d 2�7fI/0 ∅ b c .N4,; � BGF/0)450�B[4 {x1 � x2} � {x1 � x3} � {x2 � x3} d 2L7/79;�Bs2L.A.I90 d F/;<450L7 � I90 d 23=94,0OBGF/0�CJ?�CN7>BE03?[4,0 d BDC1;<7TCJ4)0LKMH<B{:�b ã1B+gq;>.A.1;�}�4+BGF<2�B�;<790f;>g!BGF/01� 03.J0LKO0L7>B�4,0�BD4WK�=94GB_I/0 d F/;<4,037 � 452�: {x1} b��YF/037~032 d F6;>g>BGF/0 d F/;<4,037)450�B[4WKM=94EBd ;<7>B[2LCN7

x1

b��YF>=94 � BGF/0 d ;<7 d .A=94�CJ;<7OF/;>.J8/4�gq;<? p = 3b�L0�B

p ≥ 4I90M2O�� �0�8fCJ7>BD0��303?�bR�3=9H9H9;<4,0�BGF<2�B�BGF/0O?D034�=/.QB�F/;>.18<4+gq;<?�2O4,0�B}+CABGF

p − 103.J0LKO0L7>B[4Lb@�L0�B

S = {x1 � x2 � . . . � xp} � 2�798~.10�B A1 � A2 � . . . � A2p−1I9068>CJ4EBDCJ7 d Bä4�=nI<4,0�BD4s;>gS4G= d F)BGF<23B

Ai ∩ Aj ∩ Ak 6= 0 � gq;<?ä2L.A.i < j < k

b�L0�B

npI/06BGF/0�7<=9KOI903?�;>g�4,0�BD4

AiBGF<23B d ;<7>B[2LCN7

xpb6ãqg

np < 2p−2 � BGF/0L7BGF/0�7<=9KOI903?_;>g>4,0�BD4Ai79;�B d ;<7>BD23CJ7<CN79�

xpCJ4

2p−1−np > 2p−1−2p−2 = 2p−2b�YF/034,0 4,0�BD4�gq;<?DK 2 d ;>.A.10 d BDC1;<7 ;>geKR;<?D0´BGF<2�7

2p−28<CN4GBDCN7 d B 4G=nI/450�B[4#;>g

{x1 � x2 � . . . � xp−1} 4�= d F�BGF<23B Ai ∩ Aj 6= ∅ gq;<?+2L.A. i2�798

j � d ;<7>BE?[2�8<C d BDCJ79�BGF/0+.103KOKM2nb���F>=94np ≥ 2p−2

b� CABGF�79;Ý.J;<454�;>g��303790L?[23.NCQB�: � }60�KM2�:�79;�}�4�=9H/Hn;<450�BGF<23Bxp

I/03.J;<79��4)BD;A1 � A2 � . . . � Anp

b

��¢��

pE�"6���$�bnp > 2p−2

b�L0�BA′

i = Ai \ {xp} gq;<? i = 1 � 2 � . . . � npb��YF/037

A′1 � A′

2 � . . . � A′np

gq;<?EK2 d ;>.A.10 d BDC1;<7);>g9KR;<?D0�BGF<2L72p−2

8<CN4GBDCN7 d B_4�=nI<4,0�BD4�;>g {x1 � x2 � . . . � xp−1} b_¦s4�CN79�BGF/0�.103KOKM2 � }60M890L8<= d 0�BGF<2�BsBGF/0L?D0O0G �CN4GB r2L798

s4G= d FRBGF<23B

A′r ∩ A′

s = ∅ bRã1Bgq;>.N.J;�}�4WBGF<23BAr ∩ As = {xp} b!Ü>;<?ä032 d F k 6∈ {r � s} � 4�CN7 d 0 Ar ∩ As ∩ Ak 6= ∅ �}60�F<2��<0

Ar ∩ As ∩ Ak = {xp} � g1?D;<K }6F>C d F6}60�890L8<= d 0�BGF<2�B xpCJ4@2 d ;<KOKO;<703.J0LKO0L7>B�;>gY23.N./BGF/0

Ai� 4Lb

pE�"6�� �9bnp = 2p−2

b�L0�BkI/0�4�= d FsBGF<23B

xp 6∈ Ak2�798�8/03�9790

Bk = Ak∪{xp} bu�YF/037 Bk 6= Akb�3=9H9H9;<4,0�BGF<23B

Bk 6= Ajgq;<?!2L.A.

jbW�YF/037�BGF/0�4,0�BD4

A1 � A2 � . . . � A2p−1 � Bkgq;<?EK2 d ;>.N.J0 d BDCJ;<7);>g

2p−1 +18<CN4GBDCN7 d B_4G=nI/450�B[4!;>g

S � 2�798+BGF/0�CN7>BE03?[4,0 d BDC1;<7~;>g/2�7�:6H923CJ?;>gu8<CN4GBDCN7 d B!450�B[4�CJ7~BGF>CN4 d ;>.N.J0 d BDCJ;<7~CJ4@79;<79��0LKMH<B{: � d ;<7>BE?[2�8<C d BDCJ79��BGF/0+.J0LKMKO29b�YF/03?D03gq;<?D0 � gq;<?O032 d F k4�= d FáBGF<23B

xp 6∈ Ak � BGF/0L?D0 0G �CN4GB[4 j4G= d FâBGF<2�B

Ak ∪ {xp} = Ajb��3CN7 d 06BGF/0)7<=9KRI/0L?�;>g

Ai� 4s}6F>C d F�89;R79;�B d ;<7>BD23CJ7

xpCN4�BGF/0452LKR0+2L4!BGF/0+7<=9KRI/0L?�;>g

Ai� 4!}6F>C d F d ;<7>BD23CJ7

xp � }60�890L8<= d 0�BGF<2�BWBGF/0 d ;>.A.10 d BDC1;<7;>gnBGF/0Ai� 4�CJ4

A1 � A2 � . . . � Anp� A1 ∪ {xp} � A2 ∪ {xp} � . . . � Anp

∪ {xp} �}6F/0L?D0xp 6∈ Ai

gq;<?i = 1 � 2 � . . . � np

b�P;�}A1 � A2 � . . . � Anp

2L?D02p−2

8>CJ4EBDCJ7 d B!4G=nI/450�B[4�;>g {x1, x2, . . . , xp−1}4�= d FMBGF<23B � gq;<?s23.N. i < j < k � }60~F<2��<0 Ai ∩ Aj ∩ Ak 6= ∅ b~©<:MBGF/06CJ798>= d BDC1;<7F�:/H9;�BGF/0L4GCJ4 � BGF/0,:64,F<2L?D0P2 d ;<KMKR;<7)03.J0LKO0L7>B � 452�: ab .J0L2L?D. : � a ∈ Ai ∪{xp} gq;<?0L2 d F

i � 2L798MBGF/0L?D0�gq;<?D0 � a CN4�2 d ;<KOKO;<7�03.J0LKO0L7>Bs;>g�23.N.nBGF/0 Ai� 43b@�YF>CN4P03798/4sBGF/0CJ798>= d BDC1;<7M4GBD0LHM2�798)BGF/0�H9?D;/;>gDb

Û � b@�L0�Ba � b 2L798 c

I/0�?D032L.n7<=9KRI/0L?[4!}+CQBGF�4�=9K � b@-9?D;3�<0�BGF<2�Ba7 + b7 + c7

7=

(

a5 + b5 + c5

5

)(

a2 + b2 + c2

2

) b= ÆL\{ÊLË�Õ'Ñ�Ø Ë�Z{×DÍYË,\�Æ�Ó/ÌJÓ3ZA\N\{Ë�]§Å_ÆnÇ>Ë�Ï<] ��X[Ó�Ï�×GË � Â/Í/X[ZJÉ�Ì�Æ3VYÍ�Ë�X+¿LÔ�Æ�X[Ó<Õ�\{Ë�ØL]§Æ�X[ZJÉ�Ì�ÆL\N]��Ä � ¼/Z{Ë�XEXGË Æ�Æ�XEÏ/É ��Ì�Ë,Z1Ï<]dË Ó�ZJÉ�Æ�Ï/É�� �LÓ���Ì[Ì�Ë¡] ��X[Ó�Ï�×GË � Ó�Ï�Õ��nËLÕ�Ç3\JÓ ¾)Ô�ËeÇ�X,Ì1ØL]�PÆ>ÊLË�XD]�¼3U!]�� = UPÔ��~ËPÐ�Z�ÊLË¡Æ�X[Ó<Õ�\{Ë�Ø�É ÉsÉ�ÆL\[Ç<ÌJZ�Æ�Ïub

�L0�Ba � b � c I/0�BGF/0+?D;/;�B[4@;>g x3 + qx − r = 0 � 2L798~.J0�B Sk = ak + bk + ck b�YF/037

S1 = a + b + c = 02�798

S2 = a2 + b2 + c2

= (a + b + c)2 − 2(ab + bc + ca) = −2q�P;�} }60�F<2��<0S3 + qS1 − 3r = 0 ==⇒ S3 = 3r

S4 + qS2 − rS1 = 0 ==⇒ S4 = 2q2

S5 + qS3 − rS2 = 0 ==⇒ S5 = −3qr − 2qr = −5qr

S7 + qS5 − rS4 = 0 ==⇒ S7 = 5q2r + 2q2r = 7q2rb

�YF/037 S7

7= q2r =

S5

5· S2

2

b

��¢ �Þ Û bç��0L��KR037>BD4

AC2L798

BDCJ7>BD0L?[450 d B~2�B~Hn;>CN7>B

Eb�-<;>CJ7>B[4

K2�798

M � ;<74,0L��KR037>BD4AB

2�798CD � ?D0345H90 d BDC �<03. : � 2L?D0�4�= d F)BGF<23BWBGF/0�4,0L��KR037>B KM

H/2�4�4,034BGF<?D;>=n�3FEb@-9?D;L�<0PBGF<23B

KM ≤ max{AC � BD} b= ÆL\{ÊLË�Õ�Ñ�Ø�Â/Í/X[ZJÉ�Ì�Æ3VYÍ�Ë�X�¿LÔ4Æ�XDÓ>Õ�\�Ë�Ø3]�Æ�X[ZJÉ�Ì�ÆL\N]���Ä � Ó�Ï�Õ �3Æ�É,Í>Z�Æ = Ë¡ZJÒ�ZqØ3Ó3]�Ä�Ó<ÈsÓ3É,ÓF0�ZJ]¿,Ó�VYÓLÏ�Ô �~ËPÐ�Z�ÊLË = Ë,Z1Ò�Z1Ø3Ó�É ÉsÉ�ÆL\[Ç<ÌJZ�Æ�Ïub

�YF/0+gq;>.A.1;�}+CN79��.103KOKM2�CN4ä}603.A.���79;�}�7 � 2L798�CAB[4@H9?D;/;>g9}+CA.N.979;�B�I90���CQ�<037�br:���¡���nb ãqgP

CN4 2 H9;>CJ7>B ;<7 BGF/0´4�CJ890BC

;>g BE?DCN2�79��.J0ABC � BGF/0L7

AP ≤ max{AB, AC} bpE�"6���$�bAB ‖ CD

b_�q��0L0�BGF/068>CJ2���?[2�KÙ;<7)BGF/0+.10�gNB�I/03.J;�}Ob ��L0�BX � Y

I90�H9;>CJ7>B[4~;<7CD

4G= d F�BGF<23BKX ‖ AC

2�798KY ‖ BD

b'�3CJ7 d 0AKXC

2L798BKY D

2�?D0�I9;�BGF�H/2�?[23.N.J03.J;/��?[2�KM4 � }606F<2��<0KX = AC

2L798KY = BD

b�3CJ7 d 0

MCJ4@2�Hn;>CN7>B�;<7~BGF/0�450���KO0L7>B

XY � }60�F<2��<0 � IL:)BGF/0+.J0LKMKO2 �KM ≤ max{KX, KY } b�YF/03?D03gq;<?D0 �KM ≤ max{AC, BD} b

.

..

.

..

..

..

.

..

..

..

.

..

..

..

..

.

..

..

..

.

..

..

..

.

..

..

..

..

.

..

..

..

.

..

..

..

.

..

..

..

..

.

..

..

..

.

..

..

..

.

..

..

..

..

.

..

..

..

.

..

..

..

.

..

..

..

..

.

..

..

..

.

..

..

..

.

..

..

..

.

..

..

..

..

.

..

..

..

.

..

..

..

.

..

..

..

..

.

..

..

..

.

......................................................................................................................................................................................................................................................................................................................................................................................................................................................................................................................................................................................................................................................................................................................................................................................................................................................................

.............................................................................................................................................................................................................................................................................................................................................................................................................................................................................

..

..

.

..

..

..

..

..

.

..

..

..

..

..

.

..

..

..

..

..

.

..

..

..

..

.

..

..

..

..

..

.

..

..

..

..

..

.

..

..

..

..

..

.

..

..

..

..

..

.

..

..

..

..

..

.

..

..

..

..

.

..

..

..

..

..

.

..

..

..

..

..

.

..

..

..

..

..

.

..

..

..

..

..

.

..

..

..

..

..

.

..

..

..

..

.

..

..

..

..

..

.

..

..

..

..

..

.

..

..

..

..

..

.

..

..

..

..

..

.

..

..

..

..

..

.

..

..

..

..

.

..

..

..

..

..

.

..

..

..

..

..

.

..

..

..

..

..

.

..

..

..

..

..

.

..

..

..

..

..

.

..

..

..

..

.

..

..

..

..

..

.

..

..

..

..

..

.

..

..

..

..

..

.

..

..

..

..

..

.

..

..

..

..

..

.

..

..

..

..

.

..

..

..

..

..

.

..

..

..

..

..

.

..

..

..

..

..

.

..

..

..

..

..

.

..

..

..

..

..

.

..

..

..

..

.

..

..

..

..

..

.

..

..

..

........................................................................................................................................................................................................................................................................................................................................................................................

......................................................................................................................................................................................................................................................

A

BC

D

EK

M

X

Y

.

..

..

..

..

..

..

..

..

..

..

..

..

...

..

..

..

..

..

..

..

..

..

..

..

..

..

...

..

..

..

..

..

..

..

..

..

..

..

..

..

..

...

..

..

..

..

..

..

..

..

..

..

..

..

..

..

...

..

..

..

..

..

..

..

..

..

..

..

..

..

..

...

..

..

..

..

..

..

..

..

..

..

..

..

..

..

...

..

..

..

..

..

..

..

..

..

..

..

..

..

..

...

..

..

..

..........................................................................................................................................................................................................................................................................................................................................................................................................................................................................................................................................................................................................................................................................................................................................................................................................................................................................................................................................................................................................................................................................................................................................................................................

..

..

..

..

..

..

..

..

...

..

..

..

..

..

..

..

..

..

..

..

..

..

..

...

..

..

..

..

..

..

..

..

..

..

..

..

..

..

...

..

..

..

..

..

..

..

..

..

...

..

..

..

..

..

..

..

..

..

..

..

..

..

..

..

..

..

..

..

..

..

..

..

..

..

..

..

..

..

..

..

..

..

..

..

..

..

..

..

..

..

..

..

..

..

..

..

..

..

..

..

..

..

..

..

..

..

..

..

..

..

..

..

..

..

..

..

..

..

..

..

..

..

..

..

..

..

..

..

..

..

..

..

..

..

..

..

..

..

..

..

..

..

..

..

..

..

..

..

..

..

..

..

..

..

..

..

..

..

..

..

..

..

..

..

..

..

..

..

..

..

..

..

..

..

..

..

..

..

..

..

..

..

..

..

..

..

..

..

..

..

..

..

..

..

..

..

..

..

..

..

..

...

.

..

..

.

..

..

..

..

..

..

..

..

..

..

..

..

..

..

..

.

..

..

..

..

..

..

..

..

..

..

..

..

..

..

..

.

..

..

..

..

..

..

..

..

..

..

..

..

..

..

..

.

..

..

..

..

..

........................................................................................................................................................................................................................................................................................................................................................................................................................................................................................................................................................................................................................................................................................................................................................................................................................................................................................................................................................................................................................................................................................................................................................................................................

.......................................................................................................................................................................................................................................

.

.

..

.

..

.

..

..

..

..

.

..

.

..

.

.

..

..

.

..

.

.

..

.

..

..

.

..

.

.

..

.

A

BC

D

E

K

M

P

Q

S

T

X

Y

pE�"6�� �9bAB ∦ CD

bW�q��0L0�BGF/068>CJ2���?[2�K#;<7~BGF/0�?[C1�3F�B!2�I/;L�<0/b �ã{7�BGF>CN4 d 2L4,0 � }6F/037f}60 d ;<7/4GC18/0L?��<=92�8/?[CN.N23BD0L?[23. ABCD � }60RF<2��<0�03CQBGF/0L?∠A + ∠D > 180◦ ;<?

∠B + ∠C > 180◦ b � 0MKO2�:�2L454G=9KR0)}+CQBGF/;>=<B�.J;<454+;>g�30L7903?[2L.ACAB{:)BGF<23B∠A + ∠D > 180◦ b��YF/0�.ACJ790�BGF<?D;>=n�3F D

H/2�?[23.N.J03.<BE;AB

KO0�0�BD4EC

2�798EM

2�BP2�798

Q � ?D0345H90 d BDC �<03. :�b��F/0L7 P2L798

Q2�?D0�H9;>CJ7>B[4~;<7TBGF/04,0L��KR037>BD4

EC2L798

EM � ?D0345H90 d BDC �<03. :�b�©<:'�'037903.N2L=94����YF/0L;<?D0LK�gq;<? 4DPC �}606F<2��<0P Q

QD· DM

MC· CE

EP= 1

b

��¢ �

�YF/03?D03gq;<?D0 �P Q

QD· DM

MC=

EP

CE< 1 �

BGF<23BäCN4 � CM

MD>

P Q

QD

bW�3CN7 d 0DP ‖ AB � }60��30�B P Q

QD=

AK

KB

b_��F>=94 �CM

MD>

AK

KB

b � � ��L0�B

XI/0R2RHn;>CN7>B�4�= d FfBGF<23B

CX ‖ AK2�798

KX ‖ ACb��L0�B

YI/0R2Hn;>CN7>Bä4�= d F6BGF<2�B

DY ‖ BK2L798

KY ‖ BDb��L0�B

SI90�BGF/0�CN7>BE03?[4,0 d BDC1;<7�;>g

XY}+CABGFCD

b�YF/037CX = AK � KX = AC � Y D = KB � 2�798 KY = BD

bO�3CN7 d 0CX ‖ AB ‖ Y D � }60�F<2��<0

CS

SD=

CX

Y D=

AK

KB

b ;<7/450��>=n0L7>BD. : � =94GCJ79�6� � �G� }60)F<2��<0 CS

SD<

CM

MD

b+�+0L7 d 0 � M CJ4�2~Hn;>CN7>B�;<7OBGF/04,0L��KR037>BDS

b�L0�BTI90�BGF/06CN7>BE03?[4,0 d BDC1;<7�;>g

KM}+CQBGF

SYbs�YF/037

MCN4�2~Hn;>CN7>Bs;<7OBGF/04,0L��KR037>B

KTb6��F>=94 � KM ≤ KT

b)�3CJ7 d 0TCN4�2MH9;>CJ7>B�;<7�BGF/0�4,0L��KR037>B

XY �}606F<2��<0 � IL:~BGF/0�.103KOKM2 �KT ≤ max{KX, KY } = max{AC, BD} b

�YF/03?D03gq;<?D0 � KM ≤ max{AC, BD} bÞPÞ b@-9?D;L�<0PBGF<23Bn∑

k=0

(

n

k

)

(a + k)k−1(b + n − k)n−k−1 = (a + b + n)n−1(

1

a+

1

b

) b

= ÆL\�Ç/ÌJZ{Æ�Ï�Ñ�Ø�ËÝZ�×DÍ�Ë¡\�Æ�Ó<Ì1Ó�ZN\A\�Ë¡]8Å�ÆnÇ<Ë�Ï/] ��X[Ó�Ï�×GË�b�L0�B

S8/0L79;�BD06BGF/06.10�gNBs4GC18/0~;>guBGF/0)2�I9;3�<0�0L�<=92�BDC1;<7 � 2�798O.10�B N = n − 1

b¦s4�CN79� (nk

)

=(N

k

)

+( Nk−1

) � }60�}�?DCQBE0 S = S1 + S2 � }6F/0L?D0

S1 =

N∑

k=0

(

N

k

)

(a + k)k−1(b + N + 1 − k)N−k

2L798S2 =

N+1∑

k=1

(

N

k − 1

)

(a + k)k−1(b + N + 1 − k)N−k b

��¢"

� ;<?G�LCJ79��}+CQBGFS1

�n?[4EB � }60+79;�BE0PBGF<23B � gq;<?!0L2 d F k = 0 � 1 � . . . � N �(

N

k

)

(a + k)k−1(b + N + 1 − k)N−k

=

(

N

k

)

(a + k)k−1(

a + b + N + 1 − (a + k))N−k

=

(

N

k

)

(a + k)k−1

N−k∑

j=0

(

N − k

j

)

(a + b + N + 1)j(

−(a + k))N−k−j

=

N−k∑

j=0

(

N

k

)(

N − k

j

)

(−1)N−k−j(a + b + N + 1)j(a + k)N−1−j

=

N−k∑

j=0

(

N

j

)(

N − j

k

)

(−1)N−k−j(a + b + N + 1)j(a + k)N−1−j

�3=9KOK�CJ79��;3�<0L?k � 2L798�CJ7>BD0L? d F<2L79��CJ79�PBGF/0�;<?D8903?�;>gY4G=9KOKM23BDCJ;<7 � }60��30�B

S1 =N∑

j=0

(

N

j

)

(a + b + N + 1)j

N−j∑

k=0

(

N − j

k

)

(−1)N−k−j(a + k)N−1−j b�P;�}j}60�KM2���0�=9450�;>g>BGF/0�gq;>.N.J;�}+CJ79���303790L?[23./?D0L4G=/.AB � gq;<?W0,�<03?{:)Hn;>. :/79;<K�CJ23.

P}+CQBGFM8/0���?D0L0

< m �m∑

k=0

(−1)m−k

(

m

k

)

P (k) = 0b

�J��F/0�4G=9K;<7ÝBGF/0�.J03gNB�4�CJ890RCJ4∆mP (0) � }6F/03?D0 ∆

CJ4�BGF/0�8<C���03?D0L7 d 0�;<H90L?[2�BE;<?890��n790L8MIL:∆P (x) = P (x + 1) − P (x)

b � ã1B_gq;>.N.J;�}�4WBGF<2�B�BGF/0PCJ7/7903?W4�=9K CJ76BGF/02�I/;L�<060G �H/?D0L4�4�CJ;<7�gq;<?S1

CJ400�  d 0LH>BW}6F/037

j = Nb��YF>=94 �

S1 =

(

N

N

)

(a + b + N + 1)N

(

00

)

(−1)0(a + 0)−1 =1

a(a + b + n)n−1 b

Ü<;<?S2 � }60�F<2��<0

S2 =

N+1∑

k=1

(

N

N − k + 1

)

(a + k)k−1(b + N + 1 − k)N−k

=N∑

j=0

(

N

j

)

(a + N + 1 − j)N−j(b + j)j−1

c H9H<.Q:>CJ79��BGF/0�452LKR0�2�?D��=9KO0L7>B@2�4@gq;<?S1 � }606�30�B S2 =

1

b(a + b + n)n−1

b!��F/089034�CN?D0�8M?D034�=/.QBägq;>.A.1;�}�43b

� ���

Â�Æ�ÒOÒfË�ÏYÌWÑ�Øu¼/Z{Ë�XEXGË¡Æ�Æ�XEÏ<É ��Ì{Ë¡ZJÏ/]�Ë Ó�ZJÉ�Æ�Ï/É�� �LÓ���Ì[Ì�Ë¡] ��XDÓLÏY×EË>b�YF>CN4sH/?D;/I<.J0LK�CN4sH/?D;<Hn;<450�8M2L4�0G �0L? d CN4,0 £�£ �5IO�Q}+CABGF�4,;>.A=<BDC1;<7 � CJ7��/bu�L;3��@2L4G� �Â�Æ�Ò�ÑLZ1Ï/Ó/Ì{Æ�X[Z1Ó�\�¼nXGÆ9ÑL\�Ë�ÒMÉ�ÓLÏYÕ�� �<Ë�XG×�ZJÉ�Ë�É � ��;<?[BGF/�,��;>.A.J2L798 � �" � bÞPà b��YF/0�H<.J2L790+CJ4s8>CQ�>C18/0�8�CN7>BE;�?D0L��C1;<7/4sIL:

n.NCN790L4�CN7R�303790L?[23.�H9;<4�CQBDC1;<7/43b@-9?D;L�<0BGF<23B!2�Bä.1032�4EB

n − 2;>gnBGF/0�?D0L��C1;<7/4@2L?D0�BE?[CJ2L79��.1034Lb

= ÆL\�Ç/ÌJZ{Æ�Ï�Ñ�Øu¼/Z{Ë�XEXGË¡Æ�Æ�XEÏ/É ��Ì�Ë,Z1Ï<]�Ë Ó�ZJÉ�Æ�Ï/É�� �LÓ���Ì[Ì�Ë¡] ��X[Ó�Ï�×GË�bÜ�CN?[4GB�79;�BD0~BGF<23B � CAg�BGF/0�.NCN790 `

KO0�0�BD4PBGF/0�CN7>BE03?DCJ;<?P;>g�BGF/0M79;<79�58/0��303790L?[2�BE0BE?DCN2�79��.J0ABC � BGF/037 `

K�=94GB d ?D;<4�4�B�}6;f4GC18/0L4+;>g�BGF/0~BE?[CJ2L79��.10 � 452�: [AB]2�B

M2�798[AC]

2�BNb!ã1BWgq;>.N.J;�}�4äBGF<2�B

AMNCJ4�2+79;<79�58/0��303790L?[2�BE0PBE?[CJ2L79��.10/b��YF>=94 � CAg2�.ACJ790�KO0�0�BD4�BGF/0sCJ7>BD0L?[C1;<?�;>g/2sBE?[CJ2L79��.10 � CQB_8>CQ�>C18/0L4�BGF/0@BE?DCN2�79��.J0�CN7>BE;PB�}6;�?D0L��C1;<7/4 �23Bä.J0L2L4GB!;<7906;>g/}6F>C d F~CN4@2+BE?[CJ2L79��.10/b�L0�B

n ≥ 3I90O2L7�CJ7>BD0��303?�b��L0�BsBGF/0OH/.N2�790OI90O8<C �<CJ890L8�CJ7>BD;�?D0L��C1;<7/4�IL:

n.NCN790L4`1 � `2 � . . . � `n

CN7f�303790L?[23.�H9;<4�CQBDC1;<7�b � CABGF/;>=<B�.1;<4�4�;>g_�303790L?[23.NCQB�: � }60)KM2�:4�=9H/Hn;<450+BGF<2�B � gq;<? i = 1 � 2 � . . . � n − 1 � BGF/0+.NCN790 `iKO0�0�BD4

`n23B

Mi � 4G= d F�BGF<2�BBGF/0MHn;>CN7>BD4M1 � M2 � . . . � Mn−1

2L?D0MH923CJ?�}+CJ450M8<CN4GBDCN7 d BP�14GCJ7 d 0~BGF/0�.ACJ79034�2�?D0�CN7�30L7903?[2L.�H9;<4�CQBDC1;<7 � 2L798�CJ7~BGF>CN4s;<?D8903?�;<7 `nbÜ<;<?

i ≤ n − 2 � BGF/0�.NCN790L4 `i � `i+1 � 2�798 `ngq;<?DKÙ2�BE?DCN2�79��.J0

Ti � }6F/0L?D0 Mi2�798Mi+1

2L?D06B�}6;~�<0L?�BDC d 0L4P;>gTib)�';<?D0L;L�<03? � g1?D;<K#BGF/0�;<?D8/0L?�;>gYBGF/0

Mi� 4P;<7BGF/0�.ACJ790

`n � 2�7�:)B�}6;~;>g9BGF/0�BE?DCN2�79��.J0L4 T1 � T2 � . . . � Tn−2

F<2��<0+79;6CJ7>BD0L?[C1;<?WHn;>CN7>BCJ7 d ;<KOKO;<7�b_��F>=94 � }60�F<2��<0�0� L2 d BD.Q: n − 2BE?[CJ2L79��.1034Lb�L0�B

k ≤ n − 2I/0��> �0L8�b!ãqg�79;<790�;>g9BGF/0P.NCN790L4

`iKO0�0�BD4äBGF/0�CN7>BE03?DCJ;<?!;>g

Tk �BGF/0L7TkCJ4�;<790~;>gYBGF/06?D0���CJ;<7/4Lb �!BGF/03?�}+CN4,0 � 26.NCN790 `i

KR0L0�B[4sBGF/06CN7>BE03?DCJ;<?s;>gTkb�L0�B

i1I90PBGF/0�.J0L2L4GBWCN7>BE0L�30L?ä4G= d F)BGF<23B

`i1

KO0�0�BD4äBGF/0+CN7>BE03?DCJ;<?!;>gTkb��YF/037 � g1?D;<KBGF/0�CN7<CABDCN2L.ä?D03KO2L?G��2�I/;L�<0 � BGF/0R.ACJ790 `i1

8<C �<CJ89034Tk

CN7>BE;�B�}6;T?D0���CJ;<7/4 � 2�B+.J0L2L4GB;<790);>gn}6F>C d F�CJ4s2+BE?DCN2�79��.J0 � 4�2�: Tk1

b���;�BE0�BGF<23B�79;<790);>gnBGF/0�.NCN790L4`iKR0L0�B[4@BGF/0CJ7>BD0L?[C1;<?!;>g

Tk1

gq;<?i ≤ i1

bãqg � gq;<? i > i1 � 79;<7906;>g9BGF/0+.ACJ79034 `iKR0L0�B[4!BGF/0�CJ7>BD0L?[C1;<?!;>g

Tk1� BGF/037 Tk1

CN4;<790);>g�BGF/06?D0���CJ;<7/4Lbsã{7MBGF/0);�BGF/03? d 2L4,0 � 4�=nI<4GBDCQBD=<BDCJ79� Tk1

gq;<?Tk � }60+gq;>.N.J;�} BGF/0452LKR0�?D032�45;<7<CJ79��2�4!2�I/;L�<0/bW�3CJ7 d 0 � 2�Bä0L2 d F~4EBE03H � BGF/0�7<=9KOI903?!;>g9.NCN790L4!?D0LKM2LCN7<CJ79�BE;RI90 d ;<7/4�CJ8903?D0�8OCN4P8/0 d ?D032�4GCJ79� � BGF>CN4�H/?D; d 03454@}+CA.N.u0,�<037>BD=92L.A.Q:R4EBE;<H � ��C �<CN79��=94�2?D0���CJ;<7~}6F>C d F~CJ4@2�BE?DCN2�79��.J09b� CABGFRBGF>CJ4�H9?D; d 0L4�4 � 032 d F�;>g�BGF/0 Ti

� 4�.1032�8<4PBD;�2�BE?[CJ2L79��=/.J2L?�?D0���CJ;<7�bO�3CJ7 d 02�7�:OB�}6;f;>guBGF/0�?D0L��C1;<7/4TiF<2��<0�79;RCN7>BE03?DCJ;<?sHn;>CN7>BsCN7 d ;<KMKR;<7 � BGF/0)BE?[CJ2L79��=/.J2L??D0���CJ;<7/4s2L?D068>CJ4EBDCJ7 d B � 2L798)}60+2�?D0�89;<790/b

�YF<2�B d ;<KOH<.10�BE034@BGF/0}Â�Æ�XEÏYË�X!gq;<?äBGF>CN4sCN454G=n09b@��0L798OKR0�:<;>=9?�7<C d 0645;>.N=<BDCJ;<7/42�7986�303790L?[23.NCQ�G23BDCJ;<7/4Lb �ä�<03?YBGF/0�790G LB�4,0,�<03?[2L.�CJ4�4�=n034 � }60�}+CA.N.>I/0�KM2��LCJ79�@2�760 ��;<?�BBE;�4,F/;<?�BE037�BGF/0@BDCNKR0�I/0�B�}60L0L7PBGF/0�2LH9H90L2L?[2�7 d 0�;>g<2�H/?D;/I<.J0LK 4,0�B�2L798�H<=nI<.AC d 23BDCJ;<7;>gnBGF/0�45;>.N=<BDCJ;<7/4Lb

� ���� � V \2^ b]`6��W}^��

V �º`+`NWYX ����� e'W2^��1������ ��������������

�â"~( ����* ��Ý=9?E?[2�:���0,:/KO;>=9? T .N2�KO��CN7�}�2�4_I/;<?E7�;<7~��2�? d F ¢ � �" >��� CJ7�©n?D;/;/�L.Q:<7 � �P0¡}Ã�;<?G�nb!�+0�F<2�8�2+KR;<4EBäH/?D;/8<= d BDC �<0�2L798~gJ=/.N�/.N.ACJ79��.ACNgq0 � 8<C �<CJ890L8MI90�B�}60�037�CN798<=94EBE?{:2�798�2 d 2�8/0LK�CJ29b c .QBGF/;>=n�3F+F/0ä}�2L4_F>CJ�3F>.Q:�4G= dGd 0L4�4�gJ=/.3CJ7+0,�<0L?{:�BGF>CJ79�@F/0@2�B[BE03KOH>BE0L8 �F/0�}+CA.N._H/?D;/I/2�I<. :ÝI/0O?D0LKO0LKOI903?D0�8TKO;<4GB�gq;<?�F>CN4+CJ7��<;>. �<0LKO0L7>B�CJ7�KO2�BGF/0LKM23BDC d 4H9?D;/I>.103KR�G4,;>. �<CN79��2�798 d ;<KMHn0�BDCABDCJ;<7/4Lb��0�23=<BGF/;<?D0�8�;<?�0�8>CABD0�8�gq;>=9?�H9?D;/I>.103KÂI/;/;/��4P2�798�.10�gNBPF>CN4�KM2�?E�OCN7�0,�<03?{:KO2 ;{;<? ;{;>=9?E7/23.�}6F>C d F�F<2�8�2@H9?D;/I>.103K 4,0 d BDC1;<7�b_��0W}�2�4YBGF/0s0L8<CQBE;<?u;>gLBGF/0�H9?D;/I>.103K4,0 d BDC1;<7�;>gä�>ã c � ¤Y0,�>C10¡}�gq;<?�2�.J;<79�6BDCNKR0 � 2�798O}�2�4�}60�.N.���79;�}�7�BD;f?D032�8/0L?[4+;>gBGF>CJ42;{;>=9?E7/23.�2L4�BGF/0�CJ7/23=n��=9?[2L.Y0L8<CQBE;<?!;>gu;>=9? ��.Q:<KOH<CJ2�8 ;<?E7903?�b�Ý=9?E?[2�:O}�2L4�F/032��>CN. :RCN7��<;>.Q�<0L8RCJ7OBGF/0�¦�� c ��2�BGF/0LKM23BDC d 23. ��. :/KMH/CN2�8�2L798BGF/0�¦�� c ��2�BDC1;<7/23.Y�9032�KÙCJ7OBGF/0Oã{7>BD0L?D7/23BDCJ;<7/2L.W��23BGF/03KO2�BDC d 2L.���.Q:<KOH<CJ2�8 � 4,03?{�9�CJ79�O2L4)8/0LH<=<B�:T.J0L2�8903?Pg1?D;<K �� � ¢ BE; �� � � 2�798T2L4�.1032�8/0L?Pg1?D;<K �" ��� BD; �" ��£ bc gNBE03?�KR;3�<CN79�6BD; 2�7/2�8/2 � F/0�KM2�8/0�4�CJ��7<CN� d 2L7>B d ;<7>BE?DCJI<=<BDCJ;<7/4�BE;OBGF/0 2�7/2�8<CN2�7��23BGF/03KO2�BDC d 2L. ��.Q:<KOH<CJ2�8M2L4ä}603.A.n2�4@H/?D;L�>CJ7 d CN2L.nKO2�BGF/0LKM23BDC d 4 d ;<KMHn0�BDCABDCJ;<7/4Lb��0�}�2L4W2L?D��=92�I>.Q:�BGF/0�}6;<?D.J8Y� 4äI9034GBG����79;�}�7�KO2�BGF/0LKM23BDC d 4_H9?D;/I>.103KR�G4,;>. �<0L? �F>CJ4äCNKOKO0L7/450�BD23.1037>BW45F>CJ7<CN79�sBGF<?D;>=n�3F�F>CN4äCJ7 d CN4�C �<0�CN7/4�CJ�3F�B � F>CJ4 d .ACJ7<C d 23.90�� d CJ0L7 d : �2�798�F>CN4�HnF/0379;<KR037/2L._KO0LKO;<?{:�bO�PCJ4�?D034,032�? d F�CJ7>BD0L?D034GB�?[2�79�30L8T;3�<0L?@BGF/06}6F/;>.1045H90 d BE?[=9Kç;>g>KO2�BGF/0LKM23BDC d 4 � g1?D;<K c H9H<.NCJ0�8~��2�BGF/0LKM23BDC d 4uBE;+�LH9F/0L?[C d 2L.>¨W0�;<KO0�BE?{:ub��;�}60,�<03? � }60MKO2�:�4GCJ79��.J0R;>=<B�F>CN460G �H90L?�BDCJ450MCN7�CN790��>=92L.ACABDCJ0L4�CN7Ý�303790L?[23.W2�798�CN7BGF/0+�9?DCN2�79��.J0~ã{790L�<=923.NCQB�:�CN7MH92L?[BDC d =/.J2L?�b�Ý=9?E?[2�:�037�;{;L:<0L8+?D0LKM2�?E�L2�I>.Q:+�3;/;/8+F/032L.QBGF+8>=9?DCN79�sF>CN4�.J;<79�!.ACNgq0/b �@7 c =n��=94EB

� � �����3£ � 2 d ;<KOI<CN7/23BDCJ;<7M;>gnCJ7>BD0L4EBDCJ7/23./BD=9KO;>=9?ä2L798MF/0L2L?[BWg12LCA.N=9?D0�BE;/;/�6F>CNK�2�}�2�:g1?D;<K�=943bR�PCJ4�H/2�4�4�CN79�~KM2�?E��4�BGF/0O03798�;>gW2L7�0L?[2�CJ7�BGF/06}6;<?D.J8�;>g_KO2�BGF/0LKM23BDC d 4d ;<KMHn0�BDCABDCJ;<7/4@2�798MH/?D;/I<.J0LKO��45;>.Q�>CJ79�<b!�+0P}+CA.N.nI90�890L0LH<.Q:MK�CJ4�4,0L8�b

�9Õ�Ô � 0�H/.N2�7)BE;~H9?[CJ7>Bä2��Ý=9?D?[2�: T .N2�KO��CN7 d ;<KOKO0LKO;<?[23BDC �<0�CN454G=n0�CN7 �L����¢ bãqg�:<;>=+}6;>=/.18+.AC1��0@BE;�F<2��<0�4,;<KO0�BGF>CN79�@CN7 d .N=n8/0�8�CN7+BGF>CN4_CJ4�4�=n0 � H<.1032�450�037/4�=9?D0sBGF<2�B}60~?D0 d 03C �<0)CAB�IL:�h��'#��/��� $ �������/b c .J45; � I90~4G=9?D06BE;OC18/0L7>BDCAgA:M:<;>=9?s4�=nI<KMCN454GC1;<72�4sI90�CJ79�Pgq;<?_BGF<23B d ;<KOKO0LKO;<?[23BDC �<0+CN454G=n09b

� ���� � � � � ��� �����

� &)�6" $ �>��"����� ��&��% @��!�*5"¼�Ç�� �E\{Ë�X�É:É ��X[Z�ÑnÇ/Ì{Ë�>nU �/Ë�ÓLÉ�ÌYÎ�Æ�X�ÌqÍ�ËzËÝZ1Ï�Õß/8>CABD0�8OIL:f¥�2��>C18 � ;>.Ngq0+2L798��9;<K�¤Y;/89�303?[4 � H<=nI<.ACJ45F/0�8OIL: c b T bu-<0�BE03?[4 � �L�����ã��/©�� � � ¢ ��� ��� � ����� � � � F<2L?D8 d ;L�<03? � £/��� H92��30L4 � ¦��� ��¢ b ��� bÅ_Ë3Ê¡Z�ËLÈPË�Õ�Ñ�Ø ��������������������� �!�!"$#��&%(')�_] �uÏ<Z{ÊLË�X�É�Z�ÌqØjÆ�Ρ¾�Ë�È Æ�X¡Ç�Ï<É�È�Z{× 0�]��XGË�Õ>X[Z{×5Ì�Æ�Ï<]�¾zÆ_b

¨W2L?D8/7903? � �3CN79��KO2L4GBD0L? � ¨ä;>.J;<KRI � ;<7�}�2�: � T CJK � �LKM=/.A.Q:>2L7 � -<037/?D;<4,0 �-/C d ��;3�<0L? � ©�0L?[.10L�L2�KMH � b�b3b � ;�} �+��F>CJ4�CJ4P2�4�Hn0 d CJ23. d ;>.A.10 d BDC1;<7f;>g_H/?D;/I<.J0LKM4�2L798H/=<�D�E.J0L43b��YF>CN4âI9;/;/�ç;>=n�3F�BÝBD; �9798 CQBD4503.AgOCJ7çBGF/0�.AC1I<?[2�?[C1034â;>gR?D0 d ?D0323BDCJ;<7/2L.KO2�BGF/0LKM23BDC d 23.�037>BGF>=94�CN2�4EBD43bP��F/0�I/;/;/�M4EBE03KO4�g1?D;<K´H90L?[C1;/8>C d 2L.�¨W2�BGF/0L?[CJ79�3�J4 � gq;<?¨W2L?D8/7903?P�1¨ £ ¨ � BD;f¨ £ ¨ £ � 2�B�}6F>C d F�KM23BGF/03KO2�BDC d CN2�7/4 � KM2���C d CJ2L7/4 � 2L798fH<=<�D�E.NCN4GB[4KR0L0�B�BD;çF/;<79;>=9? ��2L?[BDCN7 ¨W2L?D8/7903?�b ¼�Ç�� �E\�Ë�X[É�É ��X[Z�ÑnÇ/Ì{ËáCN4'890L8<C d 2�BE0L8 BE;�BGF/0KR03KR;<?{:á;>g+�'0�.���BD;L�<03? � �+2L?E?{:jß>79� � 2�798 ¥�2��<CJ8 T .J2L?E7903?~�NBGF<?D0L0TH/2�?�BDC d CNH92L7>BD423B�BGF/0O��23BGF/03?DCN79��4 � b c H9H/?D;<H9?[CJ2�BE0�.Q: � BGF/0MCN7<CABDCN2L._450 d BDCJ;<7�gq0323BD=9?D03460�C1�3F�BP2L?[BDC d .1034BGF<23BäKO2���0�=9H � ��F/0��9;<2L4GBW�9?[C1I>=<BE034���b+7:9<;<=Y>@?:APBED�F�?:AJGHI�K"9<;'F ?D0L2�8903?[4ä}+CN.A. d 03?G�BD23CJ7<. :'?D0 d ;/��7<CQ��0RBGF/0�7/2LKR034M;>g d ;/�G2L=<BGF/;<?[4�;>g � ��F<?D0�0�-9?D;/I<.J0LKM4�� � 2�H/CJ0 d 0�IL:c 798�: ��CA=Ý2L798 ©9CN.A.W�L2L798/43bR��F/0O79;�BE0O;<7fBGF/0O2L=<BGF/;<?[4+4GB[23BD0L4 � � ©9CA.N.W�L2L798/4+.AC1��034}+CJ7>BD0L?[4!;<7�BGF/0 2�7/2�8<CN2�7)H9?[23CJ?[C10 � 2L798+}�2L4ä2�7~0L8<CQBE;<?_;>g3749�;<=z>@?:APBED�F�?:AJGHI�K"9<;'Fgq;<?WBD0L7):<0L2L?[4 � I<=<BäCJ4s;�BGF/03?�}+CN4,0+?D0�.J2�BDCQ�<0�.Q:�4�2�790/b �ã{79890L0�8�BGF/0 d ;>.A.10 d BDC1;<7 ;>g�;3�<0L?64�C  LB{:'2L?[BDC d .1034~CN4)BE;/;�?[C d FÝBD;�?D0,�<CJ0�}�CN7 2H92��30+;<?�B�}6; � .10�BW2L.J;<790�2�H92L?[2���?[2LHnFub��YF>CN4�?D0 d ?D0323BDCJ;<7/2L./KO2�BGF�037>BGF>=94�CN2�4EBäF>C1�3F>. :?D0 d ;<KOKO0L798<4�BGF/0�I9;/;/�nb���F/0�0�8>CABD;<?[4 � ¥�2��>C18 � ;>.Agq0)2�798R�9;<K%¤Y;/8/�30L?[4 � 0� 3BE037982�7+CN7��<CQBD2�BDC1;<7PBD;http://www.g4g4.com � }6F/03?D0�;<790!}+CA.N.L�9798�2L?[BDC d .1034 � H/?D;/I<.J0LKM4 �KO2�BE03?DCN2L.N4 � 2�798�.ACJ79��4�Hn03?[BDCN790L7>B9BE;sBGF/0�H92L?[BDC d CJH/2�7>B[4�2L798+*92��<;>=9?Y;>g3BGF/0@��2�BGF/0L?[CJ79�<b

Ç{ÏfÂ�Æ/Õ9Ë�>nU Ë'Ó/ÌqÍ�Ë�ÒOÓ<ÌJZ�×�Ó3\�¿�ÆnÇ�XEÏ�Ë�Ø©<:��L2�?[2�F Ü�.N2�7/7903?{:�}+CQBGFâ¥�2��<CJ8 Ü�.N2�7/7903?{: � H/=nI>.NCN4,F/0L8'IL: c .1�3;<79�>=/CJ7j©�;/;/��4);>g F<2LHn0�.���CA.N. � �L�����ã��/©�� � � ¢ ��¢���� � � ���L� � � H/2�H90L?EI9;>=9798 � � ��� H92��30L4 � ¦��� ��� b �¢ bÅ_Ë3Ê¡Z�ËLÈPË�Õ�Ñ�Ø ��������������������� �!�!"$#��&%(')�_] �uÏ<Z{ÊLË�X�É�Z�ÌqØjÆ�Ρ¾�Ë�È Æ�X¡Ç�Ï<É�È�Z{× 0�]��XGË�Õ>X[Z{×5Ì�Æ�Ï<]�¾zÆ_b

�YF>CN4´H9;<H/=/.N2�?Ù�303790L?[23.1�ECJ7>BD0L?D034GBÙI9;/;/�·CJ4´�123=<BE; � I<CJ;/��?[2�H9F>C d 2L. CN7 BGF<2�BCAB gq;>.N.J;�}�4áBGF/0 KM23BGF/03KO2�BDC d 2L. ;{;>=9?D790,:¯;>g��L2�?[2�F´Ü�.N2�7/7903?{:#g1?D;<K d F>CN.J89F/;/;/8KO2�BGF/0LKM23BDC d 23.MH<=<�D�E.1034�CJ7 BGF/0 ��CQB d F/037çBD; 0�.103KR037>BD2L?{: 7<=9KOI903?TBGF/0L;<?{: 2L798 �4�=nI<4,0L�<=n037>BD.Q: � d ?{:<H<BD;/��?[2�H9F�:�b �@790�2LH9H90L23.�;>g@BGF/0TI/;/;/��CJ4)BGF<23B6CQB6CN4O032�4[:ÝBD;?D0L2�8�?D0���2L?D8<.J0L4�4s;>g�KM23BGF/03KO2�BDC d 2L.nI/2 d ����?D;>=9798�b �@7<. : F<2�H>BE03?[4 � 2L798���2�798)BGF/02�H/Hn03798<C d 034M?D0L�<=/CN?D0�2�7�:'KM23BGF/03KO2�BDC d 2L.�45;<HnF>CN4GBDC d 2�BDC1;<7�bá��F/0�4GBD;<?{:'4G=9?E?D;>=9798<4BGF/0+KM23BGF/03KO2�BDC d 4 � BGF/;>=n�3F)CQBWCN4äBGF/0+?D03KO2L?G�L2�I<.J0+KM23BGF/03KO2�BDC d 2L.92 d F>C10,�<03KR037>BD4@;>gBGF/0s:<;>=979��23=<BGF/;<?�BGF<2�B d ?D0323BD0�8+BGF/0�CN7>BE03?D0L4EB�CN7�BGF/0�4EBE;<?{: � 2L4_BD;>.18)IL:)F/0L?[4503.Ag92L798F/0L?äg12�BGF/0L? � 26H/?D;>gq0L4�4,;<?!;>gYKM23BGF/03KO2�BDC d 4�CJ7 ;<?G�nb

� ����ub]^�\���\�� ��� �"c �<e����+X�� `6����\2`6��W �3b]`

AX4 + BX2 + C� ���<��!�*,&���~$ �%������s"�����s*

�L0�BZI90�2�7�2�?EI<CQBE?[2�?{:~=97<CJ�<=n0�g12 d BD;<?DCQ�G23BDCJ;<7M8/;<KO23CJ7�b � 0+0L4EBD2�I<.ACJ45F/0�8)BGF/0gq;>.N.J;�}+CJ79�P?D0L4G=/.AB[4@CN7� ��� b

k����Lx�#���� $ �L0�Bf(X)

I90f2L7�:�79;<79�E��03?D;ÝHn;>. :/79;<K�CJ23.!CN7Z[X]

b ��F/0�gq;>.N.J;�}+CJ79�4GB[23BD0LKO0L7>B[4s2L?D060L�<=/C �>2L.J0L7>B ��JC � f(X2)

CN4@?D0�8>= d C1I>.10+CN7Z[X] ��NCNC � f(X)

CN4!?D0L8<= d CJI<.J0�CJ7Z[X] � ;<?�BGF/0L?D0�0G �CN4GB_H9;>.Q:<79;<KMCN2L.N4 G(X)

2�798H(X)CN7

Z[X]2�798�2�=97<CQB

u;>g

Z�ABGF<23BuCJ4 � 2�7+CJ7��<03?[BDCJI<.J0P0�.103KR037>B_;>g Z \{0} � 4�= d FBGF<2�B

uf(X) = G2(X) − XH2(X)b

p�x:#�x�yQy-�'#o* � �L0�Bf(X)

I90Ý2L7�:âHn;>. :/79;<K�CJ23.P;>gZ[X]

}6F>C d FáCN4OCN?E?D0L8<= d CJI<.J0ÝCN7Z[X]

b c 454G=9KR0OBGF<2�Bf(X)

F<2�46.1032�8>CJ79� d ;/0 � d C1037>BA2L798 d ;<7/4EBD2L7>B�BE03?EK

Cbã{7�2�8/8<CQBDC1;<7 � 4�=9H/Hn;<450�BGF<23Bä0�CABGF/03? uA

CJ4!79;�Bä2�4,�>=92�?D0PCN7Zgq;<?ä032 d F6=97<CAB

u;>g

Z;<?WBGF<2�BAC

CN4@79;�B!2645�<=92L?D0+CJ7Zb_��F/0L7

f(X2)CJ4�CN?E?D0L8<= d CJI<.J0+CJ7

Z[X]b

Ü<;<?s89;<KM2LCN7/4P;>g d F<2�?[2 d BE03?DCN4GBDC d2�ABGF<23B@CJ4 � gq;<?�8/;<KO23CJ7/4�CJ7M}6F>C d F −1 = 1�1450�0 � � H�b �" >��� �1�G� }606F<2��<0�2�KR;<?D0+H9?D0 d CJ450�?D034�=/.QB�bp�x:#�x�yQy-�'#o* � �3=9H9H9;<4,0)BGF<2�B

ZF<2�4 d F<2�?[2 d BE03?DCN4GBDC d

2bO�L0�B

f(X)I90~2�7�:�Hn;>. :/79;/�KMCN2L.�;>g

Z[X]}6F>C d FRCN4�CN?E?D0L8<= d CJI<.J0~CN7

Z[X]b��L0�B

AI906BGF/0).1032�8>CJ79� d ;/0 � d C1037>B;>g

f(X)b_�YF/037�BGF/0�gq;>.N.J;�}+CJ79�P4EBD2�BE03KR037>BD4@2�?D0�0��>=/CQ�>23.1037>B�b

�JC � f(X2)CN4@?D0�8>= d C1I>.10+CN7

Z[X]b

�NCNC � ��F/0L?D0s0G �CN4GB[4�2�=97<CAB u ;>g Z 4G= d F�BGF<2�Bu0,�<0L?{: d ;/0 � d C1037>Bu;>guf(X)

CN4�2s45�<=92L?D0CN7Zb

¼nX�Æ/Æ�Î[Ô��YF/0�0��>=/CQ�>23.1037 d 0�;>g@�NC � 2L798��NCNC � gq;>.N.J;�}�4�23B+;<7 d 0Og1?D;<KÂ��F/0�;<?D03K � 2L798(a1 + · · · + an)2 = a2

1 + · · · + a2n

gq;<?ä2�7�:a1 � . . . � an ∈ Z

b2

�YF/03?D0T2L?D0�KR;<?D0�H/?D0 d CN4,0�?D034�=/.QBD4~gq;<?~I<CJ�<=92�8/?[2�BDC d H9;>.Q:<79;<KMCN2L.N4~CJ7Z[X] �BGF<23B�CN4 � gq;<? f(X2) � }6F/03?D0 f(X)

CJ4�2j�>=92�8<?[23BDC d H9;>.Q:<79;<KMCN2L.�;>gZ[X]

�Jgq;<?I<CJ�<=92�8/?[2�BDC d H9;>.Q:<79;<KMCN2L.N4s;L�<03?!2���0�.18 � 4,0L03 ¢ � H9H�b ��� � �>���� � � bsã{7O;<?D8903?WBE;M��CQ�<003.J0LKO0L7>B[2�?{:ÝH9?D;/;>g14 � }60�?D0 d 23.N._BGF<23B+2f79;<79�D��0L?D;TH9;>.Q:<79;<KMCN2L.!;>g Z[X]CN4 d 23.N.J0�8VYX[Z1Ò�Z�ÌJZ�ÊLË+CAg�BGF/0���?D0323BD0L4EB d ;<KMKR;<7f8<C �<CN4,;<?s;>g�CAB[4 d ;/0 � d C1037>BD4�CJ4�2~=97<CAB�;>g

Zb c�<=92�8/?[2�BDC d Hn;>. :/79;<K�CJ23.

f(X) = AX2+BX+C ∈ Z[X]CN4�CJ?D?D0�8>= d C1I>.10@CN7

Z[X]CNg/2L798);<7<. :+CNgf(X)

CN4W2�H9?[CJK�CABDC �<0�Hn;>. :/79;<K�CJ23.L}6F/;<450�8<CN4 d ?[CJK�CJ7/2L7>B � B2 −4AC �CJ4@79;�B!2�4,�>=92�?D0�CJ7Zb

�� �������� �����c© ������� � ��!���"�� ��!$#%�&�&��'�()�&�*� +,��- .� �+*� '*���

� ��£

� 0�F<2��<0�BGF/0�gq;>.A.1;�}+CN79� d F<2L?[2 d BD0L?[CA�G2�BDC1;<7/4!;>g<BGF/0�I>C1�>=92�8<?[23BDC d H9;>.Q:<79;<KMCN2L.N4;>gZ[X]

}6F>C d F�2L?D0�?D0L8<= d CJI<.J0+CJ7Z[X]

bk����Lx�#���� � �L0�B

f(X) = AX2 + BX + CI/0�2L7�: H/?DCNKMCQBDCQ�<0��<=92�8/?[2�BDC dHn;>. :/79;<K�CJ23.�;>g

Z[X]bj�L0�B

f̃(X) = X2 + 2ABX + A2(B2 − 4AC)b���F/0gq;>.N.J;�}+CJ79��BGF<?D0�0�4EBD2�BE03KR037>BD4@2L?D060L�<=/C �>2L.J0L7>B �

�JC � f(X2)CN4@?D0�8>= d C1I>.10+CN7

Z[X] ��NCNC � f(X)CJ4�?D0�8>= d C1I>.10)CN7

Z[X] � ;<?�BGF/03?D0�0� LCJ4EBD4 S ∈ Z}+CQBGF

AC = S24�= d FBGF<2�B

A(2S − B)CN4@2645�<=92L?D0+CN7

Z ��NCNCAC � f(X)CN4u?D0�8>= d C1I>.10äCJ7

Z[X] � ;<?n45;<KR0ä4,�>=92�?D0�03.J0LKO0L7>BY;>g Z CN4u2�?D;/;�B�;>g f̃(X)b

¼nX�Æ/Æ�Î[Ô 03?[B[2LCN7<.Q:ÝBGF/0�BGF<?D0�0�4EBD2�BE03KR037>BD4�2L?D0�2L.A.!BE?[=n0�CAgf(X)

CN4�?D0L8<= d CJI<.J0�CN7Z[X]

b_��F>=94 � }60+KO2�:M2L454G=9KR0PBGF<2�B f(X)CN4�CJ?D?D0�8>= d C1I>.10+CN7

Z[X]b� 06�9?[4GB@4G=9H9H9;<4,0�BGF<2�Bs�JC � CJ4@BE?D=n0/b+ã1B�gq;>.A.1;�}�4sg1?D;<KÙ��F/0�;<?D03K � BGF<23B!BGF/03?D00� LCJ4EB!H9;>.Q:<79;<KMCN2L.N4

G(X)2L798

H(X) ∈ Z[X]2�798M2+=97<CQB

u;>g

Z4G= d F)BGF<23B

u(AX2 + BX + C) = G2(X) − XH2(X)b

�YF>=94 � G(X)CN4�.NCN790L2L?ä2�798

H(X)CJ4 d ;<7/4GB[2�7>B � 4�2�:

G(X) = αX + γ2L798

H(X) = βb

�YF/037uA = α2 � uC = γ2 � 2L798 2αγ − uB = β2

b6�+0L7 d 0 � AC = (u−1αγ)22�798u(2u−1αγ − B) = β2

b��YF/037R�NCNC � CN4!BE?D=n0�}+CABGF S = u−1αγ � I90 d 23=94,0A(2S − B) = (u−1)2(uA)(u(2S − B)) = (u−1αβ)2

b�P;�} 4G=9H9H9;<4,0@BGF<23B_�NCNC � CJ4�BE?[=n0 � BGF<23B�CN4 � BGF/0L?D0P0G �CN4GB S

2�798TCJ7

Z4G= d F+BGF<2�B

AC = S22�798

A(2S − B) = T 2b_��F/0L7O�JCACNC � CJ4!BE?[=n0P}+CABGF f̃(T 2) = 0 � I/0 d 2L=9450

f̃(X2) = (X2 + AB)2 − 4A2S2

= (X2 − 2AS + AB)(X2 + 2AS + AB)

= (X2 − T 2)(X2 + 2AS + AB)b

Ü�CN7/2L.A.Q: � 4G=9H9H9;<4,0MBGF<23B��NCNCAC � CN4+BE?D=n0/b'ã1B�CJ4 d .J0L2L?PBGF<2�BPBGF/0�8<CN4 d ?[CJK�CJ7/2L7>B�;>gf̃(X)

CN460L�<=923.�BE;16A3C � }6F>C d F�CN4�2R45�<=92L?D0OCN7 Z

I90 d 23=94,0f̃(X)

F<2�4�2O?D;/;�BCJ7Zb���F/0L7fBGF/03?D0O0� LCJ4EB

S2�798

TCN7

Z4G= d F�BGF<2�B

AC = S22�798

f̃(T 2) = 0b��037 d 0 �

0 = (T 2 + AB)2 − 4A2S2 = (T 2 − A(2S − B))(T 2 − A(2(−S) − B))b

�YF/03?D03gq;<?D0 � 4GCJ7 d 0 AC = S2 = (−S)2 � }60MKO2�:�2L454G=9KR0~BGF<23B AC = S22L798

A(2S − B) = T 2b�ã1Bägq;>.A.1;�}�4!g1?D;<K�BGF>CN4!BGF<23B

Af(X2) = A2X4 + ABX2 + AC = (AX2 + S)2 − T 2X2

= (AX2 − TX + S)(AX2 + TX + S)

= d2(A′X2 − T ′X + S′)(A′X2 + T ′X + S′) � � � �

� ��¢

}6F/0L?D0d = gcd(A, T, S)

2�798A′ = A/d � T ′ = T/d � S′ = S/d

bj�+0L7 d 0 �A′X2 −T ′X +S′ 2�798 A′X2 +T ′X +S′ 2L?D0PH/?DCNKMCQBDCQ�<0�Hn;>. :/79;<K�CJ23.J4!;>g Z[X]

b ;<7��<03?[4,0�.Q: � 2)}603.A.1����79;�}�7f?D0L4G=/.ABP;>gä¨_2L=94�4+�1450�04 � � H�b ��� � � � 0L4EBD2�I<.ACJ45F/0L4�BGF<2�BBGF/0MH/?D;/8<= d BP;>g_H9?[CJK�CABDC �<0�H9;>.Q:<79;<KMCN2L.N4PCN4�23.J45;�2MH/?DCNKMCQBDCQ�<0�H9;>.Q:<79;<KMCN2L.�b6��F/0L7 �4�CN7 d 0

f(X2)CN4@H/?DCNKMCQBDCQ�<0 � g1?D;<K#� � �G� }60��30�B A = d2 � g1?D;<K }6F>C d FM�NC � gq;>.A.1;�}�43b 2

�������'#���b��YF/0�H9;>.Q:<79;<KMCN2L.f(X2)

F<2L4+0G L2 d BD. :�gq;>=9?�?D;/;�B[4�CJ7Z� d ;>=97>BDCJ79�~K�=/.1�BDCJH<.NC d CABDCJ0L4 � CNguBGF/03?D0�0G �CN4GB S

2�798TCN7

Z4�= d FMBGF<23B

S2 = AC � f̃(T 2) = 0 � 2L798T 2 − 4AS

CJ4ä2�4,�>=92�?D0�CJ7Z �;�BGF/0L?�}+CJ450 � CABWF<2�4!79;6?D;/;�BD43b ��gn4�Hn0 d CJ23.<CNKOH9;<?[B[2�7 d 0CJ4@BGF/0 d 2L4,0+}6F/037

T 2 − 4ASCN4�79;�B@2~45�<=92L?D06CJ7

Z � 4�CN7 d 0 f(X2)BGF/0L7�F<2�4sB�}6;CJ?D?D0�8>= d C1I>.106�>=92�8<?[23BDC d g12 d BE;<?[4 � I>=<B�79;�?D;/;�B[4Lb�P;�}á}60sH9?D;3�<0 � }+CABGF/;>=<Bu=94GCJ79� d ;<KOH<.10G P7<=9KOI903?[4 � BGF<2�B�2�7�: � �303790L?[23.NCQ��0�8��I<CJ�<=92�8/?[2�BDC d Hn;>. :/79;<K�CJ23.�;>g / [X]

CN4@?D0�8>= d C1I>.10+CN7 / [X]b

p�x:#�x�yQy-�'#o* � �L0�BmI90+2�7�:�Hn;<4GCABDC �<0�CJ7>BD0��303? � 2L798�.10�B

F (X) = AX4m + BX2m + CI90�2�H9;>.Q:<79;<KMCN2L.9CN7 / [X]}+CABGF

A 6= 0b_��F/0L7

F (X)CN4@?D0�8>= d C1I>.10+CN7 / [X]

b¼nX�Æ/Æ�Î[Ôäã1BYCJ4 d .1032�? � �<CN2@BGF/0 d F<2L79�30�;>gL�>2L?DCN2�I>.10 Y = Xm � BGF<23BYCABu4G=.� d 0L4�BD;�H9?D;3�<0BGF/0 d 2L4,0

m = 1b � 0�KM2�:�2L.N4,;R2�4�4�=9KO0

A > 0b��P;�}�4G=9H9H9;<4,06BGF<23B

F (X)CN4CJ?D?D0�8>= d C1I>.10~CN7 / [X]

b�ã1Bsgq;>.A.1;�}�4�g1?D;<K �JCAC � ;>g��YF/0L;<?D0LK � BGF<23BB2 − 4AC < 02�798�BGF<2�B � gq;<?�2L7�:T?D032L.W7<=9KRI/0L? S � }60RF<2��<0 AC < S2

;<?A(2S − B) < 0

b�3CJ7 d 06BGF>CJ4�CN4 d .1032�?[.Q:�0L�<=/C �>2L.J0L7>B�BD;MBGF/0 d ;<7>BE?[2�8<C d BDC1;<7B2 < 4AC

2�798C < 0 �BGF/0�H9?D;/;>g�CN4 d ;<KMH/.J0�BD09b

2

c 4)2�7Ý2�H/H/.AC d 23BDCJ;<7';>g!�YF/0L;<?D0LK � � }60 d ;<7/4GC18/0L?��Ngq;<?�2L7�: a � b ∈ Z � BGF/0Hn;>. :/79;<K�CJ23.Fa,b(X) = X4 − 2(a + b)X2 + (a − b)2

b� 0��9?[4GB�890�BE03?EK�CJ790f790 d 03454�2�?{:Ý2�798�4G=.� d CJ0L7>B d ;<798<CQBDC1;<7/4~;<7

a2L798

b4,;BGF<23B

Fa,b(X)CN4P?D0�8>= d C1I>.10~CJ7

Z[X]�1�30L7903?[2L.ACA�ECN79�6BGF>CJ4s}�2�:fH/?D;/I<.J0LK c@� ;>g�BGF/0�����>� -/=<BE7/2�K ;<KMHn0�BDCABDCJ;<7� � � H/H�b ���� � ������� � bãqg

ZF<2�4 d F<2L?[2 d BD0L?[CJ4EBDC d

2 � CQB�CJ4 d .1032�?YBGF<23B Fa,b(X) = (X2 +a−b)2b!��0� 3B4�=9H/Hn;<450+BGF<23BäBGF/0 d F<2L?[2 d BD0L?[CJ4EBDC d ;>g

ZCJ4�8<C���03?D0L7>B!g1?D;<K

2b��?D;<K#�YF/0L;<?D0LK � � CQBgq;>.N.J;�}�4�2�B�;<7 d 06BGF<2�B

Fa,b(X)CN4�?D0�8>= d C1I>.106CJ7

Z[X]CNg�2L798f;<7<. :OCNg�[email protected];<790;>g�BGF/0603.J0LKO0L7>B[4

4(

(a + b)2 − (a − b)2)

= 16ab � 2(a − b) + 2(a + b) = 4a �2�798 −2(a − b)+2(a+ b) = 4bCJ4ä2�45�<=92L?D0�CJ7

Z �BGF<23B�CJ4 � CNg92L798~;<7<.Q:�CNg92�B�.1032�4EB;<7906;>gnBGF/0�03.J0LKO0L7>B[4

a � b � 2�798 abCJ4@2�4,�>=92�?D0�CJ7

Zb� 0�79;�BD0�BGF/0�gq;>.N.J;�}+CJ79� d ;<7/4,0L�<=n037 d 034 �

� b���F/0~H9;>.Q:<79;<KMCN2L.Fa,b(X)

CJ4�?D0L8<= d CJI<.J0)CJ7��p[X]

gq;<?s2L7�:�H/?DCNKR0p � I/0 d 2L=9450 �IL:�BGF/0M.N2�}#;>g!�<=92�8/?[2�BDC d ?D0 d CNH9?D; d CQB�: � BGF/0MH9?D;/8>= d B+;>gä79;<79�G4,�>=92�?D034+CN7�� p

CN423.Q}�2�:/4+2R4,�>=92�?D0MCJ7��pb��N�YF>=94 � CAB�CJ4�0L2L4[:�BD;��9798�CN?E?D0L8<= d CJI<.J0RH9;>.Q:<79;<KMCN2L.N4;>g��

[X]}6F>C d F�2�?D0O?D0�8>= d C1I>.10MCJ7��

p[X]gq;<?�2L7�:TH/?DCNKR0MCJ7>BD0��303?

p �BGF/0 d 2L4,0

a = 5 � b = 41CN4 d ;<7/4GC18/0L?D0L8�CJ7� £ � H/H�b �������>����£ � ���L��� � b

� � �

� bX4 − 2(Y + W )X2 + (Y − W )2

CJ4�CN?E?D0L8<= d CJI<.J0+CJ7��[X, Y, W ]

b� b)�L0�B

α2�798

β8/0L79;�BD0s2L?GI>CABE?[2L?{:+45�<=92L?D0s?D;/;�B[4_;>g

a2L798

b � ?D0L4�Hn0 d BDCQ�<0�.Q: � CN7+4,;<KO00G LBD0L7/4GC1;<7Ý��0�.18';>g!BGF/0f�n03.J8';>g��>=n;�BDC1037>BD4�;>gZ � 452�: QZ

�J4,0L08 � � H9H�b ���L� ���£�£�� � bR�3CJ7 d 0 Fa,b(α + β) = 0 � }60OF<2��<0M2L.N4,;���C �<0L7�2�7�03.J0LKO0L7>B[2�?{:�H/?D;/;>gBGF<2�BFa,b(X)

CJ4sBGF/0�K�CJ7<CNKO23.�H9;>.Q:<79;<KMCN2L.�;>gα + β

;3�<0L?QZ

CAg_2L798�;<7<.Q:RCAg79;<790�;>g�BGF/0�0�.103KR037>BD4a � b � ;<? ab

CN4P2�4,�>=92�?D0)CJ7Z�A}6F>C d F d ;<7/4EBDCABD=<BD0L4 � gq;<?4�Hn0 d CN� d �>2L.A=n0L4+;>g

a � b ∈ � � 2)�<0L?{: d ;<KMKR;<7�0� �03? d CJ450�CN7�2L7�:�CN7>BE?D;/8<= d BD;<?{:d ;>=9?[4,0�;<7O0G LBD0L7/4GC1;<7��n03.J8/4 � b� CABGF)KR;<?D0+�30L7903?[2L.ACAB{: � IL:)B[2��LCJ79� A = 1}60+�30�BWg1?D;<K���F/0�;<?D03K � BGF/0�gq;>.J�.1;�}+CN79� d F<2L?[2 d BD0L?[CA�G2�BDC1;<7�;>g9BGF/0+03.J0LKO0L7>B[4@;>g�45;<KR0�23.1�30LI/?[23C d�d .J;<4�=9?D0+;>g

QZ�1450�0

£ � H9H�b ��¢�� � ��¢� �>����� � � }6F/;<4,0)KMCN7<CJKM2L.uHn;>. :/79;<K�CJ23.�;L�<03? QZCN4P2�I<CJ�<=92�8/?[2�BDC dHn;>. :/79;<K�CJ23.�;>g

Z[X]b

p�x:#�x�yQy-�'#o* £ �3=9H9H9;<4,06BGF<23BZ8/;/0L4P79;�B�F<2��<0 d F<2L?[2 d BD0L?[CJ4EBDC d

2bM�L0�B

QZI/0�2L7�:2L.J�30�I<?[2LC dPd .1;<4G=9?D0�;>g

QZ � 2�798R.10�B ξI/0�2L7�:f03.J0LKO0L7>B�;>g

QZb��L0�B

F (X)I/0)BGF/0KMCN7<CJKM2L.nHn;>. :/79;<K�CJ23.�;>g

ξ;3�<0L?

QZb���F/0+gq;>.A.1;�}+CN79��4EBD2�BE03KR037>BD4@2�?D0�0��>=/CQ�>23.1037>B �

�JC � F (X) = X4 + BX2 + CCJ4@26I<CJ�<=92�8/?[2�BDC d Hn;>. :/79;<K�CJ23.�;>g

Z[X]b

�NCNC � � 0 d 2�7)}�?DCQBE0 ξCJ7M45;<KR0�;>gnBGF/0�gq;>.N.J;�}+CJ79��gq;>=9?_}�2�:/4 �

±√

B ±√

B2 − 4C

2

b}6F/03?D0

B2�798

C2L?D0+03.J0LKO0L7>B[4@;>g

Z4G= d F6BGF<23B

B2 − 4CCN4�79;�BW2�45�<=92L?D0�CN7

Z � 2�798 B ± 2√

C2�?D0+79;�B!4,�>=92�?D034@CN7

ZCNg

CCN4@2645�<=92L?D0+CN7

Zb

����&P��#���$ ���'6 ��� ��2�BD23.NCJ;á��b!¨_=n03?[4,037>�[�>2LCJ� � U = Z1Ò~Vn\{Ë Ç{XEXGË�Õ�Ç>×�Z�ÑLZA\NZ�ÌqØ[Â/X[Z�Ì{Ë�X[Z{Æ�Ï'Î�Æ�X

f(X2) � �L����� �<�������>������� b ��� �L0�;<7/2L?D8 ÜYb T .1;<4GCJ7/45��C � ¨ä03?[2L.J8 �/b c .10G L2�798/0L?[45;<7 � 2�798 �L;<?D037 b)�32�?[45;<7 �

�LÍ�Ë = Z �9ÌqØ �[É�Ë�×EÆ�ÏYÕ ��ZN\A\NZJÓ�Ò �>Æ/ÈPË¡\A\q¼�Ç/ÌqÏ/Ó�Ò�Ë Ó<ÌqÍYË�ÒOÓ/ÌJZ{×[Ó�\¡Â�Æ�Ò~V�Ë�ÌJZ�ÌJZ�Æ�Ï �c KR03?DC d 2L7f��23BGF/03KO2�BDC d 2L.��';<7>BGF>.Q: � $������ �����>� �G� �>� � ���>�" b ���Ý� b T 0�CABGF���C d F/;>.J45;<7 � � ã{7>BE?D;/8>= d BDC1;<7~BE; c I<4GBE?[2 d B c .J�30�I<?[2 � � � CA.10,: � �" : � b £�� ¨ä0L;<?D�30�- @;>. :>2�2L7986¨W2�I9;<?�����0����; � � -9?D;/I>.103KO4_2L798+�YF/0L;<?D0LKM4_CN7 c 7/23.Q:<4�CN4!ã[ã � ��LH9?[CJ79�303? � �" � � b ¢�� ��BE0,�<037�¤Y;<KM2�7 � � Ü�CJ03.J8~�YF/0L;<?{:� � �LH9?[CJ79�303? � �� � >¢ b

��2�BD23.NCJ;R��b9¨�=n0L?[450L7>�[�>23C1�¦s7<CQ�<03?[4�CJ8/2�8 9c ß ß©9=n0L79;<4 c CJ?D034c ?D�30L7>BDCN7/[email protected]

� � �� � � � � � �

¸ r�k"tOj�s�s�r/mHk"t~lOr<p's�x�k�����v{r"��m<�j�yzj�sR��x<p"s � j3p k�y]�j�v{rN�:r/lO|�j�p�t�p4r�k�s0�4x<v{|/j�p�l~v x�k��:mHk/s�t~x<vP��mOj� �/�R�/���{� �:�/�<�'����»0p�jY�j<tOr/l mOj

(?)x���v4�j�sRmOj8p k�y]�j�v{rNl~p:�<lw�4k"jN�4k'j�mOj���v{r"��m��j�y¡j�x¢�j<t��j�s�r�k<y�l-ss°x<p's�s�r/mHk"t~lOr<p �

F<2��<=n0ÝH/?D;/I<. �0LKO0Ý450L?[2TH/=nI>.NC @0�8<2�7/4M.1034R8/03=� Ý.N2�79��=n034�;�� d C10�.N.J0L4O8>= 2L7/2�8<2�12L79��.J23CJ4M0�B6g1?[2L7�Ìd 23CJ4 � b�¥�2�7/4~.1034�7<=9K4@03?D;<4 � � � � ¢ 0�B � � .�� 2�79��.N2LCN4�H9?�@0 d @0L8903?[2�.10g1?[2�7�Ìd 2LCN4 � 0�B�8/2L7/4M.1034O7<=9K4@0L?D;<4 � � £ � � 0�B � � .10�g1?[2�7�Ìd 2LCN4RH/?�@0 d @0�8/0L?[2T.�� 2�79��.N2LCN4Lb¥�2�7/4P.J2�4,0 d BDC1;<7�89034�4,;>.A=<BDC1;<7/4 � .J0�H9?D;/I>. �03KR0�4,03?[2MH/=nI>.NC @0O8<2�7/4P.N2�.J2L79��=n0O8/0~.J2H9?[CJ7 d CNH923.10�45;>.N=<BDCJ;<7�H9?�@0L450L7>B�@0L09b�32�?�@0�8<2 d BDCJ;<7+45;>=nF<2LCQBE0@?D0LKO0L? d CJ0L?9§50L2L79����2L? d �903?E?[C103?�0�B_��2�?�BDCJ7�¨W;>.18<4GBD03CN7 �890+.{� ¦s7<C �<0L?[4GCAB�@0�890~�';<7>BE?�@032L. � 8Y� 2��<;>CN?WBE?[2�8<=/CQBä.1034@H9?D;/I>. �03KR034Lb

Þ �Pæ+à b¤¼nX�Æ3V�Æ�É�½Ë�VYÓLX�Ë�ZJÍ�½Ó3\1ØuÆ�Ë�Ï�× �¡Ë�]�Æ�XDÓ3É�Æ>Ê�]�Å_ÆnÇLÒOÓ�Ï>Z�Ë�b��;>CAB

ABC=97�BE?DCN2�79��.J0 2 d =<BD2L79��.10/b���;>CABá?D0345H90 d BDC �<0LKO0L7>B

r0�B

R.1034?[2�:<;<7/4�8/0L4 d 0L? d .J0L4�CJ7/4 d ?DCQB�0�B d CN? d ;<7/4 d ?DCQB � s

.J0 450LK�C1�GHn03?DCNK �0�BE?D0 � d � 0L4EBG� �2��58>CJ?D0s = 1

2(a+b+c)

b>��;>CQBma

.J2�.J;<79��=n03=9?�890s.N2�K4@0�8>CJ2L790�890A

�2BC � 0�B�4,;>CQB wa

.J2.1;<79��=n0�=9?W8/0�.N2�I>CJ4�4,0 d BE?DC d 0sCN7>B @0L?[C10�=9?D0P890�.{� 2L79��.10A � KO0L4G=9?�@0L0P890 A

23= d'È;�B @0BC

b�L0L4W.J;<79��=n03=9?[4mb � mc � wb

0�Bwc4,;<7>BW8�@03�97<C1034!890�KO2L7<C �03?D0P2L7/2L.J;/��=n09b9�';<7>BE?D03?�<=n0 �

�12 � 3s2 − r2 − 4Rr

8sRr≤

� ����� � ���

ma

awa

≤ s2 − r2 − 4Rr

7sRr�

�1I � 3

4≤

� ����� � ���

m2a

b2 + c2≤ 4R + r

4R

bÞ �Pæ@½ b¤¼nX�Æ3V�Æ�É�½Ë�VYÓLX3¿�Æ/Ë �6Æ/ÈsÓ�XGÕ�]�¼nÆ�X,Ì1Ó�\�Ë¡É5]�¾qË']�� = UPb��ã{7/4�H/CN?�@068>=T-9?D;/I>. �03KR0 ��� bq¥ � ��23BGFub9¨_23��0�B[BE0 ��� � £ �: � � �� � � � H�b ��� � b ���;>CAB

x ∈(

0, π2

) b��';<7>BE?D0L?��>=n0 ��12 � [2 + cos x

3

]

[

2(1 − cos x)

x2

]

>1 + cos x

2�

�1I � 2 + cos x

3<

1 + cos x

2<

2(1 − cos x)

x2

bÞ �Pæ å b¤¼nX�Æ3V�Æ�É�½Ë�VYÓLX ��Z�ÌDÇ�Á�ÊLÆ�Ï9ÓLX¡ÇL]�Æ_Ç<×�Ó�XGË�É�Ì1]8Å�ÆnÇ�ÒOÓLÏ<Z{Ë>b

��;>CABABCD

=97~H92L?[2L.A. @0�.1;/��?[2LKOKO09b c .{� 23C18/0�8Y� =9790�? �0���.J0P452L7/4���?[2�8<=92�BDC1;<7 �BE?D;>=>�<0L?ä=97MHn;>CN7>BM4�=9?

ABBE0�.Y�>=n0

AM = 1

5AB

b

� ���Þ �PæPæ b ¼nXGÆ3V�Æ�É�½ËRVYÓ�X}ËÝZ�0>Í/Ó<Ë,\zÄ�Æ9Ì�×DÍ�Ë�Ì{Æ<Ê�] ��Ï>Z�ÊLË�X[ÉGZ�Ì'½ËdËjË�ÒfÆ�X[Z1Ó�\�Õ9Ë �LË�XEX�Ë �¾�Ë>Ç>ÊLË�] = ÌDÔ�¿�Ë�ÓLÏ/]�¾ �/b

�@7 d ;<7/4�CJ8 �03?D0 8/03=�  d 0L? d .J0L4f8<CN4 ;{;>CN7>BD4f0�BO79;<79� d ;<79��?[=n0L7>B[4Γ1

0�BΓ2 � 890d 037>BE?D0L4ä?D0345H90 d BDCAg14

O1

0�BO2

b���;>CQBQ.J0�H9;>CJ7>B_8�� CN7>BE03?[4,0 d BDC1;<7)8/0L4ä890�=� �B[2�79�3037>BE034d ;<KMKM=979034 � t1

0�Bt2 � �>=/C�790 d ;>=9Hn037>B�H92L4M.10T450���KO0L7>B O1O2

bs¦s790�BD2L79�30L7>BD0d ;<KMKM=9790 � tc � �<=/C d ;>=9Hn0O.J0R4,0L��KR037>B O1O2

CJ7>BD0L?[450 d BD0O.1034+B[2�79�3037>BE034t10�B

t20L7E1

0�BE2 � ?D0L4�Hn0 d BDCQ�<03KR037>B�b��;>CABP.J06Hn;>CN7>B@K�CN.AC10�=f8>=�4,0L��KR037>B

O1O2

b��';<7>BE?D03?s�>=n0P � Q � E1

0�BE24,;<7>B!4G=9?ä=97MK È03KR0 d 03? d .10/b

Þ �Pæ � b¤¼nX�Æ3V�Æ�É�½Ë�VYÓLX �<ÓLÉGZN\{ËYÂ��� X,Ì�Æ�Ó���Ë¡]��uÏ<Z{ÊLË�X�É�Z�Ì"½Ë6Õ9Ë¡¼/\{ÆLZ�Ë¡É3ÌJZN]�Å_ÆnÇLÒOÓ�Ï>Z�Ë�b��;>CABa1 � a2 � . . . � an

8/0L4@79;<KOI/?D034s?�@0L03.N4@Hn;<4GCABDCAg14s0�BEn =

n∑

i=1

(n−1∑

j=0

aji

)−1 b

�3Cr = n

√a1a2 · · · an ≥ 1 � KO;<7>BE?D0L?!�<=n0 En ≥ n

(n−1∑

j=0

rj

)−1 H9;>=9? ��12 � n = 2 � �qI � n = 3 � � d � ? n ≥ 4

bÞ �Pæ�� b¤¼nX�Æ3V�Æ�É�½Ë�VYÓLX �<ÓLÉGZN\{ËYÂ��� X,Ì�Æ�Ó���Ë¡]��uÏ<Z{ÊLË�X�É�Z�Ì"½Ë6Õ9Ë¡¼/\{ÆLZ�Ë¡É3ÌJZN]�Å_ÆnÇLÒOÓ�Ï>Z�Ë�b��;>CAB

a1 � a2 � . . . � an8/0L4@79;<KOI/?D034s?�@0L03.N4@Hn;<4GCABDCAg14s0�B

En =1 + a1a2

1 + a1

+1 + a2a3

1 + a2

+ · · · +1 + ana1

1 + an

b��;>CAB

r = n√

a1a2 · · · an ≥ 1b

�12 � �';<7>BE?D03?��>=n0 En ≥ n(1 + r2)

1 + r

H9;>=9?n = 3

0�Bn = 4

b�1I � ? �';<7>BE?D03?��<=n0�;>=O?�@0�gJ=<BE03?

En ≥ 5(1 + r2)

1 + r

bÞ �Pæ�� b¤¼nX�Æ3V�Æ�É�½Ë�VYÓLX �<ÓLÉGZN\{ËYÂ��� X,Ì�Æ�Ó���Ë¡]��uÏ<Z{ÊLË�X�É�Z�Ì"½Ë6Õ9Ë¡¼/\{ÆLZ�Ë¡É3ÌJZN]�Å_ÆnÇLÒOÓ�Ï>Z�Ë�b��;>CAB

a � b � c � d 0�B r8/0L4�79;<KRI<?D0L4@?�@0L03.N4@H9;<4�CQBDCNg14äBD03.N4s�<=n0

r =4√

abcd ≥ 1b�';<7>BE?D0L?��>=n0

1

(1 + a)2+

1

(1 + b)2+

1

(1 + c)2+

1

(1 + d)2≥ 4

(1 + r)2b

Þ � � � b¤¼nX�Æ3V�Æ�É�½Ë�VYÓLX ��Z�ÌDÇ�Á�ÊLÆ�Ï9ÓLX¡ÇL]�Æ_Ç<×�Ó�XGË�É�Ì1]8Å�ÆnÇ�ÒOÓLÏ<Z{Ë>b�3C

m0�B

n45;<7>B�89034+037>BDC103?[4�Hn;<4GCABDCAg14�BE0�.J4��<=n0

m ≥ n � 0�B�4�C a � b � c > 0 �KR;<7>BE?D03?��<=n0am

bm + cm+

bm

cm + am+

cm

am + bm≥ an

bn + cn+

bn

cn + an+

cn

an + bn

b

� �� Þ � � Û b¤¼nX�Æ3V�Æ�É�½Ë�VYÓLX�Ë�Z{×DÍYË,\�Æ�Ó<Ì1Ó�ZN\A\�Ë¡]8Å_ÆnÇ>Ë�Ï<] ��XDÓLÏY×EË>b-<;>=9?

a � b � c ∈ (0, 1) � BE?D;>=>�<03?!.N26H/.A=94sHn0�BDCABD06I9;<?D79064G=9H�@03?DCJ03=9?D060�Bä.J26H<.N=94��?[2�798/0+I/;<?E790�CJ7<g @03?DCJ03=9?D0�8/0a+ b+ c+abc

0L7�BD0L7/2L7>B d ;<KOH>BE0�8/0�.N2 d ;<7>BE?[23CJ7>BD0ab + bc + ca = 1

bÞ � � Þ b¤¼nX�Æ3V�Æ�É�½Ë�VYÓLX �<ÓLÉGZN\{ËYÂ��� X,Ì�Æ�Ó���Ë¡]��uÏ<Z{ÊLË�X�É�Z�Ì"½Ë6Õ9Ë¡¼/\{ÆLZ�Ë¡É3ÌJZN]�Å_ÆnÇLÒOÓ�Ï>Z�Ë�b�12 � �';<7>BE?D03?��<=n0�4GC 0 ≤ λ ≤ 4 � 2L.J;<?[4 � H9;>=9?WBE;>=94�.1034s79;<KOI/?D034s?�@0L03.N4sH9;<4�CQBDCNg14 x �

y � z � t �(t2 + 1)(x3 + y3 + z3) + 3(1 − t2)xyz

≥ (1 + λt)(x2y + y2z + z2x) + (1 − λt)(xy2 + yz2 + zx2)b

�1I � �3C t = 1

4

0�Bλ = 4 � .�� CJ7�@0L��2L.ACAB�@0 d CJ�58/0L4�4�=94s8/0,�<CJ0L7>B

17(x3 + y3 + z3) + 45xyz ≥ 32(x2y + y2z + z2x)b

�9?D;>=>�<03?WBE;>=<BD0L4�.1034ä�>23.10�=9?[4sHn;<4GCABDC �<0L4@890δBD03.A.1034s�<=n0�.�� CJ7�@0L��2L.ACAB�@0

x3 + y3 + z3 + 3δxyz ≥ (1 + δ)(x2y + y2z + z2x)45;>CABs�>2L.N2�I>.10OH9;>=9?�BE;>=94x � y � z

�<=/Cä45;<7>B � �NC � 8/0L4�79;<KRI<?D0L46?�@0�0�.J4�H9;<4�CQBDCNg14 ��NCNC � 8/0L4�.J;<79��=n03=9?[4�8/0L4 d'È;�B @0L4s8�� =97�BE?[CJ2L79��.10/bÞ � � à b�¼nX�Æ3V�Æ�É�½ËäVYÓLX0¿LÇLÓ�Ï ��Æ�Æ�É�×GÆzÅ_Æ�ÒfË�X�ÆqË ½ÓLX�Ê�Ç>Ë �G]���Ï>Z�ÊLË�X[ÉGZ�Ì'½Ë�Õ9Ë �<Ó�\N\JÓ<Õ9ÆL\AZ�Õ�]�<Ó3\A\1Ó>Õ9ÆL\NZ{Õ�]���ÉGVYÓ3Ð�Ï�Ë

(×EÆ�Ï/É5Ó<×DX ½Ë Ó �3Æ�É,Í>Z�Æ = Ë¡ZJÒ�ZqØ3Ó

)b

��;>CABABC

=97MBE?[CJ2L79��.10�?D0 d B[2�79��.J0679;<79�GCN4,; d �0�.108Y� 2�79��.J0P8/?D;>CQB_037A0�BuBD03.>�<=n0

AC > ABb���;>CAB

D.J0H/CJ0�8O890+.J26F<2L=<BD03=9?�2�I/23CJ4�4 @0L068/0

A4�=9?ä.10 d'È;�B @0

BCb��;>CAB

G.10)Hn;>CN7>B�8�� CN7>BE03?[4,0 d BDC1;<7�8/0).J2M8<?D;>CABD0

AD0�B890~.J2MH/2�?[23.N. �03.J0 �2

ABH92L454�2�7>B�H92L?

Cbu��;>CQB

E0�B

F89034�H9;>CJ7>B[4�BE0�.J4��<=n0ACGE

0�BBFGE

gq;<?DKR037>B?D0L4�Hn0 d BDCQ�<03KR037>B�89034P?D0 d BD2L79��.1034LbY��;>CABH.{� CN7>BE03?[4,0 d �BDC1;<7�890

AG0�B

BFbu��;>CABs�n7/23.103KR037>B

O1

0�BO2

.J0L4CJ7>BD0L?[450 d BDCJ;<7/4w?D0L4�Hn0 d BDCQ�<034 8/0L4 8>CJ2��3;<7/2L.J0L4w89034�<=92�8/?[CN.N23B �03?D0L4CDHF

0�BBDGE

b�';<7>BE?D0L?O�<=n0Ý.1034OBE?[CJ2L79��.1034ABC � DFE

0�BDO1O2

4,;<7>B!450LKOI<.N2�I>.1034Lb ...................................................................................................................................................................................................................................................................................................................................................................................................................................................................................................................................................................................................................................................................................................................................................................................................................

..

.

..

.

..

.

..

.

..

.

..

.

..

.

..

.

..

.

..

.

..

..

.

..

.

..

.

..

.

..

.

..

.

..

.

..

.

..

.

..

.

..

.

..

.

..

.

..

..

.

..

.

..

.

..

.

..

.

..

.

..

.

..

.

..

.

..

.

..

.

..

.

..

.

..

.

..

..

.

..

.

..

.

..

.

..

.

..

.

..

.

..

.

..

.

..

.

..

.

..

.

..

.

..

..

.

..

.

..

.

..

.

..

.

..

.

..

.

..

.

..

.

..

.

..

.

..

.

..

.

..

..

.

..

.

..

.

..

.

..

.

..

.

..

.

..

.

..

.

..

.

..

.

..

.

..

.

..

..

.

..

.

..

.

..

.

..

.

..

.

..

.

..

.

..

.

..

.

..

.

..

.

..

.

..

..

.

..

.

..

.

..

.

..

.

..

.

..

.

..

.

..

.

..

.

....

..

..

..

..

..

..

..

..

..

..

..

..

..

..

..

..

..

..

..

..

..

..

..

..

..

..

..

..

..

..

..

..

..

..

..

..

..

..

..

..

..

..

..

..

..

..

..

..

..

..

..

..

..

..

..

..

..

..

..

..

..

..

..

..

..

..

..

..

..

..

..

..

..

..

..

..

..

..

..

..

..

..

..

..

..

..

..

..

..

..

..

..

..

..

..

..

..

..

..

..

..

..

..

..

..

..

..

..

..

..

..

..

..

..

..

..

..

..

..

..

..

..

..

..

..

..

..

..

..

..

..

..

..

..

..

..

..

..

..

..

..

..

..

..

..

..

..

..

..

..

..

..

..

..

..

..

..

..

..

..

..

..

..

..

..

..

..

..

..

..

..

..

..

..

..

..

..

..

..

..

..

..

..

..

..

..

..

..

..

..

..

..

..

..

..

...........................................................................................................................................................................................................................................................................................................................................................................................................................................................................................................................................................................................

..

..

..

..

..

..

..

..

..

..

..

..

..

..

..

..

..

..

..

..

..

..

..

..

..

..

..

..

..

..

..

..

..

..

..

..

..

..

..

..

..

..

..

..

..

..

..

..

..

..

..

..

..

..

..

..

..

..

..

..

..

..

..

..

..

..

..

..

..

..

..

..

..

..

..

..

..

..

..

..

..

..

..

..

..

..

..

..

..

..

..

..

..

..

..

..

..

..

..

..

..

............................................................................................................................................................................................................................................................................................................................................................................................................................................................................................................................................................................................................................................................................................................................................................................................................................................................................................................................

...........................................................................................................................

A

B

C

D

E

F

G

H

O1

O2

Þ � � ½ b�¼nX�Æ3V�Æ�É�½ËäVYÓLX0¿LÇLÓ�Ï ��Æ�Æ�É�×GÆzÅ_Æ�ÒfË�X�ÆqË ½ÓLX�Ê�Ç>Ë �G]���Ï>Z�ÊLË�X[ÉGZ�Ì'½Ë�Õ9Ë �<Ó�\N\JÓ<Õ9ÆL\AZ�Õ�]�<Ó3\A\1Ó>Õ9ÆL\NZ{Õ�]���ÉGVYÓ3Ð�Ï�Ë>b

�@7�89;<7/790�=976BE?DCN2�79��.J0ABC

0�BW45;>CABP=97~H9;>CJ7>Bä�>=n03. d ;<79�>=n0�4�=9?�.N2+8/?D;>CQBE0

BCb>��;>CAB

A1

.{� CN7>BE03?[4,0 d BDC1;<7~8/0AP

�JH9?D;>.J;<79� @0�0PHn0�=<BG� È0�BE?D0 � 2��<0 d .J2�8/?D;>CQBE0�H92L? BH92L?[2L.A. �0�.10 �2AC � 0�B_45;>CAB A2

.�� CJ7>BD0L?[450 d BDCJ;<7~8/0AP

�JH9?D;>.J;<79� @0�0�Hn0�=<BG� È0�BE?D0 � 2��<0 d .J28/?D;>CQBE0+H92L?CH/2�?[23.N. �03.J0 �2

ABb�';<7>BE?D0L?��<=n0).{� 23CJ?D0�8>=�BE?[CJ2L79��.10

[email protected]�KO;L:<037/790M�.@0L;<K4@0�BE?DCJ�<=n0�8/0L42LCN?D0L4s89034!BE?DCN2�79��.J0L4

A1BC0�B

A2BCb

� � �Þ � �På b�¼nX�Æ3V�Æ�É�½ËäVYÓLX0¿LÇLÓ�Ï ��Æ�Æ�É�×GÆzÅ_Æ�ÒfË�X�ÆqË ½ÓLX�Ê�Ç>Ë �G]���Ï>Z�ÊLË�X[ÉGZ�Ì'½Ë�Õ9Ë �<Ó�\N\JÓ<Õ9ÆL\AZ�Õ�]�<Ó3\A\1Ó>Õ9ÆL\NZ{Õ�]���ÉGVYÓ3Ð�Ï�Ë>b

�@7�89;<7/790~=97T�>=92�8<?DCA.J2�B �0L?D0~CJ7/4 d ?DCNH<BDCJI<.J0 d ;<7��<0G �0O8/;<7>B�.1034P.1;<79��=n0�=9?[4�8/0L4d'È;�B @0L464,;<7>Bm � n � p

0�Bq � H9?[CJ4)8/2L7/4 d 0�B�;<?D8/?D0/bä�3CW.1034~8>CJ2��3;<7/2L.J0L46KO0L4G=9?D0L7>B d0�B

d′ � KR;<7>BE?D03?��>=n0 √mp + nq ≤ 1

2(d + d′)

b b b b b b b b b b b b b b b b b b b b b b b b b b b b b b b b b b b b b b b b b b b b b b b b b b b b b b b b b b b b b b b b bÞ �Pæ+à b ¼nXGÆ3V�Æ�É�ËLÕRÑ�Ø�ËÝZ1Í8½Ó�\qØ�Æ�Ë�Ï�× �¡Ë�]8Æ�X[ÓLÉ�Æ<Ê�]8Å�Æ�ÒOÓ�Ï>Z1Ó/b

�L0�BABC

I/0s2L7�:+2 d =<BD0��G2�79��.J0�8PBE?[CJ2L79��.10/b!�L0�Br2L798

RI/0�BGF/0@CN7/?[2�8>CN=94_2L798d CJ? d =9KM?[2�8>CN=94 � ?D0L4�Hn0 d BDCQ�<0�.Q: � 2L798·.10�B

sI/0´BGF/0 4,03KMCNHn03?DCNKR0�BE03?

�BGF<2�B CN4 �

s = 1

2(a + b + c)

b��L0�Bma

I90RBGF/0f.J0L79�¡BGF';>g�BGF/0fKR0L8<CN2�7�g1?D;<KABE;

BC �2�798f.10�Bwa

I90~BGF/0�.J0L79�¡BGF�;>g�BGF/0�CJ7>BD0L?D7/2L.WI>CJ450 d BD;<?�;>g∠A

g1?D;<KABE;RBGF/0�4�CJ890

BCb � 0�890��n790

mb � mc � wb2�798

wc4GCJK�CN.N2�?[.Q:ub�-9?D;L�<0PBGF<23B

�12 � 3s2 − r2 − 4Rr

8sRr≤

� ����� � �

ma

awa

≤ s2 − r2 − 4Rr

7sRr�

�1I � 3

4≤

� ����� � �

m2a

b2 + c2≤ 4R + r

4R

bÞ �Pæ@½ b ¼nXGÆ3V�Æ�É�ËLÕRÑ�Ø�¿�Æ/Ë �6Æ/ÈsÓ�XGÕ�]8¼nÆ�X,ÌJÓ3\{Ë�É�]�¾qË�]�� = UPb��ã{7/4�H/CN?D0�8OIL:�-9?D;/I<.J0LK ��� bq¥ � ��2�BGFubn¨_23��0�B[BE0 ��� � £ �� � � �" : � � H�b ��� � b ��L0�B

x ∈(

0, π2

) bW��F/;�}�BGF<2�B ��12 � [2 + cos x

3

]

[

2(1 − cos x)

x2

]

>1 + cos x

2�

�1I � 2 + cos x

3<

1 + cos x

2<

2(1 − cos x)

x2

bÞ �Pæ å b ¼nXGÆ3V�Æ�É�ËLÕRÑ�Ø ��Z�ÌDǧÁ>ÊLÆ�Ï/Ó�X¡Ç3]8Æ_Ç>×DÍ9ÓLX�Ë¡É3Ì1]�Å_Æ�ÒOÓLÏ<ZJÓ9b

�L0�BABCD

I90�2�H/2�?[23.N.J03.J;/��?[2�K�bW¦s4GCJ79��;<7<.Q:~2L7�=97/KM2�?E��0�8�4GBE?[23C1�3F�BD0�8/�30 ��n798M2�Hn;>CN7>BM;<7

AB4�= d F)BGF<2�B

AM = 1

5AB

bÞ �PæPæ b�¼nX�Æ3V�Æ�É�Ë�Õ Ñ�Ø ËÝZ�0>Í/Ó<Ë,\�Ä�Æ9Ì�×DÍ�Ë�Ì{Æ<Ê�] ËâË�ÒfÆ�X[ZJÓ3\ ��Ï>Z�ÊLË�X[ÉGZ�Ì1Ø�Æ�ξ�Ë�È@Î�ÆnÇ�Ï�Õ�\1ÓLÏYÕ�] = ÌDÔ�¿�Æ�Í/Ï�É É�]�¾ �/b

;<7/4GC18/0L?679;<79�GCN7>BE03?[4,0 d BDCJ79�f2L798 79;<79� d ;<79��?[=n0L7>B d CN? d .1034Γ1

2L798Γ2

}+CQBGFd 037>BE?D0L4O1

2�798O2 � ?D0345H90 d BDC �<03. :�b��L0�B Q

I90~BGF/0OHn;>CN7>B�;>gWCJ7>BD0L?[450 d BDCJ;<7Ý;>g_BGF/0B�}6; d ;<KMKR;<7�BD2L79�30L7>B[4 � t12�798

t2 � }6F>C d FÝ89;T79;�B+CN7>BE03?[4,0 d BPBGF/0R.NCN790�4,0L��KR037>BO1O2

b c�d ;<KMKR;<7�BD2L79�30L7>B � tc � }6F>C d F6CJ7>BD0L?[450 d B[4WBGF/0P4,0L��KR037>B O1O2

KO0�0�BD4WBGF/0BD2L79�30L7>B[4t12�798

t22�B

E1

2�798E2 � ?D0L4�Hn0 d BDCQ�<0�.Q:ub�L0�B

PI/0~BGF/0MKMCJ89�GHn;>CN7>B�;>g�BGF/0�.NCN790M4,0L��KR037>B

O1O2

b�-9?D;3�<0~BGF<23BP � Q �

E1 � 2�798 E2

2L?D0 d ;<7 d : d .AC d b

� � �Þ �Pæ � b ¼nXGÆ3V�Æ�É�ËLÕRÑ�Ø �<Ó3É�ZA\�Ë¡Â��� X,Ì�Æ�Ó���Ë¡]���Ï>Z�ÊLË�X[ÉGZ�Ì1Ø�Æ�Î�¼/\�ÆLZ{Ë�É�ÌJZJ]�Å�Æ�ÒOÓ�Ï>Z1Ó/b

�L0�Ba1 � a2 � . . . � an

I/0�H9;<4�CQBDCQ�<0+?D032L.n7<=9KOI903?[4 � 2L798�.J0�B

En =

n∑

i=1

(n−1∑

j=0

aji

)−1 b

ãqgr = n

√a1a2 · · · an ≥ 1 � H9?D;3�<0�BGF<23B En ≥ n

(n−1∑

j=0

rj

)−1 gq;<? ��12 � n = 2 � �qI � n = 3 � � d � ? n ≥ 4

bÞ �Pæ�� b ¼nXGÆ3V�Æ�É�ËLÕRÑ�Ø �<Ó3É�ZA\�Ë¡Â��� X,Ì�Æ�Ó���Ë¡]���Ï>Z�ÊLË�X[ÉGZ�Ì1Ø�Æ�Î�¼/\�ÆLZ{Ë�É�ÌJZJ]�Å�Æ�ÒOÓ�Ï>Z1Ó/b

�L0�Ba1 � a2 � . . . � an

I/0�H9;<4�CQBDCQ�<0+?D032L.n7<=9KOI903?[4 � 2L798�.J0�BEn =

1 + a1a2

1 + a1

+1 + a2a3

1 + a2

+ · · · +1 + ana1

1 + an

b�L0�B

r = n√

a1a2 · · · an ≥ 1 � b�12 � -9?D;3�<0�BGF<2�B En ≥ n(1 + r2)

1 + r

gq;<?n = 3

2L798n = 4

b�1I � ? -9?D;3�<06;<?!8>CJ4�H9?D;3�<0�BGF<23B

En ≥ n(1 + r2)

1 + r

gq;<?n = 5

bÞ �Pæ�� b ¼nXGÆ3V�Æ�É�ËLÕRÑ�Ø �<Ó3É�ZA\�Ë¡Â��� X,Ì�Æ�Ó���Ë¡]���Ï>Z�ÊLË�X[ÉGZ�Ì1Ø�Æ�Î�¼/\�ÆLZ{Ë�É�ÌJZJ]�Å�Æ�ÒOÓ�Ï>Z1Ó/b

�L0�Ba � b � c � d � 2L798 r

I/0sH9;<4�CQBDCQ�<0@?D0L23.�7<=9KOI903?[4_4�= d F�BGF<23Br =

4√

abcd ≥ 1b-9?D;L�<0PBGF<23B

1

(1 + a)2+

1

(1 + b)2+

1

(1 + c)2+

1

(1 + d)2≥ 4

(1 + r)2b

Þ � � � b ¼nXGÆ3V�Æ�É�ËLÕRÑ�Ø ��Z�ÌDǧÁ>ÊLÆ�Ï/Ó�X¡Ç3]8Æ_Ç>×DÍ9ÓLX�Ë¡É3Ì1]�Å_Æ�ÒOÓLÏ<ZJÓ9bãqg

m2L798

n2�?D0~H9;<4�CQBDCQ�<06CN7>BE0L�30L?[4P4G= d FOBGF<2�B

m ≥ n � 2L798RCNg a � b � c > 0 �H9?D;3�<0�BGF<23Bam

bm + cm+

bm

cm + am+

cm

am + bm≥ an

bn + cn+

bn

cn + an+

cn

an + bn

bÞ � � Û b ¼nXGÆ3V�Æ�É�ËLÕRÑ�Ø�ËÝZ�×DÍ�Ë¡\�Æ�Ó/ÌJÓ3ZA\N\{Ë�]8Å�ÆnÇ<Ë�Ï/] ��XDÓLÏY×EË>b

Ü<;<?a � b � c ∈ (0, 1) � �9798�BGF/0�.1032�4EB!=9H/Hn03?sI/;>=9798R2L798�BGF/0)��?D0L2�BE034GB!.1;�}603?I9;>=9798O;>ga + b + c + abc � 4G=nI�;{0 d BWBE;)BGF/0 d ;<7/4EBE?[2LCN7>B ab + bc + ca = 1

b

� � �Þ � � Þ b ¼nXGÆ3V�Æ�É�ËLÕRÑ�Ø �<Ó3É�ZA\�Ë¡Â��� X,Ì�Æ�Ó���Ë¡]���Ï>Z�ÊLË�X[ÉGZ�Ì1Ø�Æ�Î�¼/\�ÆLZ{Ë�É�ÌJZJ]�Å�Æ�ÒOÓ�Ï>Z1Ó/b�12 � -9?D;3�<0�BGF<2�BäCNg 0 ≤ λ ≤ 4 � BGF/0L7 � gq;<?ä2L.A.nHn;<4GCABDC �<0+?D0L23.�7<=9KOI903?[4 x � y � z � t �

(t2 + 1)(x3 + y3 + z3) + 3(1 − t2)xyz

≥ (1 + λt)(x2y + y2z + z2x) + (1 − λt)(xy2 + yz2 + zx2)b

�1I � Ü>;<? t = 1

4

2L798λ = 4 � BGF/0�2�I/;L�<0�CJ790L�<=923.NCQB�:MI90 d ;<KO0L4

17(x3 + y3 + z3) + 45xyz ≥ 32(x2y + y2z + z2x)b

Ü�CJ798�2L.A.nHn;<4GCABDC �<0��>2L.A=n0L4s;>gδ4�= d F)BGF<2�BWBGF/0+CN790��>=92L.ACAB{:

x3 + y3 + z3 + 3δxyz ≥ (1 + δ)(x2y + y2z + z2x)F/;>.J8/4�gq;<?!23.N.x � y � z }6F>C d F�2L?D0 � �NC � H9;<4�CQBDCQ�<0+?D032L.n7<=9KRI/0L?[4 � �JCAC � 4GC18/0+.J0L79�¡BGF<4;>g�2+BE?[CJ2L79��.10/b

Þ � � à bM¼nXGÆ3V�Æ�É�ËLÕ Ñ�Ø ¿LÇ�ÓLÏ ��Æ�Æ�É�×EÆhÅ_Æ�ÒfË�X�Æ Ë ½Ó�X%Ê�Ç<Ë �G] �uÏ<Z{ÊLË�X�É�Z{Õ>Ó<Õ Õ9Ë�<Ó3\A\1Ó>Õ9ÆL\NZ{Õ�] �<Ó�\N\JÓ<Õ9ÆL\AZ�Õ�] = VYÓ�Z1Ï

(Õ9Ë�Õ�Z{×[Ó<Ì�ËLÕ�Ì{Æ �3Æ�É,Í>Z�Æ = Ë¡ZJÒ�ZqØ3Ó

)b

�L0�BABC

I90�2�79;<79�ECJ45;<4 d 03.J0L4~?[C1�3F�B+BE?DCN2�79��.J0}+CABGFf?[C1�3F�B�2�79��.J0M2�BA2�798

AC > ABbf�L0�B

DI/0BGF/0Rgq;/;�B�;>gWBGF/0�23.ABDCQBD=n890Og1?D;<K

ABD;�BGF/0�4GC18/0

BCb�L0�B

GI90�BGF/06H9;>CJ7>B@;>gYCN7>BE03?[4,0 d BDC1;<7O;>g�BGF/0�.ACJ790

AD�q0G LBD0L798/0�8 � }+CQBGFOBGF/0~.ACJ7906BGF<?D;>=n�3F C}6F>C d FRCJ4PH/2�?E�2L.A.10�.<BE;

ABb��L0�B

EI/0PBGF/0�Hn;>CN7>Bä4�= d F6BGF<2�B

ACGECJ4~2�?D0 d B[2�79��.J0 � 2�798�.J0�B FI/0�BGF/0�H9;>CJ7>B64G= d FÝBGF<23B

BFGECN4P2�?D0 d BD2L79��.10/bM�L0�B

HI/0)BGF/0)CN7>BE03?[4,0 d BDC1;<7;>g

AG2�798

BFbs�L0�B

O1

I90PBGF/0+CJ7>BD0L?[450 d BDCJ;<7O;>gnBGF/08<CN2��3;<7/23.J4ä;>g<BGF/0P�>=92�8<?DCA.J2�BE03?[2L.CDHF � 2L798�.10�B O2I90�BGF/0�CJ7>BD0L?[450 d BDCJ;<7~;>g<BGF/0�8>CJ2��3;<7/2L.N4�;>g<BGF/0P�<=92�8/?[C1�.J2�BE03?[2L.

BDGEb .............................................................................................................................................................................................................................................

.

..

.

..

.

..

.

..

.

..

.

..

.

..

.

..

.

..

.

..

.

..

.

..

..

.

..

.

..

.

..

.

..

.

..

.

..

.

..

.

..

.

..

.

..

.

..

.

..

.

..

..

.

..

.

..

.

..

.

..

.

..

.

..

.

..

.

..

.

..

.

..

.

..

.

..

.

..

..

.

..

.

..

.

..

.

..

.

..

.

..

.

..

.

..

.

..

.

..

.

..

.

..

.

..

..

.

..

.

..

.

..

.

..

.

..

.

..

.

..

.

..

.

..

.

..

.

..

.

..

.

..

..

.

..

.

..

.

..

.

..

.

..

.

..

.

..

.

..

.

..

.

..

.

..

.

..

.

..

..

.

..

.

..

.

..

.

..

.

..

.

..

.

..

.

..

.

..

.

..

.

..

.

..

.

..

..

.

..

.

..

.

..

.

..

.

..

.

..

.

..

.

..

.

..

.

....................................................................................................................................................................................................................................................................................................................................................................................................................................................................................................................................................................................................................................................................................................................................................................................................................................................................................................................................................................................................................................................................................................................................................................................................................................................................................................................................................................................................................................................................................................................................................................................................................................................................................................................................................................................................................................................................................................................................................................................................................................

............................................................................................................................................................................................................................................................................................................................................................................................................................................................................................................................................................................................................................................................................

...........................................................................................................................

A

B

C

D

E

F

G

H

O1

O2

-9?D;L�<0PBGF<23BWBGF/0PBE?DCN2�79��.J0L4ABC � DFE � 2�798 DO1O2

2L?D0�4GCJK�CN.N2�?¡bÞ � � ½ bM¼nXGÆ3V�Æ�É�ËLÕ Ñ�Ø ¿LÇ�ÓLÏ ��Æ�Æ�É�×EÆhÅ_Æ�ÒfË�X�Æ Ë ½Ó�X%Ê�Ç<Ë �G] �uÏ<Z{ÊLË�X�É�Z{Õ>Ó<Õ Õ9Ë�<Ó3\A\1Ó>Õ9ÆL\NZ{Õ�] �<Ó�\N\JÓ<Õ9ÆL\AZ�Õ�] = VYÓ�Z1Ïub

�L0�BPI/0+2�7�:~Hn;>CN7>B!;<7)BGF/0�.NCN790

BCCJ7 4ABC

b��L0�BA1

I90PBGF/0PCJ7>BD0L?[450 d �BDC1;<7�;>gAP

�JHn;<4�4�CJI<. :�0� 3BE0379890L8 � }+CQBGF�BGF/0).ACJ790�BGF<?D;>=n�3F B}6F>C d FOCN4�H92L?[2L.A.10�.�BD;

AC � 2�798M.10�B A2

I/0�BGF/0�CJ7>BD0L?[450 d BDCJ;<7R;>gAP

�JHn;<4�4�CJI<. :R0� 3BE0379890L8 � }+CQBGF�BGF/0�.ACJ790BGF<?D;>=n�3FC}6F>C d F~CJ4@H/2�?[23.N.J03./BE;

ABb-9?D;L�<0�BGF<2�B�BGF/0O2L?D0L2�;>g 4ABCCN4�BGF/0R�30L;<KR0�BE?DC d KO0L2L7Ý;>g_BGF/0O2�?D032�4�;>g

4A1BC2L798 4A2BC

bÞ � �På bM¼nXGÆ3V�Æ�É�ËLÕ Ñ�Ø ¿LÇ�ÓLÏ ��Æ�Æ�É�×EÆhÅ_Æ�ÒfË�X�Æ Ë ½Ó�X%Ê�Ç<Ë �G] �uÏ<Z{ÊLË�X�É�Z{Õ>Ó<Õ Õ9Ë�<Ó3\A\1Ó>Õ9ÆL\NZ{Õ�] �<Ó�\N\JÓ<Õ9ÆL\AZ�Õ�] = VYÓ�Z1Ïub

¨_C �<0L7�2L7�CJ7/4 d ?DCJI90L8 d ;<7��<0G f�>=92�8<?DCA.J2�BE03?[2L.u}+CABGF�4GC18/0L4�;>g_.10379�¡BGFm � n � p �

q � BD2���0L7~CJ7M;<?D8903?!2L?D;>=9798~BGF/0��>=92�8<?DCA.J2�BE03?[2L. � 2�798M8>CJ2��3;<7/2L.N4s;>gn.10379�¡BGF d2�798

d′ �H9?D;3�<0�BGF<23B √mp + nq ≤ 1

2(d + d′)

b

� � �� � �� ����� � �

¾�Æ�VYXGÆ9ÑL\�Ë�Ò·ZJÉ�Ë3ÊLË�X+V�Ë�XEÒOÓLÏYË�ÏuÌJ\1Ø'×�\�Æ�É�Ë�Õ�Ô �LÍ�ËTËLÕ�Z�Ì{Æ�X�ZNÉMÓ3\{ÈsÓ�Ø3É~Vn\�Ë�ÓLÉ�Ë�ÕâÌ{Æ×GÆ�Ï<É�Z{Õ9Ë�XäÎ�Æ�X�VWÇ/ÑL\AZ�×�Ó/ÌJZ{Æ�ÏOÏ�Ë�ÈâÉ�ÆL\�Ç/ÌJZ{Æ�Ï/ÉPÆ�X!Ï�Ë�ÈáZJÏ/ÉGZJÐLÍuÌNÉ�Æ�Ï)VYÓ3É3ÌnVYX�Æ9ÑL\{Ë�ÒMÉ3Ô� 0�F<2��<0�8>CJ4 d ;3�<0L?D0L8RBGF<2�B�H/?D;/I<.J0LK �" ��� CN4�CJ89037>BDC d 2L.YBD;RH9?D;/I>.103K � � ��� � }6F/;<4504,;>.A=<BDC1;<7MF<[email protected]?D0L2�8�:MI/0�037MH/?DCN7>BE0L8� �����>� �>����¢ ��������� b ;<7/450��>=n0L7>BD. : � H9?D;/I>.103K��� ��� CJ4 d .J;<4,0L8 � 2L798~79;)45;>.N=<BDCJ;<7/4ägq;<?_CQB�}+CA.N.9I/0�2 dGd 0LH>BE0L8�b � 0P2�H9;>.1;/��CQ��0�gq;<?�BGF>CJ4;L�<03?[4�CJ�3F�B�bÞ �Pæ@½ b� �����>� �<�� � � ¼nX�Æ3V�Æ�É�Ë�ÕRÑ�Øu¼<Ó�Ï�Æ�É��nÔ �¡É5Ó<ÆnÇ3É5É�Æ�Ð3\{ÆnÇL]/U�ÌqÍYË�Ï/É�]��uX�ËLË�×EË>b

ãqga � b � c 2L?D0�BGF/0+4�CJ89034s;>gY2�7M2 d =<BE0+2�79��.J0�8~BE?[CJ2L79��.10 � H/?D;L�<0�BGF<2�B

� ����� � �

a2 + b2 − c2√

a2 − b2 + c2 ≤ ab + bc + cab

oGb = ÆL\[Ç<ÌJZ�Æ�ÏfÑ�ØuÂ/Í9X[ZNÉ3Ì{Æ3VYÍYË�X3¿LÔRÆ�XDÓ>Õ�\�Ë�Ø3]8Æ�X[ZNÉ3Ì{ÆL\J]��8Ääb�L0�B

x2 = b2 + c2 − a2 � y2 = c2 + a2 − b2 � 2�798 z2 = a2 + b2 − c2 �}6F/0L?D0x � y � z > 0

b¯��F/0L7a2 = 1

2(y2 + z2) � b2 = 1

2(z2 + x2) � 2L798

c2 = 1

2(x2 + y2)

b��YF/0���CQ�<037MCN790��>=92L.ACAB{:OI/0 d ;<KR0342(xy + yz + zx) ≤

� ����� � �

y2 + z2√

z2 + x2 �;<? ∑

� ����� � �(yz + zx) ≤

� ����� � �

y2 + z2√

z2 + x2 � � � �}6F>C d F~CJ4!BE?[=n0 � 4GCJ7 d 0 (y2 + z2)(z2 + x2) − (yz + zx)2 = (z2 − xy)2 ≥ 0

bß/�>=92L.ACAB{:�F/;>.18<4�CN7T� � � CNgW2�798�;<7<. :fCNg z2 = xy � x2 = yz � 2L798 y2 = zxb�YF/034,0 d ;<798>CABDCJ;<7/4s2�?D060L�<=/C �>2L.J0L7>BäBD;

x = y = zbs�+0L7 d 0 � 0L�<=923.NCQB�:OF/;>.18<4@CJ7�BGF/0H9?D;<H9;<4,0L8+CN790��>=92L.ACAB{:+CAg<2L7986;<7<. :�CNg

x = y = z �BGF<23BYCJ4 � CAg<2L7986;<7<. :�CNg a = b = c

bo�oEb = ÆL\�Ç/ÌJZ{Æ�ÏfÑ�ØRÅnÇ3Î�Ë,Z0ÁLÍ/Ó<Æ�]�É�ÌDÇ<Õ9Ë�ÏuÌ1] ��Æ�Ï¢Ë�ZA\N\NÉ�Â�ÆL\N\{Ë�Ð�Z1Ó<Ì�Ë¡ÇqÏ/É�ÌJZ�ÌDÇ<Ì�Ë¡] �LÆ�XGÆ�ÏuÌ{Æ�]�N¾)b

©<: 2L= d F�:�� 4Pã{790L�<=923.NCQB�: � gq;<?ä2L7�: x � y � z � w ≥ 0 � }606F<2��<0√

xy +√

zw ≤√

(x + z)(y + w)b

��037 d 0 �∑

� ����� � �

a2 + b2 − c2√

a2 − b2 + c2

=1

2

� ����� � �

(

a2 + b2 − c2√

a2 − b2 + c2

+√

c2 + a2 − b2√

c2 − a2 + b2

)

≤ 1

2

� ����� � �

(2a2)(2c2) =∑

� ����� � �

acb

� � £

o�o�oGb = ÆL\[Ç<ÌJZ�Æ�ÏfÑ�Ø ��ZEÁ3ÍYÆnÇ3]�¼nÆL\�0¢Â�Æ�ÒOÒ�ÇLÏ<Z�ÌqØuÂ�ÆL\N\{Ë�Ð>Ë�] ��Z1ÏYÌ�Ë�X ��Ó<ÊLË�Ï/] � �3]�� = UPb©<:~BGF/0 d ;<7��<0G �CQB�:M;>g √

x � }60�F<2��<01

2

(

a2 + b2 − c2 +√

a2 − b2 + c2)

(a2 + b2 − c2) + (a2 − b2 + c2)

2= a

b��037 d 0 � ∑

� ����� � �

√a2 + b2 − c2 ≤ a+b+c � }6F>C d F+I90 d ;<KO0L4uBGF/0�8/0L4GCJ?D0L8�CJ790L�<=923.NCQB�:=9Hn;<7M4,�>=92�?[CJ79�<b

23� ��� ��� �$+���- 587�2(9 �#2���� 2 ����A� '���� �����*A&E�2�A � �*2 ��� �� � � �C2�9 � ��2 �)2 � �E<A ���<�$+���� ���$�$7��� ��' � '���� + � A ��' � '���� + � A&>�����<� ' ' ��- � �����*� �#�#+�� ��' ������E � � � ><2���2�� � � � A:9;��!<���#A � ' ����� � E �����E ��9���� � A�� � ������! � � �<��!#��������� �#� '#�"��E�� � �$� �<�*A�� � �?� � �#A�� ;A � �*2 � ! "����� � ��9#��� � AC��� !<-������ A$������,�����%� �$���#� � '�� �F�#�,+ ��� ���,� 7 A ��' � '�B6'&�*�<� A���� A � �*2 �'� � �: �$ 2(9#� A������=� ' �,����A �(� A � �*2 �$ 2 ��� � � 9)��2 � � � A ����� ! � � ����� ��7�B6�<'����,!�B�A%�$�<����58�*!#� @�A12 ! ���$� � ' �*� ��9�9?2!� �?. ����2�� �#� ��A�F�#�,+ ��� ���,� 7:����2��,5����=� ' A � -8BC�����,���<A�2(> � � � �,+ $�2�����2 ��A��4��� � ���� � A�E �#� �<' �# � ������2 �:.-� ��9�����A� �#+���� A.� 23A � ��2 � � E��82�9#� ��� � E �/��A )�"�"� + + ��� + �!#��� �,�"@�� ���<�$+���� ���$�$7 A0� ' �#-���5?!���� A ;����B6' �?7 �><2�>1��� �-� � 9 � ��A4E ��' � @��,-8' A ���������� �2� � � � 93�;.4$6 ) ;A >?�$� � ' �F�#�,+ ��� ���,� 7 A � '.� � � '#-8' A:E�2�A � �*2 ��5�6� ��75 � ��)2(9���A;>;!#���<'�������� A;9;��B6' �<� ' ��'���-6����� ����� ��������.

2�5���!#� ��' � � ��� � ���&� !#58B��,�"�"��-���� �"!#� �$�����C! ����- �����&��' BC�&��� ' ��-8' ��- � !<58��� �,� !<� �,���<��� �$+���� � ��<� �,!<� �,������'<5��#+ �*A;�,� �<��� �����)%)�$���&� ���)2�� + 1�8�$���:9?!�' � �$�$7 '���-<;?����E<'�!#����7 � �4� ���:9?! ' � �,� 7?.

23�$������B6' ��@���- ���<'<�?� ��% �(B ! �,� � �?� 74������� �$+����4� ���:9?!�' � �$�$745872' ��- '<-�-

a2 + b2 + c2�"� 5��<� �

���,-�����A������� % �4�<5�� ' � �∑

� ����� � �

a2 + b2 − c2 ≤ a + b + c�

� �% � ���#% �,���K

'���-s-������<�"� ����� '�����' '���-6����� ����B�� ���� � BC���"��� ���;� ���4� � � ' � � �,�*A�������C!���� � �

a2 + b2 − c2 = 2ab cos C = 4(

12

ab sin C)

cot C = 4K cot C

'���- �,% �4��� B�� � ' ���$-������ �,� �,����AF������� ���:9?!�' � �$�$7 '#5��#+��4�,'��C5������ %(� �,�"�"���:'��√

cot A +√

cot B +√

cot C ≤ s

K

�*' ����! � �<5�� ' � ����-6� �����"� � �$�#%)� � � � ���:9?! ' � �,� 7 % �<�$����� �)��� ����� �#���(���<'��&����� ����� �������-6�����:=∑

� ����� � �a(b + c) cos A ≤ ab + bc + ca

�F��� � � � !#58��� �$��!#� �$��������� B�� � '��3�,� � ������� � ���<� �,!<� �,���/�?'#5��#+��*A��"� � �$�#% ��- 587 E<'<!<� �?7 � ���$���:9?!�' � �$�$7'���-4���������#% ���1� ��'��>� ���:9?! ' � �,� 7 A���� ' B6@8� �:����� '#5�� � � ����- �����3BC����� �<������� ' ������� ! �$��� ��'#�5�"���F' � �

n ≥ 2A

� ����� � �

n√

an + bn − cn n√

an − bn + cn ≤ ab + bc + ca,

�����#+��$-���-6� ����9?! '���� �$� �$��� ! ��-����(� ����� '<-��$�"' � �)' ��� ' � �?����� + ��� � '<� �,+ �#.

� � ¢Þ �Pæ å b �����>���3�" � � ¼nXGÆ3V�Æ�É�ËLÕ)Ñ�Ø �_ËLÆ�X[Ð>Ë�Æ�Ó�\�Æ�Ð�\�ÆnÇ3] = ��¾~Å �_É�ÈPË¡Ð<Æ�] �_É�ÈPË¡Ð<Æ�]¾~Å/]�� = UPb�3=9H9H9;<4,0sBGF<23B

D � E � F 2�?D0sBGF/0�H9;>CJ7>B[4ä2�Bu}6F>C d F+BGF/0 d ;<7 d =9?E?D037>B_.ACJ79034AD �

BE � CFKR0L0�B+BGF/0�4�CJ89034M;>gs2T��CQ�<037�BE?DCN2�79��.J0

ABCb �L0�B

p1

2L798p2

I/0fBGF/0Hn03?DCNKR0�BE03?[4M2L798δ1

2L798δ2

BGF/0�2L?D0L2L4M;>g 4ABC2�798 4DEF � ?D0L4�Hn0 d BDCQ�<0�.Q:ub-9?D;L�<0PBGF<23B

�J2 � 2p2 ≤ p1

CNgAD � BE � 2L798 CF

2�?D0+2�79��.J06I>CJ450 d BD;<?[4��qI � 2p2 ≤ p1

CNgAD � BE � 2L798 CF

2�?D0+2L.QBDCABD=n8/0L4�� d � 3p2 ≤ 2p1

gq;<?ä2L.A.D � E � F CAgY2�798M;<7<.Q:~CNg 4ABC

CJ4s0L�<=/CA.J2�BE03?[2L.��q8 � 4δ2 ≤ δ1

gq;<?!23.N.D � E � F 2�798�2�?EI<CQBE?[2�?{: 4ABC

b= ÆL\�Ç/ÌJZ{Æ�Ï�Ñ�Ø ��Z0ÁLÍ�ÆnÇL]�¼nÆL\ 0¢Â�Æ�ÒOÒ�Ç�Ï>Z�Ì1Ø2Â�ÆL\A\�Ë¡Ð<Ë¡] ��ZJÏuÌ{Ë�X ��Ó>ÊLË�Ï<] � �3]�� = UPb

�12 � �YF>CN4�CJ4 ¤�¦ � H9?D;/I>.103K 2502 ������� �3£9¢

������>� �>¢����>¢�£�� b

�qI � ��F/0M}6;<?D8>CJ79�O;>gäBGF/0RH/?D;/I<.J0LK 4G=n�3�30L4EBD4+BGF<2�B 4ABCCN462 d =<BE0/bT�J��F/0?D0L4G=/.ABM89;/034R79;�BMF/;>.18 gq;<?�79;<79��2 d =<BE0�BE?[CJ2L79��.1034Lb � ��F>CJ4OH9?D;/I>.103K CN4R��79;�}�7j2L4Ü>2���7/2�79;�� 4�-9?D;/I>.103K � ��� ;>gY2L.A.9BGF/0�BE?DCN2�79��.J0L4@CN7 d 4 d ?DCJI90L8MCJ7 4ABC � BGF/06;<?[BGF>C dBE?DCN2�79��.J0s2�B[BD23CJ7/4uBGF/0sK�CJ7<CNKM=9K Hn03?DCNKR0�BE03?�bWã{7�H/2�?�BDC d =/.N2�? � BGF/0�;<?[BGF>C d BE?[CJ2L79��.10sF<2�4Hn03?DCNKR0�BE03?!79;M��?D0L2�BE03?WBGF<2L7 1

2p1

b� d � ãqg 4ABC

CJ4+79;�BP0��>=/CN.N23BD0L?[23. � 2�798 BCCN4PBGF/0M.J;<79�30L4EBP4GC18/0 � BGF/0L7�}60d 2L7 d F/;/;<4,0

DBE;fI90)BGF/0�KMCJ89�GHn;>CN7>B�;>g

BCbO�L0�B

E → C2�798

F → BBD;��30�B

p2 → 2BC > 2

3p1

b ;<7��<03?[4,0�.Q: � 4G=9H9H9;<4,0 4ABC

CN4ä0L�<=/CA.J2�BE03?[2L.3}+CABGFAB = 1

b��L0�Bu = AF �

v = BD � 2�798 w = CEb���F/0L7

0 < u � v � w < 12L798

p2 = f(u, v, w) =∑

� ����� � �

u2 + (1 − w)2 − u(1 − w)b

�YF/037∂f

∂u=

2u − (1 − w)

2√

u2 + (1 − w)2 − u(1 − w)+

v − 2(1 − u)

2√

v2 + (1 − u)2 − v(1 − u)

b��0�B[BDCN79� ∂f

∂u= 0 � }606�30�B

[2u − (1 − w)]2 · [v2 + (1 − u)2 − v(1 − u)]

= [v − 2(1 − u)]2 · [u2 + (1 − w)2 − u(1 − w)] �}6F>C d F�?D0�8>= d 034�BD;u(u + w − 1)v2 = (1 − u)(1 − u − v)(1 − w)2

b � � ��YF>CN4 d 2�7�gJ=9?[BGF/03?�I90+KO2L7<CJH<=/.J2�BE0L8�CJ7>BD;uv(1 − u)(1 − w)(1 − v − w) = 0 � � � �

� � �

IL:�=94�CN79� 0,�>2<� 4s?D0�.J2�BDC1;<7uvw = (1 − u)(1 − v)(1 − w)

b � � ���/Õ�b_��0�0@BGF/0�79;�BE0sgq;>.N.J;�}+CJ79�!BGF/0�.NCN4GB�;>g/45;>.Q�<03?[4!I/03.J;�}Ob � �+0L7 d 0 � 1−v −w = 0 �BGF<23B)CN4 � v = 1 − w

2�798w = 1 − v

b �YF>=94 � u = 1

2

b ©<: 4[:/KMKR0�BE?{: � }60d ;<7 d .A=n890 (1

2, 1

2, 1

2

) CJ4sBGF/0�;<7<. : d ?DCQBDC d 2L.uHn;>CN7>B�;>gfb6�P;�BD06BGF<2�B

f(

1

2, 1

2, 1

2

)

= 3

2

bc .J45; � 2�4PBGF/0 d ;<7 d =9?D?D0L7>B�H9;>CJ7>BP2�H/H9?D;<2 d F/0L4�2O4�CJ890O;>g 4ABC � BGF/0MgJ=97 d BDCJ;<7 fd .1032�?[.Q:T2�H/H9?D;<2 d F/0L4�CAB[4)KM2� LCJK�=9K �>23.N=n0�;>g2b ��037 d 0 � 1

2p1 ≤ p2 ≤ 2

3p1

gq;<?79;<79�58/0��303790L?[2�BE0 4DEFb

�q8 � �YF>CN4sH/2�?�Bä[email protected];6}60�.N.���79;�}�7� ��� b����&P��#���$ ���'6 ��� �Mb�¥ �;<?E?[C10 � S�`�`��uX�Ë�Ó/Ì ¼nX�Æ9ÑL\{Ë�ÒMÉWÆ�Î �L\�Ë�ÒfË�ÏYÌ1ÓLXEØ�Ë'Ó/ÌqÍ�Ë�ÒOÓ<ÌJZ�×�É � ¥@;L�<03? � �" ��¢ �H9H�b ��¢� �>���>� b ��� �ebA�9b T .J2LKR�LCJ7 � ÇqÏuÌ{Ë�XEÏ/Ó/ÌJZ{Æ�Ï9Ó�\8Ë Ó<ÌqÍYË�ÒOÓ/ÌJZ{×[Ó�\ ��\1Ø�Ò~VnZ{×�É6S���������S�����a � � c_c ��� � : � H�b ��� 2L798MH/H�b � � � ��� �1H/?D;/I<.J0LKÙ¨sbJãGb £ 2�798~CAB[4@4,;>.A=<BDC1;<7/4 � b

23� ��� ��� �$+���- 587��� E � � � ><2���2�� � � � A69;��!<���#A � ' ����� � 9?2 � E,����E� > � � �� �� 93 �*2�� A��. >�. � B�� � �$� ����� '�� � A� <' � � '#-�� � �$-�A � ��' � � ��' �=� ��O P�,�O *�P�,�O !�P�� E � 9#���,�8 � � � 90�8.�>9?2���� � ��A(>�� � ���"� � A� � ��' �=� �0O *�P�,�O !�P���E ����� E ��9���� � A4� � ������!�� � �<��!#��������� �<� '<�,��E�� � �$� �<�*A � � �?� � �<A(� A � �*2 �#� � �3. � � �# � 9�A �� ' ��-����?� ' � � �$�*A82(> �,� ��9�9?2!� �?. ����2�� �#� ��A �F�#�,+ ��� ���,� 7 ���82��,5����=� ' A � -8BC�����"���#A#2�> � � ��� �*2 �:.:� ��9�����A-� �#+ ��� A,� 2�A � ��2 ��'��=� ��O *�P�,�O !�P��,� 2 � � � .�� ��2� � ���< ��< ��A 2(����������A�������������'��=� �@O *�P�,�O !�P�� � � � � 9��;."$6 ) ;A;>?�$� � ' �F�#�,+ ��� ���,� 7 A ��''� � � '#-8' A�E�2�A � �*2 ��'���-6����� ����� ���������.

��� �����6������+����#�,�������6���)� ��� ����'#-���� A)% � � �,+ �6�����6%('�7C���)�<5�� ' � �#� � �C�:9?!�'#� �$����O� P � ����B O��DP!���� � �/O� DP6=

uv2(1 − u − w) + (1 − u)(1 − u − v)(1 − w)2 = 0,

uv2(1 − u − w + uw) − u2v2w

+(1 − u)(1 − u − v + uv)(1 − w)2 − uv(1 − u)(1 − w)2 = 0,

uv2(1 − u)(1 − w) − u2v2w

+(1 − u)2(1 − v)(1 − w)2 − uv(1 − u)(1 − w)2 = 0,

uv2(1 − u)(1 − w) − uv(1 − u)(1 − v)(1 − w)

+ uvw(1 − u)(1 − w) − uv(1 − u)(1 − w)2 = 0,

uv(1 − u)(1 − w)[

v − (1 − v) + w − (1 − w)]

= 0,

2uv(1 − u)(1 − w)(v + w − 1) = 0�

��������������������������� �"!$#&%�'(%�)�*�+-,�.0/ %�)21�34%65�*7398�39.�: ;=<>:�?@: )2:�A�3B;DC7:�'>:�EF�G�H�IFJLKMHON�P Q�R S6TUPVIWTUXVR Y

Γ1 Z S\[ Γ2 ] J_^DR`XaPVS\R�Y l Z S\[ m Z IWRbR&c�JUR�IUS Z X\TUH�dedeH�SJ Z S\N�R S�J�Y

�gf ^DReXaPVS\RlJUH�h$T&^DR�Y

Γ1 Z S\[ Γ2 Z J A Z S\[ B ] IWR�Y2i\R�TWJWP Q�R�X j ]=Z S\[kJ_^DRXBPBS\R

mJWH�h$T&^DR Y

Γ1 Z S\[ Γ2 Z J C Z S\[ D ] IWR�Y2i\R�TWJWP Q�R�X j�ml h\i\i\H�Y7RnJ_^ Z J M

PVYmJ_^DRdoPV[\p_i$H�PBS�J(H�q�J_^DR@Y7R Nrd-R�S�J

AB ] Z S\[bJ_^ Z J P Z S\[ Q Z IWRsJ_^DR@Y7R T_H�S\[bPVS�JWR I�Y2R�TWJWPVH�SDYH�q

MC Z S\[ MDKnPaJ_^

Γ1 Z S\[ Γ2 ] IWR�Y2i\R�TWJWP Q�R�X j�

t\IWH Q�R�J_^ Z J A ] B ] P ]�Z S\[ Q Z IWR�T_H�S\Tuj�TUXBPVT �

� ���

oGb = ÆL\[Ç<ÌJZ�Æ�ÏTÑ�Ø�ÅnÇ3Î�Ë,Z�ÁLÍ/Ó<Æ�]�É�ÌDÇ<Õ9Ë�ÏuÌ1] �PÆ�Ï ËÝZN\A\JÉ¡Â�ÆL\A\�Ë¡Ð3ZJÓ/Ì{ËuÇ{Ï<É3ÌJZ�ÌDÇ/Ì{Ë�] �LÆ�XGÆ�ÏuÌ{Æ�]�N¾)b

�3CJ7 d 0MA2 = MB2 � }60�4,0L0sBGF<2�B M

CN4!;<7+BGF/0�?[2�8>C d 2L.<2¡ �CN4ä;>gΓ1

2�798Γ2

bã1B�gq;>.A.1;�}�4�BGF<2�BMP · MC = MQ · MD � 4,;fBGF<2�B CDQP

CJ4 d : d .AC d�F/0L7 d 0 �

∠MPQ = ∠CDQb@�L0�B

O1

2L798O2

890379;�BE0PBGF/0 d 037>BE?D0L4s;>gΓ1

2�798Γ2 � ?D0345H90 d �BDCQ�<0�.Q:ub��YF/037�BGF<?D;>=n�3FM45;<KR0+2L79��.10 d F<2�4GCJ79� � }60�F<2��<0

∠APQ + ∠QBA = ∠APM + ∠MPQ + ∠QBA

= 180◦ − ∠CPA + ∠CDQ + ∠QBA

= 1

2∠AO1C + ∠DBQ + ∠QBA

= ∠AO1O2 + ∠DBA

= ∠AO1O2 + ∠O1O2B = 180◦ �2�798~BGF/0+?D0L4G=/.ABägq;>.A.1;�}�43bo�oEb = ÆL\�Ç/ÌJZ{Æ�Ï�Ñ�Ø �@Ó�Ï>Z�Ë,\�Å�Ë¡ZNÉ �G] �/ZJÏY×EË¡\A\�Ë¡É5] ��X[Ó�Ï�×GË�b��/Õ�Z�Ì{Æ�X�É É�×EÆ�ÒOÒfË�ÏYÌDÔW��F/0+gq;>.N.J;�}+CJ79�PH9?D;/;>g�=9450L4ä�<0L?{:MK�= d F~BGF/0�4�2�KO0�2L?D��=9KR037>B2�4�CJ7���;>.A=<BDC1;<7'ã � 0G  d 03H<BPBGF<2�B�F/0L?D0OBGF/0�2L79��.103462�?D0f8>CJ?D0 d BE0L8�b'ã1B�CJ4�CN7>BE03?D0L4EBDCJ79�BE; d ;<KOH/2�?D0OBGF/0O=94,0�;>g!=9798<CN?D0 d BD0�8Ý2�798�8>CJ?D0 d BE0L8Ý2�79��.J0L43b Ü>;<?PBGF/0O=9798<CN?D0 d BD0�82�79��.J0L4

∠XY Z2�798

∠XY ′ZCN7/4 d ?[C1I/0�8jCN7 2 d CJ? d .J0 � ∠XY Z = ∠XY ′Z

;<?∠XY Z = 180◦ − ∠XY ′Z

2 dGd ;<?D8<CN79�R2�4Y2L798

Y ′ .NCJ0f;<7TBGF/0�452LKR0�2L? d ;<?;<7OBGF/0~;<H/Hn;<4GCABD0)2L? d 4P8/0�BD0L?DKMCN790�8�IL:X2�798

Zb�Ü<;<?äBGF/0~8<CN?D0 d BD0�8R2�79��.J0L4�g1?D;<K

Y XBD;

Y Z2L798fg1?D;<K

Y ′XBD;

Y ′Z}60MF<2��<0

(Y X, Y Z) = (Y ′X, Y ′Z)� CN7¤Y03CN4G�¡� 4s79;�B[23BDCJ;<7 � gq;<?ä2�7�:~gq;>=9?!Hn;>CN7>BD4

X � Y � Y ′ � Z ;<7M2 d CN? d .10/b �Å_Ë�ÒOÓ�X%Ê�Ç<Ë¡ÉLÔ� b

MP · MC = MA2 = MB2 = MQ · MD � 89;<7 d .1034)�>=923BE?D0RHn;>CN7>BD4C � P � Q � D 4,;<7>B d ; d : d .NCJ�<=n034Lb

� b)-<;>=9?_8/0L4W?[2LCN4,;<7/4ä890�4[:/K4@0�BE?[C10��J.N2�8/?D;>CQBE0�89034 d 0L7>BE?D034O1O2

034GB�K4@0�8>CJ2�BE?DC d 0d ;<KMKM=9790 � .1034 d ;<?D8/0L4 AC0�B

BD45;<7>B!H/2�?[23.N. �03.J0L43b

� b �@7M79;�BE03?[2(d, d′)

.J2�KO0L4G=9?D0�KR;/8>=/.1;π890�.�� 2�79��.J068/0L4s8/?D;>CQBE034

d0�B

d′ bß<7)�<03?[BD=R89034�8>C�� @0L?[2L7>BE034 d ; d : d .AC d CQB @0L4@;<7MHn0�=<B�@0 d ?DCN?D0 �

(PA, PQ) = (PA, PC) + (PC, PQ)

= (CA, CD) + (DC, DQ)

= (CA, DQ) = (DB, DQ) = (BA, BQ) �8Y� ; �=M.N2 d ; d : d .AC d CQB @0�8/0L4@Hn;>CN7>BD4A � B � P � Q bo�o�oGb = ÆL\�Ç/ÌJZ{Æ�Ï Ñ�Ø �9× 0�Ó�XGÕ = V�Ë�×DÍYÌ1] �@Ì[Ì�Æ �GÊLÆ�Ï � �äÇ<Ë�X[Z�× 0<Ë �uÏ<Z{ÊLË�X�É�Z�ÌqØ3]Ë Ó3Ð<Õ9Ë�ÑnÇ�X�ÐL]

�_Ë�XEÒOÓLÏnØ/b�L0�B

iI90)BGF/0�CJ7��<03?[4�CJ;<7R}+CABGF�?D0L4�Hn0 d B�BD;RBGF/0 d CN? d .10

Γ}+CABGF d 0L7>BE?D0

M2L798?[2�8>CN=94

r = MA = MBb~�YF/037

i(A) = A2L798

i(B) = Bbf©<:RBGF/0O��0 d 2�7>B ��/2L79�30L7>B���F/0�;<?D03K � }60M8/0�8>= d 0~BGF<2�B r2 = MP · MC

2L798r2 = MQ · MD

b

� � �

�YF/03?D03gq;<?D0 � i(P ) = C2�798

i(Q) = Db�©9=<B

A � B � C � 2�798 D2�?D0 d ;<7 d : d .NC d � 4GCJ7 d 0

ABCDCJ4�2L7�CN4,;<4 d 0�.1034äBE?[2�H90���;>CJ86}+CQBGF

AC ‖ BD2L798

AB = CDb��YF>CN4 d CN? d .10d 2L7/79;�B�H92L454�BGF<?D;>=n�3FfBGF/0OKMCJ89�GHn;>CN7>B

M;>g

AB �CAB�CJ4PBGF/0L?D0�gq;<?D0OCN7��<0L?�BE0L8ÝIL:

iCJ7>BD;�2 d CN? d .10PBGF<?D;>=n�3FA � B � P � 2L798 Q � 2�4@?D0L�<=/CN?D0�8ub

23� ������� �,+ ��- 5870��� � � �&2�� � � ��2 � E� # 2 � A E<' � ' � ��!#��� ' A(� ' � �,�����,' A � ��' � � � �� �� � � �2�9 � ��2 �)2 � �E<A���<�$+���� ���$�$7���� ��' � '���� + � A ��' � '���� + � A3>�����<� ':' ��- �4�����*� �<�#+*� �<' �#����E � � � ><2���2�� � � � A9;��!<���#A � '������ � 9?2 � E,����E� 1> � � �< ��&93 ��2�� ;A���. >�. � B�� � �,� ����� ' � � A3' ��-���2�9 �� 2 2 �<E � � �5� � �� � � � 7CE �)2�� 9�93 ;A���. >�. �<��� �� �$-���E<'���� A� �' � � '<-�� � �,-�A � ��' � � �

(>�*� �,�<�3' � ��� � !#58B��,�"�"��- '6����������-��� �"!#� �$���

)�1E � 9#���,�8 � � � 9%��. >9?2���� � ��A >�� � ���,� � A � � �� # 1����� � �;9#���#� A&��� !<-������ A�$������,�����

� �,� �<� ��' � �F�#�,+ ��� ���,� 7 A ��' � ' B6'<�*�<� A ��� A�� ��2 ���� �� � 3. � � �! � 9�A �� '���-��'�?� ' � � �$�*A�2�> � $�2 �5� � � 9��2 � � � A ����� ! � � ����� ��7�B6�<'����,!�B�A(�$�<����58�*!#� @�A(2�!���� � � ' ���� � � �<E � �< �#> � 9 �A(��� !<-������ A �4� % �?����@�F�#�,+ ��� ���,� 7 A��'��A�� �*2 �;2 ��� 9 � � �5����� ���A)����!#-������ A4E��<� ���� ���<�$���0�"��� 2(-�+�' ������BC����� ��� �<�,�,�������'���-�2 �=� A �4� % �?����@�A �'��A � ��2 ��� . ��. ��� ��� � ��A 7 ' �,�"5���B6BC�*� A(� ��� �4��� ����� � '���-8� ��� � � � 9 �;. $� ( A>?�,� � ' �F�#�,+ ��� ���,� 7 A ��'/� � � '<-8' A)E�23A� ��2 � ���17 � ��A!��� � @ E���B6B ! �<�$�$7&E�� � �,� �#�*A�$ � ���"����� '<+ ���<A � A� �*2 ��'���- � ��������� ���������.

�������b�`���������O� ����� � !$#_%�'(%�)r* +�,�.M!$#&%�'(%�)�*�+�,�.��sE�� #_*r:�)`!���b:���U3�'(%� 9: A�3V) :�E +!�:��� ���3���;��� 4%�#�3u+�:����� 9: E��V3����FE�3�� *r#L)&3�� .�; � %�L:"!>:�4%�ED;���#�;$�(5%���

& P Q�R S�JLKMHoi$H�PBS�JWYB Z S\[ C ](' S\[oJ_^DRnX9HDTUh\Y�H�q J_^DRMi$H�PBS�J

AY_h$T&^�J_^ Z J J_^DRT_R�S�JUIWRMH�q$J_^DR�S�PBS\R�p_i$H�PBS�JsTUPVIWTUXVRMH�q 4ABC

XaP9R�Y@H�S6J_^DRnXaPVS\RBC

) �$5�%� ��(�V3u%�Eg,�.�Cr%D*% F5*�W1( 4%�)�,D*�#�+�; ),�-��+\*rE.�9;0/�*�?1�D%�#UA2�FE�3�� *r#L)&3�� .�;0/3�D;$�(5%���44S�JUIWHD[�h$TUR Z T_HDH�IW[�PVS Z JURoY�j�Y_JWR d KnP�J_^

B =(

−1

2a, 0

) Z S\[ C =(

1

2a, 0

) ]KM^DR IWRaPVY`J_^DRoXVR S\N J_^kH�q Y2R�NrdeR S�J

BC ]FZ S\[gXVRrJA

^ Z Q�R-TUHDH�IW[�PBS Z JWR Y (X, Y )�

4 qD ] E ]DZ S\[ F Z IWRnJ_^DRMd�P9[Dp&i\H�PVS�J�Y�H�qFY&PV[\R�Y

BC ] AC ]DZ S\[ AB ] IWR Y&i$R TWJWP�Q�RrX�j ]J_^DR SD = (0, 0) ] E =

(

1

4a + 1

2X, 1

2Y) ] Z S\[ F =

(

−1

4a + 1

2X, 1

2Y)

�ef ^DRS�PVS\R p&i\H�PVS�JbTUPVIWTUXVR�PVY�J_^DReTUPVIWTUh\deTUPVIWTUXVR-H�q 4DEF 5 J_^�h\Y ] P�JWY�TUR S�JUIWR

Ndoh\YUJ76\R

H�S6J_^DR�i\R IWi$R�S\[�PVTUhDX Z I86�PBY7R TWJUH�I H�q EF ] KM^�P9T&^6PBY J_^DRbXBPBS\Rx = 1

2X

� 4 qN

PVY Z XBY7HH�SoJ_^DRMXaPVS\RBC 9 KM^�P9T&^-^ Z Y�R�:�h Z JWP9H�S y = 0; ] J_^DR�S N

d�h\Y_J@^ Z Q�R6TUHDH�IW[�PBS Z JWR Y(

1

2X, 0

)

�(f ^DR�S ] Y_PVS\TUR ND = NE Z IWR�I Z [�PaP>H�q\J_^DR�S�PVS\R p&i\H�PVS�JsTUPBIWTUX9R ] KMR�^ Z Q�R

ND2 =(

1

2X)2

= NE2 =(

1

4a)2

+(

1

2Y)2

<�R�S\T_R ] X Z S\[ YY Z JWPVY_qaj

x2 − y2 = 1

4a2

= H�S�Q�R�I�Y7RrX�j ] q H�I Z S�j�i\H�Y&P�JWP9H�S�H�q\Q�R�I�JWR&cA

KM^DH�Y2R�TUHDH�IW[�PBS Z JWR YsY Z JWPVY_qajMJ_^�PVYR>:�h Z JWPVH�S �@?D+�� � R_c�T_R�i�J

A = BH�I

A = C ] KM^DR IWRnJ_^DRnJUI�P Z S\N�X9R�PBY`[DR�N�R�S\R I Z JUR�]

N =(

1

2X, 0

) PVYnJ_^DR TUR S�JUIWRgH�q J_^DR0S�PVS\R p&i\H�PVS�J�TUPVIWTUXVR ] Y&PBS\T_R-P�JnPBY6R�:�hDPV[�PBY_J Z S�Jq9IWH�dD ] E ]mZ S\[ F

� f ^DR IWRrq H�IWR ] J_^DR X9HDTUh\YeH�qmJ_^DRgQ�R�I�JWR&cA

PBYeN�P�Q�R�SA6 j J_^DRR>:�h Z JWPVH�S Z 6\H�Q�R ] KM^�P9T&^n[DR Y2T_IWP�6\R�Y Z IWR�TWJ Z S\N�hDX Z I ^�jDi\R IB6\H�X Z KnPaJ_^bd Z�C H�I Z c PVY BC�@?D+�� � KnPaJ_^OJ_^DRni$H�PBS�JWY

B Z S\[ CIWR�d-H�Q�R�[ ���

� �

o�oEb = ÆL\�Ç/ÌJZ{Æ�Ï�Ñ�Ø ��Z0ÁLÍ�ÆnÇL]�¼nÆL\ 0¢Â�Æ�ÒOÒ�Ç�Ï>Z�Ì1Ø2Â�ÆL\A\�Ë¡Ð<Ë¡] ��ZJÏuÌ{Ë�X ��Ó>ÊLË�Ï<] � �3]�� = UPb©<:fBGF/0M4,;>.A=<BDC1;<7�2L798 d ;<KMKR037>B�BE;�H/?D;/I<.J0LK � � �>¢ �����>� �Y��£� �����¢�� � � BGF/07<CJ790L��H9;>CJ7>B d 037>BE?D0�.NCJ0L4f;<7

BCCNg+2L798 ;<7<.Q:âCNg |B − C| = π/2

b ;<7/4GC18/0L?C = B + π/2

bW�3=9H9H9;<4,0�BGF<23BB = (−1, 0) � C = (1, 0) � 2L798 A = (x, y) � }+CABGF

x > 1b��L0�B

P = (x, 0)2L798

Q = (2x − 1, 0)bs��F/0L7

B = ∠CAP = ∠PAQ �BGF>=94 �4GCJ7 d 0A + 2B = π/2

� � AB ⊥ AQb!��037 d 0 � y

x + 1=

x − 1

y�BGF<23BäCN4 �

x2 − y2 = 1b

� 0@�30�B9BGF/0@;�BGF/0L?�I/?[2L7 d F(x < −1)

;>gLBGF/0@F�:<Hn03?GI/;>.J2ä}6F/037B = C+π/2

bã{7 d ;<7 d .A=94�CJ;<7 � BGF/0�.J; d =94ä;>g A CN4�BGF/0�?D0 d BD2L79��=/.J2L?_F�:<Hn03?GI/;>.J2 d 037>BE?D0�8�23BYBGF/0�KMCJ89�Hn;>CN7>B�;>gBC � }+CQBGF)BGF/0P�<03?[BDC d 034 B � C 0G  d .N=n8/0�8ub

23� ��� ��� �$+���-�587 ��� E � � ��><2���2�� � � � A 9;��!<���#A � ' ����� � E � 9#���,�8 � � � 9 �8.C>9?2���� � ��A>�� � ���,� � A � � � E �'��� E �;9��5��� A�� � ������!�� � �<��!#��������� �<� '<�,��E�� � �,� �#�*A�� � �?� � �<A � ;A � ��2 ��� � �3. � � �# � 9�A��� ' ��-����?� ' � � �,�*A 2�> � $�2 �5� � � 9(��2 � � � A �F��� ! � � ����� �87 B6��' ���"! B�A-�$�<����58�*!#� @�A�2�!���� � � ' �� . �8. ��� ��� � ��A�7 ' �$� 5���B6BC�*� A������ �4��������� � ' ��-8� � � E��82�9#� �5� � E �2�=A� )�"�"� + + ��� + �!#��� �,�"@�� ���<�$+���� ���$�$7 A� ' �#-���5?!���� A;����B6'���7 ��� � � � 9 �;. $6 ) ;A)>?�,� � ' �F�#�,+ ��� ���,� 7 A � ' � � � '#-8' A�E�23A � �*2 ��� � � �#7 �)2 A����!#-������ A4� ����� � � � �6E�� � �,� � � '#�"� � ����� �,� !<�,�*A'� ���������,� A � �(�5�6� ��75 � ��)2(9���A�>;!#���<'�������� A 9;��B6' �<� ' �'���- � ��������� �������� � .

>�� '<-��$��7 ����B6'���@<��- ���<'<�4'��&' ���������:9?!#������� ����� �<� � �����<5��,��B�A6% � �<5�� ' � �1����� ��7 ����=5�� � �,�'��<' �$� ��!#� ���6�����0� '�B�� � � '���� ����������B '#5���!<�:'�� � �$��� � �����"� �,5���- � � ����B��,�,� ���,�,���&= $ �����

BC� � �����

-�� ' BC���"��� ���)'&����B��,�,� ���,�,� ������� ' � �#� � �AA � �����

B + C = π/2� % �����

BC� �������6B6' ����� '���� �6���

'4������� '�� ��! � ' � �?7 ����=5�� � ':������� ' � �#� � �AA������� |B − C| = π/2

.

��������� ��������� �k� ����� !$#&%�'(%�)�*�+ ,�.���� C�� 5 8g*�*�E\A�;�� :� ��,D%�8e8g* V; � 1 */�* � 1 *r#� V:�E +�) �44S 4ABC ] KMR�^ Z Q�R c4 = a4 + b4

9 Z ; l ^DH�K J_^ Z J 4ABCPBY Z TUh�JWR Z S\N�X9R [

9 6 ;�� R�JUR�IUd�PVS\R�J_^DR�I Z S\N�RMH�q∠ACB

9 T ; ? <nH�K T Z SOKMR�N�R S\R�I Z XaP�&R�JUHcn = an + bn

5�%� ��V34%�Eg,�.��D1�3a' ��� # �V3V)&;�� 3B)2)�%��#�3 5�%�(� 1>*�#UEe5(�9:(�u*�� %� � 4* +�*�;�Cr%�' a39E�;����=;$� 5 ���l h\i\i\H�Y7R�J_^ Z J cn = an + bn KnP�J_^

n > 2 9 n IWR Z X ; ��� RbTUX Z PBd J_^ Z J 4ABCPVY Z TUh�JUR p Z S\N�X9R [ Z S\[6J_^ Z J

cos−1

(

2 − 22n

2

)

≤ C <π

2

��������� � 4 qn > m > 0 ] J_^DR�S (1 + λn)

1n < (1 + λm)

1m

q H�I Z XaX λ > 0�

!$#&%D%���� l RrJF (λ, n) = (1 + λn)

1n

�(f ^DR�S

∂F

∂n=

λn ln(λn) − (1 + λn) ln(1 + λn)

n2(1 + λn)1− 1n

�����

�P;�} 450�Bf(t) = t ln t

b_��F/0L7f(t)

CN4s890 d ?D0L2L4�CN79��;<7(0, e−1)

2�798~CJ7 d ?D032�4GCJ79�+;<7(e−1, ∞) � }+CABGFR2~KMCN7<=9KM=9K´23B

t = e−1b6ãqg

t ≥ e−1 � BGF/037 f(t) < f(1 + t) �2�798�CNg0 < t < e−1 � }60RF<2��<0 f(t) < 0

2L798f(1 + t) > 0 � CNKOH<.Q:>CJ79�~BGF<2�B �2���23CJ7 � f(t) < f(1 + t)

b!��037 d 0 � ∂F

∂n< 0

b��YF>=94 � F CJ4@8/0 d ?D032�4GCJ79�P2�4�2�gJ=97 d BDC1;<7;>gnb_��F/0�4EBD2�BE03KR037>BäCJ7~BGF/0�.103KOKM2679;�}�gq;>.N.J;�}�4LbÜ>?D;<K

cn = an + bn � }6064,0L0�BGF<2�B CCN4@BGF/06.N2�?D�3034GB@2L79��.10/b���0�B

λ = b/ab�YF/037

c = (an + bn)1n = a (1 + λn)

1n < a(1 + λ2)

12 � IL:~BGF/0+.103KOKM2nb��+0L7 d 0 �

c2 < a2(1 + λ2) = a2 + b2b���F/0L?D0�gq;<?D0 � C < 90◦ � 2�798�BGF>=94 � 4ABC

CN42 d =<BD0��G2�79��.J0�8ub©<:~BGF/0��32�} ;>g ;<4GCJ79034 � }60�F<2��<0cos C =

a2 + b2 − c2

2ab=

a2 + b2 − (an + bn)2n

2ab

=1 + λ2 − (1 + λn)

2n

2λ�

¥@03�9790g(λ, n) =

1 + λ2 − (1 + λn)2n

bc H9H<.Q:>CJ79�sBGF/0�.J0LKMKO2��A}+CQBGF

m = 2�G� }60P450�0sBGF<23B g(λ, n) > 0gq;<?�2L.A.

λ > 02L798gq;<?!23.N.

n > 2b � 0 d .J23CJK BGF<2�B

limn→∞

(

g(λ, n) − g(1, n))

≤ 0gq;<?!23.N.

λ > 0b

Ü�CN?[4GBä79;�BE0PBGF<23Bg(1, n) =

2 − 22n

2

gq;<?ä2L.A.nb��+0L7 d 0 � lim

n→∞g(1, n) =

1

2

bpE�"6���$�b

0 < λ < 1b

�YF/037lim

n→∞(1 + λn)

1n = 1 �

F/0L7 d 0 � limn→∞

g(λ, n) =(1 + λ2) − 1

2λ=

λ

2

b�YF>=94 �

limn→∞

(

g(λ, n) − g(1, n))

=λ − 1

2< 0

bpE�"6�� �9b

λ > 1b

�YF/037lim

n→∞(1+λn)

1n = λ �

F/037 d 0 � limn→∞

g(λ, n) =(1 + λ2) − λ2

2λ=

1

b�YF>=94 �

limn→∞

(

g(λ, n) − g(1, n))

=1 − λ

2λ< 0

bpE�"6�� �9b

λ = 1b�YF/037

limn→∞

(

g(λ, n) − g(1, n))

= 0b

�YF>CN4 d ;<KOH<.10�BE034�BGF/0+H9?D;/;>gu;>gY;>=9? d .N2LCNK#2�I9;3�<09b�P;�} }60)}+CN.A._4,F/;�}�BGF<2�B � gq;<?P0L2 d F��� �0�8 n � BGF/0MgJ=97 d BDCJ;<7 g(λ, n)

F<2L4+CAB[4KO2¡ �CNKM=9K }6F/0L7λ = 1

b!Ü>;<?n ≥ m � }60�F<2��<0

(

g(λ, n) − g(1, n))

−(

g(λ, m) − g(1, m))

=1

(

1 + λm)1m − (1 + λn)

1n

)

≥ 0 �

�����

=94�CN79�PBGF/0�.103KOKM2nb���F/0L7g(λ, m) − g(1, m) ≤ lim

n→∞

(

g(λ, n) − g(1, n))

≤ 0 �2�798~BGF>=94 � g(λ, m) ≤ g(1, m) � gq;<?!23.N. m bÜ�CN7/2L.A.Q: � 79;�BD0äBGF<23B lim

λ→∞g(λ, n) = 0

bWã1B�gq;>.A.1;�}�4nBGF<23B9BGF/0�?[2�79�30@;>gg(λ, n) �

2�4@2+gJ=97 d BDC1;<7O;>gλ � CN4 (0, g(1, n)

]

=

(

0,2 − 2

2n

2

] b��YF>=94 �cos−1

(

2 − 22n

2

)

≤ C <π

2

b23� ��� ��� �$+���- 587 ��� E � � �C><2���2�� � � � A)9;��!<���#A � '������

(��' �=� �'O (P)'���-FO *�P

)� E � 9#���,�8 � � � 9

�8.3>9?2���� � ��A�>�� � ���"� � A�� �(��'��=� �=O P�'���-�O *�P�'���- ' ��'��=� � ' �3��� �"!#� �$��� �"� ��'��=�@O �3P

)�4E� ���2�� 9 �

�F93 )> � � � ;93 � �8A�� ��� �)' �<� � � �F�#�,+ ��� ���,� 7 ��� � -?!#�"'#� �$��� E�� ���� �<' �#���<A � ���<B6'���@ �� # 1����� � �;9#���#� A ��� !<-������ A $:�����"������ �$���#� � '�� �F�#�,+ ��� ���,� 7 A���' � ' B6'<�*�<� A2��� A � �*2 � 9#� E �(2(9#� ��. � � � � A9�'����������<' �$���' �����-�����A)E�2�A � ��2

(��' �=� �)O P '���-&O *�P

)��$ 2 ��� � � 9���2 � � � A ����� ! � � ����� ��7�B6�<'����,!�B�A

�$�<����58�*!#� @�A�2 ! ���$� � ' �>� ��9�9?2!� ��.�����2�� �#� ��A �F�#�,+ ��� ���,� 7 ���)2��,5����=� ' A � -8BC�����,���<A(2�> �� �2�E ��2! �; �� � �E � ���A >?� � 9�' �?�,-8��A���� A � ��2 � � ���-+ $ 2�� ��2 ��A � ��� � ���� � A�E �#� �<' � � ������2 �:.�� ��9�����A� �#+���� A(� 2�A�� �*2

(��' �=� �=O (P '���-KO *�P

)�(2 ��� 9 � � �5����� ���A)����!#-������ A4E��<� ���� ���<�$���0�"��� 2(-�+�' ����� +BC�������� �#�"�$��������' ��- 2 �=� A �4� % �?����@�A �'��A�� ��2

(��' �=� ��O (P' ��-/O *�P

)� 3."� ��� �2� �5� 2 � A � ��������' �,���#� @�� A

����������(��' �=� �@O P;'���-=O *�P;'���- ' ��' �=� � ' �?��� �"!#� �$���C�"� ��' �=��O �3P

)� � � � � � � � � .1$2� � � � A � � ���@8' A � � A

� �*2(��'��=� �@O P;'���-0O *�P

)�1� � � � 9��;.1$� ( A;>?�,� � ' �F�#�,+ ��� ���,� 7 A � '>� � � '<-8' A�E�2�A � �*2 � ���,7 �: ��A1��� � @

E���B6B !��#�,� 76E�� � �$� �<�*A!$ � ���,��� � '<+ ���<A � A�� �*2 ��'���-6����� ����� ���������.

��������������������������� �"!$#&%�'(%�)�*�+-,�.0/ %�)21�34%65�*7398�39.�: ;=<>:�?@: )2:�A�3B;DC7:�'>:�EF�& P Q�R S IWR TWJ Z S\N�X9R ABCD

KnPaJ_^ Z IWR Z S ] X9R�J E Z S\[ F6DR-i\H�PVS�J�YMH�S Y&PV[\R�Y

AB Z S\[ AD ] IWR Y&i$R TWJWP�Q�RrX�j ] Y&h$T&^-J_^ Z J [CEF ] = 1

3S ] KM^DR IWR

[PQR][DR S\H�JWR Y

J_^DR Z IWR Z H�q 4PQR�

t\IWH Q�R�J_^ Z J ∠ECF ≤ π6

�5�%� ��V34%�E ,�.��5�* � A�* ���s#L)@ V:�ED:�+r3����;7� E�3�� *�#�)_3 �9.0%�� 5�: #W:��&*,� %�; 5 :�#�:��_*,� %�;$� %�)7E�39:�:�E +�M*r# � *%+�%*��3VE\:b:�E +"#r3 ��1>%��; !$%� 9A�� %�8e8 ��E�3 �9. � %� � 4* +�*�;b3VE��4*r#8�b:(� *�ED;0�#�;8� 5 ���

� RrJa = AB ] b = BC ] α = ∠BCE ] θ = ∠ECF Z S\[ β = ∠FCD

.........................................................................................................................................................................................................................................................................................................................................................................................................................................................................................................................................................................................................................................................................................................................................................................................................................................................................................................................................................................................................................................................

................................................................................................................................................................................................................................................................................................................................................................................................................................................................................................................................................................................................................................................................................................................................................................................................................................................................................................................................................................................................................................................................................................................................................................................................................................

..

.

..

.

..

.

..

.

..

.

..

.

..

.

..

.

..

..

.

..

.

..

.

..

.

..

.

..

.

..

.

..

.

..

.

..

..

.

..

.

..

.

..

.

..

.

..

.

..

.

..

.

..

.

..

.

..

..

.

..

.

..

.

..

.

..

.

..

.

..

.

..

.

..

.

..

..

.

..

.

..

.

..

.

..

.

..

.

..

.

..

.

..

.

..

..

.

..

.

..

.

..

.

..

.

..

.

..

.

..

.

..

.

..

.

..

..

.

..

.

..

.

..

.

..

.

..

.

..

.

..

.

..

.

..

..

.

..

.

..

.

..

.

..

.

..

.

..

.

..

..

.

..

.

.

.

..

.

..

.

..

.

..............................................................

......................................... . . . . . . . . . . . . . . . . . . .....................

. . . . . . . . . . . . . . . . . . . .............................................................

.............................................................

α

.

.

.

.

..

.

.

..

.

.

..

..

.

..

.

..

..

.

β.........................

θ

A B

CD

E

F

a

b

�����

�YF/037cos(α + β) = cos(π

2− θ) = sin θ

2L798)}60�F<2��<0ab = S = 3[CEF ] = 3

(

1

2· CE · CF · sin θ

)

=3

2· b

cos α· a

cos β· sin θ

=3ba sin θ

cos(α + β) + cos(α − β)=

3ba sin θ

sin θ + cos(α − β)

b��037 d 0 � sin θ = 1

2cos(α − β) ≤ 1

2

b���F/0L?D0�gq;<?D0 � θ ≤ π6

b+ß<�<=923.NCQB�:�F/;>.J8/4�}6F/0L7α = β = π

6

b23� ���&��� �,+ ��- 587���� � � ��2�� � � ��2 ��E� ! 2 � A E<' � ' � ��!#��� ' A�� ' � �$�����"' A� ��' � ���4��� E � � �><2���2�� � � � A 9;��!#���<A � '������ � 9?2 � E,����E� > � � �� �� 93 ��2�� ;A/��. >�. � B�� � �$� ����� '�� � A� �' � � '<-�� � �,-�A

� ��' � ��� E � 9#���,�8 � � � 9 ��.�>9?2���� � ��A&>�� � ���,� � A � � �CE ����� E ��9���� � A � � ������!�� � �#��!<� ������� �#� '#�"�E�� � �$� �<�*A(� � �?� � �#A � ;A � ��2 ��9#��E �)2�9#� ��. � � � � A�9�' ��� ��� �<' �$���� �����-�����ACE�23A � �*2 � $�2 �5� � � 9��2 � � � A ����� ! � � ����� ��7�B6�<'����,!�B�A>�$�<����58�*!#� @�A 2 ! ���$� � ' � � � 9 � � 2 .�82 ���2 � � A � ' BC-����<A6E �=A� �*2 �>� �;9�9?2 � �?. � �*2�� � � ��A ���<�$+���� ���$�$7 ��� 23�$5����=� ' A � -8BC�����"���#A 2�> � � �,��2! � ��9#� � � � 9�A�F�#�,+ ��� ���,-8'#-�E�2 � E � A >;!#������� 2�� ������A 23���<����� � ��' � 2 ��� 9 � � �5����� ���AC��� !<-������ A E��<� ���� �F�#�,����"��� 2�-�+*'�������BC����� ��� �#�"�$������� ' ��-123�=� A �4� % ������@8A �'��A � ��2 �0� . �8. �5� � � � ��A�7 ' �,�"5���B6BC�*� A����� �4��� ����� � '���-8� � � E��82�9#� ��� � E �/��A )�"�"� + + ��� + �!#��� �,�"@�� �F�#�,+ ��� ���,� 7 A�� ' �#-���5?!���� A ;����B6' �?7 ���� � � � �� �> "� � ��2(9 9��;>1�� A ��' ��� '���-���� A � ��' � ���1� � � � 9.�;."$6 ) ;A>?�$� � ' �F�#�,+ ��� ���,� 7 A � ''� � � '#-8' A;E�2�A� �*2 � � � � �,7 �(2� ;A���� !<-������ A1� ��� � � � � � E�� � �,� � � '#�"��� ����� �,� !<�,�*A!�����������"� A� � ��'���-6����� ����� ���������.

$��<������B6BC�����"��-����<'<�;����� �����<5��$��B�A�'��)��� '<�,��-�A-��<���)���<��� �<�,�"!#-�� �"' ������% �������E

' ��-F

� �,���� � ��� � � �����

AB' ��-

AD (' �4� � �������-&�"� ����� ��� ��BC����� �

AB'���-

AD). �4��� ����� �����<������-���-&�,�

� ��+������ � � '<�,�&����� ��������$5��,�&�,'������ . 2�� �)�<� ����� ��� �$+���� �(� ���,�"!#-�� � ������� ����� ��������

)� B��?� �$�"�$� � 7 '���� ! BC��-

���<'<�E

' ��-F

� �$�4���&����� ��� ��BC����� �AB

'���-AD

A������ ������ �,+ �*� 7�.

��������������������������� �O!$#&%�'(%�)�*�+-,�.0/ %�)21�34%65�*7398�39.�: ;=<>:�?@: )2:�A�3B;DC7:r'>: E �& P Q�R SoJUIWP Z S\N�XVR ABC

KnPaJ_^oPBS\T_R�S�JUIWRI ] TUPBIWTUh\d-TUR S�JUIWR

O ]$Z S\[0T_R�S�JUIWH�P9[G ]Y&h\iDi$H�Y2RbJ_^ Z J ∠AIO = 90◦

� t\IWH�Q�RbJ_^ Z J IG‖BC�

5�%� ��V34%�E ,�."� 1 *�'>#&%�'(%�)�*r# �� R [\R�S\H�JUR J_^DR TUPBIWTUh\d-TUPBIWTUX9R H�q

4ABC6 j

Γ� � RrJ

E6DRkJ_^DR Y2R�TUH�S\[

PVS�JWR I�Y2R�TWJWPVH�SnH�qAI

KnP�J_^Γ ]rZ S\[`X9R�J D

6DRJ_^DR-PVS�JWR I�Y2R�TWJWPVH�S H�q

AE Z S\[ BC� � RrJ

AGd-R RrJ

BC Z J M� l PBS\T_R

GPVYMJ_^DR

T_R�S�JUIWH�P9[ Z S\[ AMPVY Z d-R [�P Z S ] AG :

GM = 2 : 1�

� RM^ Z Q�R ∠ABI = ∠DBI Z S\[

∠BAE = ∠BAI = ∠IAC

= ∠EAC = ∠EBC

= ∠EBD�

..

..

..

.

..

..

.

..

..

..

.

..

..

..

.

..

.

..

..

..

.

..

..

..

..

.

..

..

..

.

..

..

..

.

..

.

..

..

..

.

..

.

..

..

..

..

..

..

..

..

..

..

..

..

..

..

..

..

..

..

..

..

..

..

..

..

..

..

..

..

..

..

..

..

..

..

..

..

..

..

..

..

..

.......................................................................................................................................

.............................................................................................................................................................................................................................................................................................................................................................................................................................................................................................................................................................................................................................................................

...................................................................................................................................................................................................................................................................................................

.

.............................................................................................................................................................................................................................................................................................................................................................................................................................................................................................................................................................................................................................................................

..

..

..

..

..

..

..

..

..

..

..

..

..

..

..

..

..

..

..

..

..

..

..

..

..

..

..

..

..

..

..

..

..

..

..

..

..

..

..

..

..

..

..

..

..

..

..

..

..

..

..

..

..

..

..

..

..

..

..

..

..

..

..

..

..

..

..

..

..

..

..

..

..

..

..

..

..

..

..

..

..

..

..

..

..

..

..

..

..

..

..

..

..

..

..

..

..

..

..

..

..

..

..

..

..

..

..

..

..

..

..

..

..

..

..

..

..

..

..

..

..

..

..

..

..

..

..

..

..

..

..

..

..

..

..

..

..

..

..

..

..

..

..

..

..

..

..

..

..

..

..

..

..

..

..

..

..

..

..

..

..

..

..

..

..

..

..

..

..

..

..

..

..

..

..

..

.

..

..

.

..

..

.

..

..

.

..

..

.

..

.

..

..

.

..

..

.

..

..

.

..

..

.

..

..

.

..

.

..

..

.

..

..

.

..

..

.

..

..

.

..

.

..

..

.

..

..

.

..

..

.

..

..

.

..

..

.

..

.

..

..

.

..

..

.

..

..

.

..

..

.

..

..

.

..

.

..

..

.

..

..

.

..

..

.

..

..

.

..

.

..

..

.

..

..

.

..

..

.

..

..

.

..

..

.

..

.

..

..

.

..

..

.

..

..

.

..

..

.

..

.

..

..

.

..

..

.

..

..

.

..

..

.

..

..

.

..

.

..

..

.

..

..

.

..

..

.

..

..

.

..

..

.

..

.

..

..

.

..

..

.

..

..

.

..

..

.

..

.

..

..

.

..

..

.

..

..

.

..

..

.

..

..

.

..

.

..

..

.

..

..

.

..

..

.

..

..

.

..

.

..

..

.

..

..

.

..

..

.

..

..

.

..

..

.

..

.

..

..

.

..

..

.

..

..

.

.........................................................................................................................................................................................................................................................................................................................................................................

.

..

.

..

..

..

.

..

..

..

.

..

..

.

..

..

..

.

..

..

..

.

..

..

..

.

..

..

..

.

..

..

..

.

..

..

..

.

..

..

..

.

..

..

..

.

..

..

..

.

..

..

.

..

..

..

.

..

..

..

.

..

..

..

.

..

..

..

.

..

..

..

.

..

..

..

.

..

..

..

.

..

..

..

.

..

..

..

.

..

..

.

..

..

..

.

..

..

..

.

..

..

..

.

..

..

..

.

..

..

..

.

..

..

..

.

..

..

..

.

..

..

..

.

..

..

..

.

..

..

.

..

..

..

.

..

..

..

.

..

..

..

.

..

..

..

.

..

..

..

.

..

..

..

.

..

..

..

.

..

..

..

.

..

..

................................

A

B CD

E

GI

M

O

Γ

.

..

.

.

..

.

.

.

..

.

.

...

...............................................

.................................................

.......................................

.

.

.

..

.

..

.

.

..

..

.

..

.

...

..

..

..

..........

··

f ^DR�S∠BIE = ∠BAI + ∠ABI = ∠EBD + ∠DBI = ∠EBI ]�Z S\[e^DR S\TUR

BE = EI� l PBS\T_R

∠BAE = ∠EBD ] P�Jbq H�XBXVH�K�YnJ_^ Z J 4ABEPBYMY&PBdoPaX Z I`JUH

4BDE ]�Z S\[6J_^DR�IWR�q H�IWRAB : BD = AE : BE = AE : EI

�����

� 0�2�?D0T��C �<0L7'BGF<2�B∠AIO = 90◦ �

BGF<23B)CN4 � OI ⊥ AEb �YF>CN4�CJKMH/.AC1034BGF<23B

AI = IE � 2L798�F/037 d 0 AE : EI = 2 : 1b���F/0L7

AB : BD = 2 : 1b�3CJ7 d 0

∠ABI = ∠IBD � }60��30�B AI : ID = AB : BD = 2 : 1bá�YF>=94 �

AI : ID = AG : GM � }6F>C d F�CNKOH<.NCJ0L4!BGF<2�B IG ‖ DMb���F>=94 � IG ‖ BC

b23� ���&��� �,+ ��- 587���� � � ��2�� � � ��2 ��E� ! 2 � A E<' � ' � ��!#��� ' A�� ' � �$�����"' A� ��' � ���4��� E � � �><2���2�� � � � A 9;��!#���<A � '������ � 9?2 � E,����E� > � � �� �� 93 ��2�� ;A/��. >�. � B�� � �$� ����� '�� � A� �' � � '<-�� � �,-�A

� ��' � ��� E � 9#���,�8 � � � 9 ��.�>9?2���� � ��A&>�� � ���,� � A � � �CE ����� E ��9���� � A � � ������!�� � �#��!<� ������� �#� '#�"�E�� � �$� �<�*A � � �?� � �<A.� A � ��2 �.$�2 �5� � � 9 ��2 � � � A ����� ! � � ����� ��7�B6�<'����,!�B�A�� ���<��58�*!<�"@8A&2�!���� � � ' �� ��9�9?2!� �?. ����2�� �#� ��A ���<�$+���� ���$�$7 ��� 2��,5����=� ' A � -8BC�����"���#A42�> ��2 ��� 9 � ���5����� ���A4��� !<-������ AE��<� ���� ���<�$��� �"��� 2(-�+�' ������BC�����&��� �<�,�,������� '���- 23�=� A �4� %8�?����@�A �'��A � �*2 � � . �8.��� ��� � ��A7 ' �$� 5���B6BC�*� A � ��� �4��� ����� � '���-8� � � E��#2(9#� ��� � E �/��A )�"�"� + + ��� + �!#��� �,�"@�� �F�#�,+ ��� ���,� 7 A�� ' �#-���5?!���� A;����B6'���7 ��� � � � 9��;. $� ( A)>?�$� � ' ���<�$+���� ���$�$7 A���' � � � '#-8' A(E�2�A � ��2 ����� 7 �: ��A ��� � @�E���B6B ! �<�$�$7E�� � �$� �<�*A!$ � ���,��� � '#+����#A � A � ��2 ��' ��-���� ����75 � �)2�9���A;>;!<� ��' ������� A9;��B6'��#� '�.

�<� + ��� ' ����� �,+ ��� �3�<58������+���- � ��'#��������������+ ��� ����� �' � ��� �$�*!#�:=?� �IG ‖ BC

A�� �����∠AIO = 90◦ .

���������#+ �(���#� ��A<��! ���<��� +���� ���)� ���3���"� ����� �:� ����� ' ��� ��'�� ' ��� ' � �����?�����!�"��'#��! ����-���� �"!#� �$���. !��=� ������BC�����*AIG ‖ BC

� �' ��-&���<� 72a = b + c

A�% �������aAbA'���-

c' ���:����� �,��� �<� ��� ���3� ��� ���$-���� .>� �#� � %(' ��'

� + �%&M� ����<� ��M� ��� ����B �������<.(�<��� � ��������2�(W�AF���)� ���(��� ��� � ���,��-:+ ��� ���,���4� � � ����&��� ���W<. ���)� ���������� �"��������������AF% � ���<'��<@ � ���� ����.

)2 BC��� ��! ' �(E��#+*'��:�"� ' � B6�:���<'<� A �"���4� � � ' � � �,��� % �������2a = b + c

A��<� ����� ���*� '<� �,���<� �<� ��� ' ���� �$+����:� � � ���� ���� �� 5 ���W ' ��-6� �����"� � �,�#%)� � � ��� �"�������������&=� �(W . 3. + � . A���H�����$ ���(�.!����������%���D���3 � A�� � �C��-��$� �$���<A1�<' � � ��A � ��.��������������<.� (W . 3. + � . A���H�����$ ���(�.!��"���3 �(������%��� �$� � A1�<'�� � ��A�� ��.�� ������� ����A��� �#.� (W ��. � �<'�� � ��! � �#A"!����*�+ �(%; ��.!���������%&�D���#�$ %!�+ ��� %&�D���#�$ A��-��,�,��� � ' � � � � A ���� ��=A�� ��.����������8.

�����'&O��������������(D��� �"!$#&%�'(%�)�*�+-,�.�� 3V1�:, 9."� *�E.��� * ; � #W:�)r%*�r;�!=%�8e: E�3V:\�44S 4ABC ] [DR S\H�JWR�J_^DROY_P9[DR Y�6 j

a ] b ] c ] J_^DROY�j�dedeR�[�P Z SDY 6 j sa ] sb ] sc ]Z S\[6J_^DR�TUPVIWTUh\doI Z [�PBh\Y 6 j R

� t\IWH Q�R�J_^ Z J

bc

sa

+ca

sb

+ab

sc

≤ 6R�

5�%� ��V34%�E ,�. �8� 3u+�3 � �� #&+��r3V; )��-��+\*�E��9; 6* ),�4*r#UE � 3��W1�3�+�: E � E�3�� *�#�)_3 �9. ;<>:� 9: 8e:�� %D%�; �*) ;��(5%���

� Rsi\IWH�Q�R Z d-H�IWRmN�R�S\R I Z X PBS\R>:�h Z XaPaJ4j �+-, ��.-/ ��� � � RrJ

ABC6\R Z JUI�P Z S\N�X9R Z S\[eX9R�J

M ] N ] P ] 6DR Z IB6�P�JUI Z I4j�i$H�PBS�JWY@H�S6J_^DRnXaPVS\RY7R Nrd-R�S�JWY

BC ] CA ] BA ] IWR Y&i$R TWJWP�Q�RrX�j �f ^DR�Sbc

AM+

ca

BN+

ab

CP≤ 6R

�.........................................................................................................................................................................................................................................................................................................

..

..

..

..

..

..

..

..

..

..

..

.

..

..

..

..

..

..

..

..

..

..

..

..

..

..

..

..

..

..

..

..

..

..

..

..

..

..

..

..

..

..

..

..

..

..

..

..

..

..

..

..

..

..

..

..

..

..

..

..

..

.

..

..

..

..

..

..

..

..

..

..

..

..

..

..

..

..

..

..

..

..

..

..

..

..

..

..

..

..

..

..

..

..

..

..

..

..

..

..

..

..

..

..

..

..

..

..

..

..

..

.

..

..

..

..

..

..

..

..

..

..

..

..

..

..

..

..

..

..

..

..

..................................................................................................................................................................................................................................................................................................................................................................................................................................................................................................................................................................................

.

.

..

..

.

..

..

.

..

..

.

..

.

..

..

.

..

..

.

..

..

.

..

..

.

..

..

.

..

..

.

..

..

.

..

..

.

..

.

..

..

.

..

..

.

..

..

.

..

..

.

..

..

.

..

..

.

..

..

.

..

..

.

..

.

..

..

.

..

..

.

..

..

.

..

..

.

..

..

.

..

..

.

..

..

.

..

..

.

..

.

..

..

.

..

..

.

..

..

.

..

..

.

..

..

.

..

..

.

..

..

.

..

..

.

..

.

..

..

.

..

..

.

..

..

.

..

..

.

..

..

.

..

..

.

..

..

.

..

..

.

..

.

..

. .................................................................................................................................................................................................................................................................

A

B CM

NP

.

.

.

.

.

.

.

.

.........α

..

.

.

.

.

.

.

.

.

.

......β

.

.

.

.

..

..

..

...

.

.

.

γ

!$#&%D%��10 � R�Jα = ∠AMB ] β = ∠BNA ] γ = ∠APC ]�Z S\[6X9R�J

∆6DR�J_^DR Z IWR Z H�q

4ABC��� R�^ Z Q�R

1

2a · AM sin α = ∆ =

abc

4R]

����£

��037 d 0 � bc

AM= 2R sin α

b��3CNKMCA.J2L?D. : � ca

BN= 2R sin β

2�798 ab

CP= 2R sin γ

b�YF>=94 �bc

AM+

ca

BN+

ab

CP= 2R(sin α + sin β + sin γ) ≤ 6R

b23� ��� ��� �,+ ��- 587*�� �- � � ��2�9 � ��2 �)2 � �E<A ���<�$+���� ���$�$7 ����� '�� ' ��� +�� A � '�� ' ��� +�� A&>�����<� ' ' ��-

� �����*� �#�#+�� ��' ������E � � �&><2���2�� � � � A 9;��!<���#A � '������ � 9?2 � E,� �<E� > � � �� �� 93 ��2�� ;A ��. >�. � B�� � �,� ����� ' � � A <' � � '#-�� � �$-�A � ��' � � � E �49#� �-�8 �� � � 9 ��. >9?2���� � ��A E#� � �"�"���#A >�� � ���,� � A � � � $�2 �5� � � 9��2 � � � A ����� ! � � ����� ��7�B6�<'����,!�B�A � ���<��58�*!<�"@8A 2 ! ���$� � ' � �8 � ���� � � ������� 23A ��'#%(' ��'�@�� A0�*' ��'����2 ��� 9 � � ��� ���� ��A���� !<-������ A;E��<� ���� �F�#�,���>�"���2(-�+�' ������BC�����;��� �<�,�,��������' ��- 23�=� A �4� % �?����@�A �'��A� �*2 �#� 2 � � � . � ��2� � ���< ��< ��A�2�� �����<��A ����������

(�,% � ��� �"!#� �$�����

)� �5�"7 �: ��A#��� � @CE���B6B ! �<�$�$7

E�� � �$� �<�*A!$ � ���,��� � '#+����#A � A � ��2 ��' ��-6� ��� ����� ���������.23� �3�<����������� �$+���� � �����#+ ��-C��������� � � � �<' �'���- ����BC�4���<�,��- � ��'#���$� %(' � ���<�3������������' ��7&�,�C� ' @��

��7�B6BC��-�� '��<� . >�'#� ' � � �,�����<�,��-6���<'<�;����� �����<����'#��� !�' � � 7 � �$+����)' ���$����� �<���3����� ! �,��=

max

{

bc

sa

, ca

sb

, ab

sc

}

≤ 2R�

�$��-�����-�A��$��� � �,�,��' �#� ����B � ��� '#5��#+�� �����<���;� ��'#�

max

{

bc

AM

, ca

BN

, ab

CP

}

≤ 2R,

% �������AM

ABN

ACP

' ��� ' �?7�� ������� ��� +*� ' ������� 4ABC.

������� � ���� ����������������

�������! "#�$�&%(')"��*�,+�"�-. "�/�&%('02143 57648902:<;�8=3 5?>A@9BDC9E;�12FHG=5I;J>A87KL;�:<;�8=3 5?>A@NMPOQ8?>AGI;SRAT C9T4UWV2FJXZY4;�8

[]\<^�_ "#�$�&%(')"��*�,+#` \<^ 'a26H>QbA143 bAcd0Q143 5I648=@9BDC9E;�12FHG=5I;J>A8=@eK7fg64bQ12FN57;N>Q8?@NMhRAE;�64iA6Hj 1DU�FN>2klE;nmoa48?;W1Q;N8?3 GLprqsT OhTAt$F�@epH;Jj j02143 57648?@u;N:v;N8?3 573ABDC9E;�12FHG=5I;J>A87KL;�:<;�8=3 5?3=M,qsT w#T4U�FWbQ12@NxyC9T 0zTHw{626212876WXSxyOQ8?>AGI;DRQT C9T2UWV4FNXsY2;N8

"#�$�&%(')"��*�,+�"�-| "�/}�&%('a26H>QbA143 bAcd0Q143 5I648=@9BDCuE;W12FWG?57;N>Q8?@eK7f 64bA12FJ5I;J>A8=@NMh~4FN578?3 GLpnUN>A8?8�Y<m�CyFJk432�lFHp�3 j0Q143 5I648=@u;�:<;�8=3 5?3yBSCuE;W12FWG?57;N>Q8?@eKL;N:v;N8?3 573�MP~AVH3 j 3 ir�L64bQc4x��L;N�*�Z3 cNV2F�:�x

��T ~,T2qh8?64@L@L:<F�bzx��PbA128?;n�,V4FWbQc4xQ�sFW62pW3yU�FJ5I6yx4�2Y28=>A@���@I3 F